You are on page 1of 153

EVIDENCE LAW AND THE SYSTEM

I. PRELIMINARY MATTERS Stages of a Trial: 1. Jury Selection: We ask the jury their age, profession, address, etc. You are looking for relationships with parties or witnesses and viewpoints of jury members. 2. Opening Statement: the lawyers version of what will happen during the trial. 3. Presentation of Proof: evidence that is admitted at trial which determines how the case should be decided. 4. Trial Motions: Motion in Limine: motions desired to introduce evidence or to keep it out before it is admitted. 5. Closing Arguments 6. Instructions: Explain to the jury what they should be ruling on. 7. Deliberations 8. The Verdict 9. Judgment and Post-Trial Motions 10.Appellate Review: one of the primary reasons for reversal is a evidentiary mistake. The Mechanics of Trial: Evidence is presented at trial in parts, and in the following general order, subject to change under certain rules, such as FRE 106 and FRE 611(b). 1. Plaintiff (or Prosecutor) presents his case-in-chief, then rests; 2. Defendant presents his case-in-chief, then rests; 3. Plaintiff (or Prosecutor) presents his case-in-rebuttal; 4. Defendant presents his case-in-rebuttal (sometimes called case-inrejoinder) 5. Each side presents further cases-in-rebuttal (sometimes called case-inrejoinder) Order of Examination: 1. Direct Examination by the calling party; 2. Cross Examination by the adverse party; 3. Redirect Examination by the calling party; 4. Re-cross by the adverse party; 5. Further redirect and re-cross may be necessary The record consists of: The Pleadings Filed Documents Record of Proceedings Exhibits: anything tangible other than an eye witness testifying Docket Entries: court has an index to the file that tell you the kind of document and when it was done.

Evidence Outline

Page 1

Types of Evidence: Testimonial: testimony of the witness Real Evidence: tangible things directly involved in the transaction or events in litigation. (ex: the weapon used in the homicide) Demonstrative Evidence: tangible proof that in some what makes graphic the point to be proved. (ex: diagrams, photographs, etc.) Writings: (ex: laboratory reports and medical records) II. SCOPE
OF

DIRECT/CROSS-EXAMINATION

FRE 611(b)Scope of cross-examination. Cross-examination should be limited to the subject matter of the direct examination and matters affecting the credibility of the witness. The court may, in the exercise of discretion, permit inquiry into additional matters as if on direct examination. Subject Matter of Direct: determines the subject matter of the cross. Courts can interpret the subject matter of direct in 3 waysJudge may take any of the following views of whether to admit the testimony: (1) Narrow interpretation: confines the cross-examiner to points raised on direct; (2) Broader interpretation: lets the cross-examiner ask about the transaction described on direct; and (3) Even broader interpretation: lets the cross-examiner ask about any issue affected by the direct. (i.e.: any issues that are raised on the direct can be asked on cross.) Matters Affecting the Credibility of the witness: goes to a witnesses motivation to lie. Credibility includes anything that is properly challenged through impeachment, which means that it includes both intentional (the witness lied) and involuntary (the witness was mistaken) lack of accuracy. It is important to note Rule 611(b) only allows you to question the witness regarding matters that affect the credibility of that witness who is then testifying, not someone else. Permitting Inquiry into Additional Matters: Even if a court finds matters to be beyond the scope of direct, the court can choose to allow the testimony of these matters (allowing all the information from each witness to be admitted all at once.) The cross-examining party can in essence change hats to become the direct examining party (asking questions beyond the scope) sooner rather than later. In a simple case, this prevents having to excuse and recall the same witnesses over and over.

**The answer to the question controls the subject matter of the direct. If the question asks one thing, what was said determines the scope.** Problem 1-A: How did it Happen: the Scope of direct/crossexamination

Evidence Outline

Page 2

At an intersection, 2 cars collidea yellow Fiat driven by Abby (Carl was the passenger), and a blue Buick driven by Eric. Abby sues Eric for personal injuries and property damage. During her case-in-chief, Abby calls Carl who testifies on direct that the Buick ran the red light. On cross-examination, Erics counsel asks Carl (1) if Carl and Abby are seeing each other; (2) whether Abby was looking out of the back window of the car at the time of the accident; and (3) whether Abby had drank 3 glasses of wine just before the accident). To each question Abbys attorney objects that the statements are beyond the scope of direct. Answer: Preliminary Questions to Answer under FRE 611: (figuring out the relative position of the testimony) Who is presenting the evidence? Plaintiffs witness What part of the trial are we in? Cross examination in Plaintiffs case-in-chief. Whose witness is it? Plaintiffs What kind of witness is it? Fact Witness What are they presenting? Question 1: This goes directly to the credibility of the witness (how truthful are they? Should we believe what is being said?) A social relationship might incline Carl to help Abby. Question 2: This will depend on how we interpret the scope of direct. The point raised on direct was the color of the light (i.e.: that the color was red and the D ran it) (1) Narrow interpretation: the cross would be beyond the scope (2) Broader interpretation: this would include anything that was going on at the intersection, in which case the question is proper. (3) Even broader interpretation: question is proper. Question 3: The fact that Abby was drinking wine is not in the subject matter of the direct. The direct is focused on Erics actions, not Abbys. Because we are looking solely at the conduct of Eric, Abbys conduct is beyond the scope. If the subject matter of the direct was causation of the crash But, this testimony would be the subject matter of direct. FRE 611(c) Leading questions Leading questions should not be used on the direct examination of a witness except as may be necessary to develop the witness' testimony. Ordinarily leading questions should be permitted on cross-examination. When a party calls a hostile witness, an adverse party, or a witness identified with an adverse party, interrogation may be by leading questions. III. MAKING
THE

RECORD & ADMITTING EVIDENCE

FRE 103Rulings on Evidence (a) Effect of erroneous ruling. Error may not be predicated upon a ruling which admits or excludes evidence unless a substantial right of the party is affected, and

Evidence Outline

Page 3

(1) Objection. - In case the ruling is one admitting evidence, a timely objection or motion to strike appears of record, stating the specific ground of objection, if the specific ground was not apparent from the context; or (2) Offer of proof. - In case the ruling is one excluding evidence, the substance of the evidence was made known to the court by offer or was apparent from the context within which questions were asked. Once the court makes a definitive ruling on the record admitting or excluding evidence, either at or before trial, a party need not renew an objection or offer of proof to preserve a claim of error for appeal. (b) Record of offer and ruling The court may add any other or further statement which shows the character of the evidence, the form in which it was offered, the objection made, and the ruling thereon. It may direct the making of an offer in question and answer form. An objection must be made in order to exclude evidence. An objection is also required to appeal the admission of evidence. The objection must be timely and should include a statement of the underlying reason (grounds for the objection). o The general objection: if overruled, a general objection does not preserve for review whatever point the objector had in mind. o Motion in Limine: a ruling in advance used (1) when a party anticipates that particular evidence will be offered to which he will object or (2) when a party anticipates that an item of proof that he plans to offer will meet objection from the adverse party. An offer of proof is required to appeal the exclusion of evidence. A lawyer faced with a ruling excluding evidence must make a formal offer of proof, if he wants to preserve the point for later appellate review, which means demonstrating to the trial court exactly what he is prepared to introduce if permitted (i.e.: an attorney offers to the court what they were going to show and why they were going to show it, to preserve it for appeal). No need to renew an objection: ex: If you have multiple Ps and one P objects, the other Ps dont also need to object to preserve the objection for appeal as it applies to them. Requiring each party to reiterate objections adequately made by similarly-situated parties would clutter and burden the proceedings.

Kinds of Error: 1. Reversible Error: the kind of mistake which probably did affect the judgment. 2. Harmless Error: the kind of mistake that probably did not affect the judgment. 3. Plain Error: the kind of error that warrants relief on appeal even though appellant did not take the right steps to preserve his rights. (i.e.: he did not object or make an offer of proof) 4. Constitutional Error: a mistake by the court in criminal cases in admitting evidence for the prosecution that should have been excluded under the Constitution. Problem 1-B: He didnt object!

Evidence Outline

Page 4

Carl joins with Abby as the second Plaintiff in the suit against Eric. (See problem 1A) The defense offers testimony by police officer Hill based on the measurements of skid marks at the scene, that Abby was traveling at 50 mph (where the posted speed limit was 35 mph). Counsel for Abby objects (1) that the officer is not qualified as an expert in accident reconstruction and (2) evidence of skid marks involve pure speculation (i.e.: this is not something someone can be an expert on (ex: speaking to the dead) or in this case, estimates of speed is not something that should be the basis of expert testimony. (See FRE 702) Carl and Abby lose and Carl appeals. Eric objects that Carl did not object at trial (it was Abby who objected). Can Carl recover? Answer: In this case the Co-Plaintiff (Abby) was the one that objected and not Carl (the party that filed the appeal.) However FRE 103(a) allows Carl to raise this issue on appeal. Carl is not done, however. In order to prevail, having shown there was an appropriate objection (under FRE 103) the party now needs to show there was an error and that error affected a substantial right of the losing party. Was a substantial right of the appellant (Carl) affected? Losing is not enough to show a substantial right was affected. You have to show that by admitting that evidence, a substantial right was affected. (FRE 103(a)) In this case, negligence by Abby in speeding might bar or reduce her recovery, but it will not bar Carls recovery. In order for Carl to show a substantial right was affected, he must argue the jury likely found for Eric because it concluded that Abbys speed caused the accident. o If there is other overwhelming evidence that would still have the court determine what it determined at trial or if this one thing would not have an effect on the ruling than there is no reversible/harmful error. o Remember, even if there is not an appropriate objection, a court may under FRE 103(d) hold a substantial right was affected and that the error is reversible/harmful. But, if an objection was made under rule 103(a), you do not need to reach 103(d).

RELEVANCE
FOREST: Paradigm to solve relevant questions: 1. Preliminary: Who decides if the evidence is Relevant? (FRE 104(a) & FRE 104(b)) a. Judge decides whether to admit the evidence b. Jury decides the weight to give the evidence 2. Relevance: Is this evidence relevant? (FRE 401, 402) 3. Specific Rule (specific rules of exclusion or admission) (FRE 891, FRE 404) 4. Probative Value verses the six factors: do the balancing test FRE 403the more compelling the argument you make the evidentiary hypothesis (story to persuade your side of why the evidence should be

Evidence Outline

Page 5

admitted and what this evidence will show) the more likely it will have a probative value and be admitted. 5. Jury Instructions to Recast the balance. As noted above, in any exam question, relevance should always be the starting point: It is always the threshold question because all areas of evidence law depend on the answer to the relevance question. On the exam, every answer to a question needs at least one sentence explaining why the evidence is relevant in the casei.e.: what is the purpose for this offer of evidence? The purpose for which you offer evidence determines all other aspects of evidence law. I. DEFINING RELEVANCE There are two categories of relevance: LOGICAL RELEVANCE (FRE 401) and PRAGMATIC RELEVANCE (FRE 403). LOGICAL RELEVANCE: FRE 401Definition of "Relevant Evidence" "Relevant evidence" means evidence having any tendency to make the existence of any fact that is of consequence to the determination of the action more probable or less probable than it would be without the evidence.

Relevant Evidence can be referred to as logical relevance: it is an easy standard of admissibilitythe evidence is relevant if it has any tendency to make a material proposition more probable or less probable than it would be without the evidence. (i.e.: Does this evidence help at all to prove a material proposition? Does it make a material proposition more probable than it was before? If yes, it is relevant.) Consequence=material. As noted, the fact must be material (i.e.: any fact that is of consequence to the determination of the action.)

FRE 402Relevant Evidence Generally Admissible; Irrelevant Evidence Inadmissible All relevant evidence is admissible, except as otherwise provided by the Constitution of the United States, by Act of Congress, by these rules, or by other rules prescribed by the Supreme Court pursuant to statutory authority. Evidence which is not relevant is not admissible.

There are four possible definitions of what relevance means, however the FRE take the most liberal definition (more probable than it was before). FRE 401 implicates you are allowed to admit direct (establishes the point for which it is offered) and circumstantial evidence (even if true, fails to support the point in question, because there is an alternative explanation) using either deductive (conclusion necessarily follows) or inductive (conclusion does not necessarily follow) reasoning. That means your support for relevance can be as weak or as minimal as a circumstantial, inductive argument which could be true. (All you need is the minimal level of information that establishes something is more probable than not.)

Evidence Outline

Page 6

We want the ultimate truth to be determined by subjecting the evidence to the tools of the adversary system: cross-examination, rebuttal, and argument, so the FRE has adopted a liberal standard. Evidence which concerns some other time, event or person will generally not be relevant and therefore not admissible. This is because the evidence is too remote to be logically relevant. (However, there are 8 exceptions where the evidence involves some other time, event or person when it will be admissiblesee examples below.)

PRAGMATIC EVIDENCE: FRE 403Exclusion of Relevant Evidence on Grounds of Prejudice, Confusion, or Waste of Time Although relevant, evidence may be excluded if its probative value is substantially outweighed by the danger of unfair prejudice, confusion of the issues, or misleading the jury, or by considerations of undue delay, waste of time, or needless presentation of cumulative evidence. HOW RELIABLE IS THE EVIDENCE? probative value it has The more reliable something is, the more

Pragmatic evidence has a stricter standard of admissibility than logical evidence. Evidence may be logically relevant but we may still keep it out because its probative value is substantially outweighed by auxiliary considerations: Fairness Danger of unfair prejudice Confusion of the issues in the minds of the jury Misleading the jury Considerations of undue delay Waste of time Needless presentation of cumulative evidence The trial judge can exclude even logically relevant evidence when its probative value is substantially outweighed for any of the above reasons. FRE 403 should be mentioned in every evidence question. Your answer should always read, this evidence is admissible subject of course to the courts discretion to exclude it should the court decide that its probative value is substantially outweighed by (insert consideration here) II. WHO DECIDES RELEVANCE? FRE 104Preliminary Questions (a) Questions of admissibility generally. Preliminary questions concerning the qualification of a person to be a witness, the existence of a privilege, or the admissibility of evidence shall be determined by the court, subject to the provisions of subdivision (b). In making its determination it is not bound by the rules of evidence except those with respect to privileges.

Evidence Outline

Page 7

(b) Relevancy conditioned on fact. When the relevancy of evidence depends upon the fulfillment of a condition of fact, the court shall admit it upon, or subject to, the introduction of evidence sufficient to support a finding of the fulfillment of the condition. (e) Weight and credibility. This rule does not limit the right of a party to introduce before the jury evidence relevant to weight or credibility. Simple Relevance: The trial judge alone determines questions of admissibility under FRE 104(a)i.e.: whether a particular point is consequential (material) within the meaning of FRE 401. Only a judge is qualified to decide this point because it turns on substantive and procedural rules. The judge also makes all findings of law before the matter is given to the jury and fact (if no reasonable person could conclude otherwise). o The Judge decides whether something is relevant in the overwhelming majority of cases. o The judge is simply required to find that a jury could find the evidence true/false. Conditional Relevance (104(b)): the jury asses (or weighs) the evidence at the end of the case determining its credibility. This includes deciding when relevance turns on the fulfillment of a condition of fact. That is the jury decides the weight of evidence, in which the relevance depends on the truth of other evidence (with the judge acting as a gatekeeper). o A jury decides if a condition is satisfied when different answers are reasonable. (ex: if you find A you may proceed to find B). o Keep in mind the judge will make sure he gives the jury careful instructions, using 104(b). (ex: if we are dealing with the occurrence of a prior act, or the occurrence of another conviction) (See FRE 404(b), 608, 609) o Examples of conditional relevance: (p. 85) Questions of Authenticity (like connecting a gun found by a cop on the scene of the crime) Whether a witness has personal knowledge Credibility of Witness Statements Prior Bad Acts

***Note there is an overlap between the responsibilities of the judge and jury. Ex: Even though the judge may make a preliminary decision over FRE 104(a) that the expert is qualified as an expert, the jury also makes its decision under FRE 104(b) as to what weight to give to that witnesss testimony. (Commentators call this conjunction.) The judge admits the evidence if it is material and relevant under FRE 104(a) However, he may decide there needs to be more facts to support the evidence (if the evidential hypothesis is too strained) and will admit the evidence only on condition that the other facts are also admitted to support the evidence under FRE 104(b).

Evidence Outline

Page 8

Huddleston verses Booker (Notes on Page 87-90) The standard to be applied when using FRE104(b) is from Huddleston preponderance of the evidence. KNOW THIS NOTE and annotate next to 104(b) what Huddleston rules. Another standard is possible (i.e.: not all states hold this). But, by choosing Huddleston, the Supreme Court rejected these other possibilities. III. LOGICAL RELEVANCE Establishing Relevance: The proponent must be prepared to advance an evidential hypothesis explaining why his proof is relevant. Malavet wants us to know how to construct the evidential hypothesis that supports admissibility and how to counter that evidential hypothesis In constructing the evidential hypothesis determine How will the evidence bear on the case? What point will this evidence tend to prove? Why does this point count? Eight General Exceptions where evidence will be admissible: 1. Complicated issues of causation: Used to prove cause and effect (See Boys on the Bridge) 2. Prior accidents or claims: Generally these are not admissible 2 two exceptions if the D wants to show prior accidents of claims of the P: (1) to show common plan and scheme of fraudif D can show the Ps prior claims were fraudulent in nature they will be admissible (2) to show the prior accidents and claims are relevant on the issue of damage to the Pif P is claiming an injury to the same part of her body that has been injured in a prior accident than the jury is entitled to know that 1 exception if the P wants to show other accidents involving the same instrumentality: (1)Other accidents involving the same instrumentality which occurred under the same or similar circumstances will be admissible because it tends to show notice or knowledge on the part of the D. Even beyond notice, other accidents can also prove that the instrumentality is dangerous and defective. 3. Where the intent or state of mind of the party is at issue: You prove intent and state of mind by the conduct of a person. Ex: Gender discrimination case in employment. The D says I dont discriminate. Can the P admit into evidence how the D treated other women? Yes, to show the discriminatory intent of the D employer. 4. To rebut: Most commonly, it is to rebut the claim or defense of impossibility. Asserting a defense of impossibility opens the door to this evidence. 5. Comparable sales to establish value: at issue is the value of your chattel or your parcel of real property. You may show what other chattels or other parcels of real property have sold for as some evidence of the value of your chattel or parcel of real property providing 3 requirements are met:

Evidence Outline

Page 9

6.

Habit Evidence: The habit of a person is admissible to infer that at the time of the litigated event, this person acted in conformity with his or her habit. Here, there are three rules which tend to overlap. Make sure not to confuse: Disposition evidence: The disposition of a person (i.e.: to be careless, careful, happy, etc.) is not admissible to show the person was careful or careless or happy at the time of the litigated event. (NOT admissible) Specific Prior Acts: Evidence that a person acted in a certain way on a prior occasion (once or twice) is not admissible to show the person acted the same way at the time of the litigated event. (NOT admissible) Habit Evidence: must be (1) specific detailed conduct (not general like disposition) and (2) reoccurringthe specific conduct must have occurred often enough, so we can say it is habitual (more than just one or two times) (ADMISSIBLE) Note: It is discretionary with the court how many times something becomes habit. If a party does anything 3 or more times, mention the possibility it could fit under habit evidence. (See Below for entire Section on Habit) 7. Industrial or business routine: this is habit evidence all over again with corporations. The rule is the same as it is for habit. 8. Industrial custom as evidence of the standard of care: the custom of the business or trade is non-conclusive evidence on the standard of care. (i.e.: just because the evidence is presented does not automatically mean the company is negligent (this is what we mean by non-conclusive). Examples of Logical Relevance Problem 2-AWas He Going to Fast? There was a collision between two cars (Gadsky and Reinhart) and both drivers die. Hill (a witness) will testify that he saw Gadsby going 80 mph 30 minutes before the accident. Is information pertaining to the speed of Gadsky (going 80 mph) 30 minutes before the accident relevant to a case determining who caused the accident? Answer: Logical Relevance: Evidence that Hill saw Gadsby going 80 MPH is relevant on the question of Gadsbys speed speed on impact (FRE 401) Pragmatic Relevance: the probative worth is small because of the 30-mile (about 23-minute) gap between sighting and impact (FRE 403). Problem 2-BBoys on the Bridge A driver is hit by a chunk of concrete while driving under a bridge. A construction company is in the process of re-building the bridge. P sues the Construction Company (D). D wants to offer evidence from Carla that there were boys running away from the bridge at around the time of the incident. Is this evidence relevant? Answer: Evidential Hypothesis: The boys running away from the bridge could have caused the chunk of concrete to fall. (i.e.: it is possible the boys are responsible for the brick falling on Ps carinductive, circumstantial evidence). This fact makes it

Evidence Outline

Page 10

less probable that the D caused the chunk of concrete to fall, because there is the possibility of an alternative cause. (Therefore it is relevant). The cause of the chunk falling is a fact that is of consequence to the determination of the action. Who caused the action is material to determining whether D should be liable. (material). Problem 2-CFlight and Guilt A man with a sawed off shotgun robs Joe and Andy. The next day Joe and Andy examine mug books and independently identify Carl as the thief. Later, police arrest Carl at his house. At trial the State calls Brenda (Carls girlfriend). The state offers testimony that when the police came to the house to get Carl, Carl hid in the closet. Carl says this is irrelevant. Answer: Evidentiary Hypothesis: This evidence could show D was hiding or fleeing showing he was guilty of robbing Joe and Andy or the D ran into the closet to flee from the police out of guilty consciousness, for committing the crime he is charged with. Evidence of flight is relevant because it can show consciousness of guilt, thus meeting the standard of FRE 401. Pragmatic Relevance: The probative value of evidence of flight can vary greatly, depending on the facts of the case. FRE 403 questions can be resolved very differently in these cases. Analytically, flight is an admission by conductIts probative value as circumstantial evidence of guilt depends upon the degree of confidence with which four inferences can be drawn: (1) from the defendant's behavior to flight; (2) from flight to consciousness of guilt; (3) from consciousness of guilt to consciousness of guilt concerning the crime charged; and (D had to know he was wanted) (4) from consciousness of guilt concerning the crime charged to actual guilt of the crime charged. Evidence of flight does not create a presumption of guilt but it can be evidence of guilt. If you cant establish that the D flew because of consciousness of the crime he is being charged with, it cannot be admitted. IV. PRAGMATIC RELEVANCE Unfair Prejudice under FRE 403: Unfair prejudice may (1) inflame or anger the jury binding the issue of guilt to the D because of the evidence or (2) induct the jury to draw a general propensity inference rather than using the proof in a more particularized manner. Balancein deciding whether the risk of unfair prejudice substantially outweighs probative worth, the judge considers such factors as Prosecutors need for the evidence The degree of probative worth on the particular question; and The nature of the prior misdeeds (is it inflammatory?) State v. Chapple Defendant was accused of killing someone related to a drug deal. Two witnesses said he did it.

Evidence Outline

Page 11

Prosecutors submitted gruesome pictures of the deceased. On appeal, D contends the trial court erred by admitting pictures of the charred body and skull of the victim. The pictures were in vivid color and were extremely gruesome (court goes into grave detail explaining the gruesomeness). Evidential hypothesis being proffered by the prosecution: the pictures prove the witnesss statement that the deceased was shot in the head (cause of death) and that it was homicide (manner of death). The two things the prosecution must establish. Relevancy is not the sole test of admissibility for the trial court. Where the offered exhibit is of a nature to incite passion or inflame the jury-and the photographs in the case at bench certainly fall within this categorythe court must go beyond the question of relevancy and consider whether the probative value of the exhibit outweighs the danger of prejudice created by admission of the exhibit. In this case, the photos were relevant under FRE 401 to establish a case for first-degree murder. However, they had little probative worth. The fact that the victim was killed, the cause of his death, and what was done to the body after death were not in controversy. The facts issued in the photograph were simply not in dispute or at issue. The only issue being tried was whether it was the D that did it-- the defense was saying we admit the person died of a violent death, but it is not our client who killed him! Because the photographs could very well have inflamed the minds of the jury enough to impair their objectivity, there was definitely a danger of unfair prejudice. Since there was so little probative value to these photographs and since their capacity to inflame is so obvious, the admission was an abuse of discretion. The probative value of the relevant photos was thus outweighed by the danger of unfair prejudice. Times when pictures probably would be allowed to be admitted: To identify the deceased To show the location of the mortal wounds To show how the crime was committed To show the brutality of the crime (when there is a standard for the crime that brutality matters) To aid the jury in understanding the testimony of the witness (although an autopsy report may do just as good) Examples of Pragmatic Relevance Problem 2-EThe Battered Wife A man is charged with killing his wife. He claims he stabbed his wife on accident. (the D is trying to say his wife fell into the knife) The state is offering testimony by a counselor at a Shelter for Battered and Abused Women that two years earlier the wife had sought refuge there for about 30 days, during which time she divorced Donald. (Prosecution is trying to say there has been a history here of abuse.) The evidence is being offered during the prosecutions case-in-rebuttal. Evidentiary hypothesisFlight to the battered womens shelter suggests that:

Evidence Outline

Page 12

The victim was afraid She remembered a frightening event The event actually happened It must be something defendant did (it was he whom she feared) Defendant was hostile toward her (his behavior indicated hostile intent) and finally He later acted on that hostility by killing her.

The judge would likely not admit under FRE 104(a) because it is difficult for the judge to determine by the preponderance of the evidence that this proves the intent to kill her. Under FRE 104(b) the judge could admit this evidence but would probably require the attorney to establish the H & W were living together at the time she fled to the shelter; that there was no one else in the house that could have been the source of abuse; and that she fled to the battered womens shelter because of abuse. This evidence is relevant, but does it pass the muster of FRE 403? Ps side: (arguing for a probative value) The victim has had a dependency on the D and endured a lot of abuse. The longer she has been with the H the more her dependency has been. As to the element of motive/intent (i.e.: was this a mistake or intentional) this evidence has a high probative value because it can go to the Ds intent. Ds side: (arguing against a probative value) Just because she went to the shelter two years ago does not mean there was abuse or even that it continued. (this was an isolated incidence) There are no signs of physical violence There are no records to establish abuse Most likely, the court would find the evidence is indeed prejudicial but not unfairly prejudicial. Therefore it ought NOT to be excluded under FRE 403. Problem 2-FExploding Gas Tank There is an exploding gas tank as a result of a car crash. The passenger (Risner) dies, but his wife sues the automaker, alleging that negligent design of the fuel tank caused Risners death. At trial, D (automaker) introduces testimony by a state trooper that the impacting vehicle was going about 68 mph at the time of the impact. The automaker also introduces a certified copy of a guilty plea, entered by the driver of the impacting vehicle to charges of involuntary manslaughter arising from the accident. In the end, jury returns a verdict for D. Mrs. Risner appeals, urging the trial court should have excluded the evidence under FRE 403. Answer: Here we have 2 pieces of evidence (testamentary evidence of the speed & a guilty plea) being offered to show the cause of the accident was the negligence of the driver and not due to the negligent design. Evidentiary Hypothesis: This evidence goes to the conclusion of whether the explosion was caused by negligence on the part of the driver verses

Evidence Outline

Page 13

negligent design on behalf of the automaker. (goes to who caused the accident) Therefore this is relevant under FRE 401, 402. Court said the guilty plea should have been excluded because it was prejudicial (it confused the jury with criminal law (guilty plea) when the law that should be used is civil (more than 1 tortfeasor can be liable, i.e.: just because the driver admitted he was guilty does not mean the tank was made good.) Evidence of the guilty plea confused the issue of causation. If this plea may be received on this issue, the trial court must instruct the jury on the concept of causation in tort law, with considerable risk of confusion. Important: This was not reversible error. It was just the appeals court letting the trial court know they should not admit it.

Common FRE 403 Exceptions: Three main situations where the specific conduct will probably be logically relevant but because of policy considerations of FRE 403, we keep it out: (1) Liability Insurance (2) Subsequent Remedial Measures (3) Settlements 1. LIABILITY INSURANCE FRE 411Liability Insurance Evidence that a person was or was not insured against liability is not admissible upon the issue whether the person acted negligently or otherwise wrongfully. This rule does not require the exclusion of evidence of insurance against liability when offered for another purpose, such as proof of agency, ownership, or control, or bias or prejudice of a witness. Liability is not admissible to show any kind of fault on the D or the ability of D to pay. Nor would the absence of liability insurance be admissible to prove the contrary.

FRE 105Limited Admissibility When evidence which is admissible as to one party or for one purpose but not admissible as to another party or for another purpose is admitted, the court, upon request, shall restrict the evidence to its proper scope and instruct the jury accordingly. It is a headache for judges that evidence tends to prove too muchevidence that proves one point can prove another (which is highly prejudicial). (ex: evidence that proves liability insurance (prejudicial) but at the same time proves admission of guilt (proves a point). This rule authorizes the court to admit the evidence, but give limiting instructions to prevent misuse on other issues or against other parties. This rule allows judges in the context of liability insurance cases, to admit evidence of liability insurance for an exception purpose as long as the judge gives the jury a limiting instruction on how to use the evidence.

Evidence Outline

Page 14

Exceptions to liability insurance: If the liability insurance is relevant on some other basis, it will be admissible.

1. To show a partys admission of liability: If I admit that I was the cause

of the accident and that I have liability insurance to cover it, this will be used to show a partys admission of guilt. The judge should give a limiting instruction telling the jury to exclude any admission of insurance. 2. To prove ownership and control when ownership and control is disputed. If someone is disputing they own a car or building you can admit the fact that they have liability insurance on the property to show the person owns it. 3. Where it is relevant to impeach the credibility of a witness by showing interest or bias or motive. It can be used if a witness has a motive for falsifying testimony. Ex: P sues D for personal injury damages arising out of an intersection automobile accident. The Ds witness said I saw the cars enter the intersection and the P ran the red light. However the attorney for the P knows the witness is a claims manager for the same auto insurance company for which the D is insured. This is relevant to impeach the credibility of the witness by showing the witness has an interest, bias, or motive to lie or exaggerate. Problem 2-GMy Insurance Will Cover it! There is a two car accident between Linda and Myra. After the accident, Linda says: Whoever screws up, her insurance pays. Im sure my insurance will cover it. Theyll pay for what happened to your Porsche. Both cars end up having about $6,000 in damages. Myra sues and Linda counterclaims. Myra wants to offer this evidence, however Linda objects. Answer: P(Myra) is trying to say D (Linda) admitted to causing the accident. An admission would prove fault (confession.) A confession relieves the opposing party from having to prove otherwise. 1. Is this admission relevant and material? Yes. It is relevant to who caused the accident. (Makes it more probable that Linda caused the accident) 2. Note: There is a hearsay issue here, however the hearsay objection should be overruled because of the admission doctrine. (FRE 801(d)(2)): A statement is not hearsay if the statement is offered against a party and is the partys own statement in an individual capacity. Basically, anything that comes out of your mouth following an accident will be treated as a party admission under the Rule, when the statement is offered by your opponent at trial. 3. This evidence is not admissible under FRE 411 because Myra is using to try and prove that Linda was at fault. This evidence is admissible under FRE 401 and 402, but not under FRE 411, what do we do? 4. We do the balancing test of FRE 403. Probative Value: showing she was the proximate cause of the accident Unfairly prejudicial use of it: the fact she has insurance cannot be used to show she was negligent. Capacity to pay should not affect an objective finding that someone should pay. We want to protect people

Evidence Outline

Page 15

who are responsible and not force them to pay for something just because they have the money. The jury should not consider the ability to pay. 5. It is key to note here, it is not the statement of insurance that is being used to prove negligence, rather it is the partys admission of liability that is being used. The statement of insurance just happens to be inseparable from the admission. 6. In admitting this evidence, the court should give a limiting instruction to the jury explaining what they should do with this evidence. (FRE 105, FRE 106) This will further avoid the possibility of unfair prejudice. The instruction language could be: You should decide this case without making any assumptions about insurance. Note, that applying FRE 105 is upon request from a party, but there may be situations where the court will be required to do it on its own motion.

2. SUBSEQUENT REMEDIAL MEASURES (i.e.: subsequent repairs):


FRE 407Subsequent Remedial Measures When, after an injury or harm allegedly caused by an event, measures are taken that, if taken previously, would have made the injury or harm less likely to occur, evidence of the subsequent measures is not admissible to prove negligence, culpable conduct, a defect in a product, a defect in a product's design, or a need for a warning or instruction. This rule does not require the exclusion of evidence of subsequent measures when offered for another purpose, such as proving ownership, control, or feasibility of precautionary measures, if controverted, or impeachment. Reasons for this Rule: Public Policy: We want to encourage Ds to make repairs after an accident and to make conditions safer than what they were at the time of the accident. So we tell the D go ahead and make the subsequent remedial measure. In the interest of public safety we will not use it against you. Relevancy: concerns over relevancy arise because efforts to prevent future accidents may not show that past practices amounted to negligence or fault. Confusion of the issues: concerns over confusion of the issues arise partly because of the relevancy problem and partly because it may be impossible to show that the changes that follow an accident were made because of the accident. Remedial measures can be proved for purposes other than establishing negligence, such as proving ownership, agency, etc. Exceptions to Subsequent Remedial Measures: To show ownership and control when ownership and control is disputed: showing someone was taking subsequent actions to repair something tends to show they are in control of the building. Feasibility: to show the feasibility of a precautionary measure when feasibility is controverted (i.e.: when Ds deny feasibility). What would not

Evidence Outline

Page 16

have been admissible as part of the plaintiffs case in chief becomes admissible because feasibility was controverted by the D. Impeachment: almost any testimony given by defense witnesses could be contradicted by way of a subsequent remedial measure.

More on Feasibility- Proof of a remedial measure may be admitted where a defendant has argued that a certain practice was impossible under the circumstances (scientifically, technologically, or economically). If the defendant has taken a remedial measure that shows that the practice is feasible, then that action may come in to prove feasibility. For the feasibility exception to apply, two questions are required: (1) what is meant be feasibility? and (2) is the feasibility controverted? (See Truer) There are several schools of thought on how feasibility can be interpreted: broad (when you are the offering party and want the exception to be applied) or narrow (when you are the objecting party and dont want the exception to be applied) o Narrow: disallowing evidence of subsequent remedial measures under the feasibility exception unless the defendant contends the measures were not physically, or economically possible under the circumstances the pertaining. o Broad: a broader spectrum of motives and explanations for not having adopted the remedial measure earlier. (ex: the cost is too high for this measure to have been adopted earlier) Controverting feasibility: o Feasibility NOT Controverted (the narrow view) Courts in the narrow view have concluded that feasibility is not controverted -and thus subsequent remedial evidence is not admissible under the Rule- when (1) a defendant contends that the design or practice complained of was chosen because of its perceived comparative advantage over the alternative design or practice or (2) when the defendant merely asserts that the instructions or warnings given with a product were acceptable or adequate and does not suggest that additional or different instructions or warnings could not have been given or (3) when the defendant urges that the alternative would not have been effective to prevent the kind of accident that occurred. o Feasible AND Controverted (the broad view) Courts announcing a more broad view have concluded that "feasible" means more than that which is merely possible, but includes that which is capable of being utilized successfully. Note: Ds are often willing to stipulate to feasibility in order to avoid having subsequent remedial evidence admitted. The big question becomes is it feasible OR it is feasible (possible) but not recommendedthe issue arises when the D offers some other explanation for not putting the measure into effect sooner. The P will often classify this as putting feasibility into issue. It all comes down to a judgment call.

Evidence Outline

Page 17

More on Impeachment- has created some of the same practical and interpretive problems presented by the exception for establishing feasibility. A plaintiff cannot impeach a defendant with evidence of a remedial measure for a broad assertion like the defendant claiming they were not negligent. However where a defendant makes a more narrow assertion, like I couldnt have possibly made it any safer or that would have not helped and only been a nuisance evidence of a remedial measure may be admitted if it tends to impeach such an assertion. If a D is asked on cross examination whether he thinks he had taken all reasonable safety precautions, and answers in the affirmative, then a subsequent remedial measure can be seen as contradicting that testimony. However, most courts would not allow this admissible for impeachment if it is offered for simple contradiction of a defense witness testimony. Methods of Impeachment: o Bias/corruption o Mistake/capacity o Bad character o Inconsistent statements o Contradiction Tuer v. McDonald (Malavet thinks this case is very well written for an exam!) There was a medical malpractice claim. The deceased died as a result of being taken off of the Heprin. After the deceased died, and apparently because the deceased died, the hospital changed its protocol for injecting Heprin. Relevance of a Remedial Measure (Evidential Hypothesis for P): You changed your protocol and that is an admission that your old protocol was wrong. FRE 407 is designed so that remedial measures are excluded so that these hypothesizes cant be used. (In other words, Malavet seems to say that if the evidence is considered a subsequent remedial measure to prove negligence, then it is not admitted at the 401 stage (we dont even go to the 403 stage). Defendants wished to exclude any reference to the change in protocol under Maryland Rule 5407 (same as rule 407) because it was a subsequent remedial measure. Plaintiff countered that: (a) the change was not a remedial measure because the defense claimed the prior protocol was correct, and (b) they are entitled to prove the change in protocol to show that continuing Heparin was "feasible." The trial court rejected the first argument (saying that defendants did not have to admit wrongdoing in order to claim that a change was remedial), but ruled that it would admit the proof if defendants denied feasibility (i.e: if feasibility was controverted) or if the Ps could show the statement is impeachment. Plaintiff got Dr. McDonald to say that restarting Heprin would have been unsafe. In other words, the doctor is saying; Given my medical knowledge and experience, at the time prior to Mr. Turers death, I considered it unsafe to restart Heprin.

Evidence Outline

Page 18

Feasibility Issue: The court said that in the context of the Drs statements, he was not denying feasibility. The court constitutes the statement as meaning the Heprin was feasible, it was just not advisable to use. Therefore the feasibility exception does not apply. (Court in Pruer chooses the narrow view, because we are dealing with human life.) Impeachment Issue: Court says you need substantial impeachment. The contradiction of the testimony has to be substantivemeaning it is substantially related to the facts and resolution of the case. The Dr. merely said in my judgment, it would have been unsafe. This is not a denying he did not take every reasonable precaution. It was merely his judgment call. o If the Dr. would have made a blanket denial, that might have opened the door to the exception. Also, if the Dr. said he used the best combination of safety and operation yet devised this would also have opened up the door for use of the exception. o Another statement that would have constituted impeachment: I could not do anything different today. o The court said here at the time of the event, the measure was not believed to be as practical as the one employed, or that the defendant was using due care at the time of the accident. Simple contradiction of testimony that does not get to the meat of the matter will also not satisfy this exception. Because the court rules the change in measures was not admissible (it did not meet the 3 exceptions, we do not move on to the other rules like FRE 403.)

3. SETTLEMENTS:
FRE 408Compromise and Offers to Compromise (a) Prohibited uses.Evidence of the following is not admissible on behalf of any party, when offered to prove liability for, invalidity of, or amount of a claim that was disputed as to validity or amount, or to impeach through a prior inconsistent statement or contradiction: (1) furnishing or offering or promising to furnish or accepting or offering or promising to accept a valuable consideration in compromising or attempting to compromise the claim ; and (2) conduct or statements made in compromise negotiations regarding the claim, except when offered in a criminal case and the negotiations related to a claim by a public office or agency in the exercise of regulatory, investigative, or enforcement authority. Permitted uses. This rule does not require exclusion if the evidence is offered for purposes not prohibited by subdivision (a). Examples of permissible purposes include proving a witness's bias or prejudice ; negating a contention of undue delay; and proving an effort to obstruct a criminal investigation or prosecution.

Settlements are not admissible as any kind of indication of fault or liability or damage. Offers to settle and conduct or statements made

Evidence Outline

Page 19

during settlement negotiations, when offered to prove liability is not admissible. Courts want to encourage settlementsthis encourages the parties to be frank and open in settlement negotiations. Ex: I say lets settle, I will admit my client was negligent. This cannot be used. Statements/Evidence that come into court that are independent of the settlement process cannot be bathed in immunity by being turned over or mentioned during settlement. In other words, this rule only applies to exclusion of material generated for the purpose of aiding in the settlement.

Exceptions to Settlements: There must be a claim: The facts have to have matured enough so that there is someone who can be identified as a claimant. Ex: If I tell my neighbor I think my dog bit you. Lets settle. This could be used against me in litigation because there was no claim. There must be a dispute about liability or amount. Ex: I will admit that I owe you $10,000 but I am not going to pay you. You have to sue me. I admitted liability and full amount. This could be used against me because there is no dispute as to liability or amount. An offer to pay hospital or medical expenses is not admissible: This is because the rule assumes I may be motivated by humanitarian motives. Malavet says statements offered to pay medical expense are excludable if such contents are trying to settle the case. However, an admission of fact that is made as part of an offer to pay these expenses is admissible. Ex: If I admit to doing it and then offer to pay the medical expenses, the offer to pay the medical expenses will not be admitted, but the admission will be admissible. (FRE 409) Impeachment: Statement can be used for other purposes such as impeachment (or bias or prejudice), so a party to a settlement negotiation should be careful about taking the stand Civil Cases Problem 5-P Two Potato, One Potato- Perrin, a potato farmer, buys a herbicide from Sosbee for his crops. The herbicide ended up not being very good. The yield that year is about half the typical yield and when Perrin tells Sosbee the first time he says, my potato plants dont look as healthy as they should and Sosbee replies, Dont worry, well take care of you. Later, on another occasion, he told Perrin again my yield isnt half what if was last year and Sosbee replies You just tell us the damages youre claiming, and well bill Cheron [the manufacturer of the herbicide]. Can this be used against Cheron? Do these statements fall under rule 408 (i.e.: is this an attempt to settle)? Answer: There has to be a claim which is disputed for this rule to apply. There can be no compromise if there is no claim. If a party has filed a claim in court, there is no question that you have a claim that is being disputed. But right when the damage/accident takes place you might not have a dispute. Also, there has to be a

Evidence Outline

Page 20

conflict as to the validity or amount. These statements do not dispute validity or amount. These statements were too early for this rule to kick in. In this case we are assuming this instance is not an attempt at compromise, and therefore is not a negotiation protected by the privilege. It is important to note, this could be classified as a negotiation in which case it would be barred from admissibility. Remember the policy behind this rule: we are only concerned about the potential of thwarting settlement negotiations. That concern is just not prevalent here. On a side note, this cannot be hearsay because this is an oral assertion admitting liability. Another Civil Case Hypo: While driving a car Plaintiff and Passenger are struck by a car driven by D, causing injuries. Passenger and D settle but P proceeds to trial. If D calls Passenger as a witness, can P cross examine him about the settlement? Answer: No. The prohibition does apply here. There is a possible exception howeverTo show the bias of passenger in testifying for the defendant 408(d). Criminal Cases Civil Settlement negotiations in the criminal context. Under FRE 403, the use of civil settlements to prove liability or invalidity or the amount of a disputed claim is also blocked in criminal proceedings. Offering or accepting a compromise in a civil case is not very probative of the defendants guilt in a later criminal case. However, statements in civil settlement negotiations by a public office or agency in the exercise of its regulatory, investigative, or enforcement authority can be used in a criminal case. When someone makes a statement in the presence of government agents, its subsequent admission in a criminal case should be expected. (think Miranda Rights) Therefore, a settlement negotiation between a criminal and the state which involves payment in exchange for dropping criminal charges is not excludable under this rule because it is seen as an obstruction of justice. Hence, it seems under this rule, it would be more likely the attorneys would do all the taking using hypothetical terms, so as not to incriminate the defendant in future proceedings. Plea Bargaining Rule 410. Inadmissibility of Pleas, Plea Discussions, and Related Statements Except as otherwise provided in this rule, evidence of the following is not, in any civil or criminal proceeding, admissible against the defendant who made the plea or was a participant in the plea discussions: (1) a plea of guilty which was later withdrawn; (2) a plea of nolo contendere;

Evidence Outline

Page 21

(3) any statement made in the course of any proceedings under Rule 11 of the Federal Rules of Criminal Procedure or comparable state procedure regarding either of the foregoing pleas; or (4) any statement made in the course of plea discussions with an attorney for the prosecuting authority which do not result in a plea of guilty or which result in a plea of guilty later withdrawn. However, such a statement is admissible (i) in any proceeding wherein another statement made in the course of the same plea or plea discussions has been introduced and the statement ought in fairness be considered contemporaneously with it, or (ii) in a criminal proceeding for perjury or false statement if the statement was made by the defendant under oath, on the record and in the presence of counsel. Again for reasons of judicial economy, plea bargaining statements and pleas of guilty or no contest are not admissible at trial [See FRE 410]. In defining exactly which statements are covered, Rule 410(4) excludes Any Statement made in the course of plea discussions with a Prosecuting attorney which do not result in a guilty plea.

Confession or Plea Bargaining? The ACN tells us FRE 410 statements to law enforcement agents are not necessarily inevitably admissible, particularly if agents purport to have authority to bargain. FRE 410(3) excludes Ds statements to law enforcement agents if (a) the agent has actual authority to bargain and is doing so, or (b) defendant thinks bargaining is occurring and his belief is reasonable under the circumstances. Objective Reasonableness Standard under FRE 410: (1) whether the defendant exhibited an actual subjective expectation to negotiate a plea and (2) whether that exception was reasonable given the totality of the objective circumstances. Most courts state that if a plea bargaining fails, then any statements made during these negotiations are not admissible at trial. There has to be some sort of waiver of rights for a statement to be admissible. Problem 5-R I Used His Stuff Rackley is charged with passing counterfeit bills. His Attorney sets up a meeting with the DA. The DA and two secret service agents meet the D and his attorney (first meeting) but no plea bargaining occurs because the DA hasnt had time to review the case. Rackleys attorney then calls the DAs office and sets up another meeting. They meet again but only the secret service agents show up. Rackley and his attorney draw up a implicating Rackleys guilt. Eventually, the plea bargain falls through. At trial the DA tries to admit Rackleys statement, but the D objects claiming these statements were made during a plea bargain. Answer: FRE 410(4) implies that the absence of the DA disposes of the defense argument: The Rule simply does not reach what Rackly told the agents.

Evidence Outline

Page 22

Remember, statements to law enforcement agents "are not necessarily inevitably admissible." Remember, FRE 410(3) excludes Ds statements to law enforcement agents if (a) the agent has actual authority to bargain and is doing so, or (b) defendant thinks bargaining is occurring and his belief is reasonable under the circumstances. Here, Rackley can advance both of these arguments, though he will probably lose. First, he can argue the agents have actual authority to bargain because the second meeting continued the first meeting. But the problem is that the DA disclaimed any interest in bargaining in the first meeting and did not even show up for the second meeting. Also there was an alternative purpose for the meetings: gathering information and usable evidence. Rackley can also argue that he thought the bargaining was going on because he had his lawyer with him and the agents were there and therefore the belief was reasonable. However, it is very unlikely a court would find there was plea bargaining taking place here. Rackley was never told by the Secret Service Agent that they had the authority to plea bargain. The agents made no offer of any deal with Rakley and they were not relaying a prosecution offer. Rackley attempts to get around the fact that no attorney was present for the government at the time of the admissions by putting forth a theory of ongoing negotiations. Rackleys attorney independently initiated the meeting with the agents. The action taken by the Ds was in no way a prescheduled continuation of the first meeting, but instead was a decision by Ds counsel to take some action in the hope of including leniency by the government. Problem 5-S Just Keep Them out of It Two men, Bragen and Roberts, are busted with a meth lab in their basement. Their wives are arrested also. They are interrogated with counsel in a Federal building and both men make incriminating statements and plead for the agents to let their wives go because the women had nothing to do with it. The agents release the wives and then at trial admit the incriminating statements. Was this a confession or plea bargaining? Answer: FRE 410(4) appears not to contemplate bargaining for a third person, and the situation is ambiguous enough to be categorized in conflicting ways: Either 1) the two men in effect confessed because they did not want to drag down the women, or 2) they were trying to make a deal that would take care of everyone. 1) Taking the first view, releasing the women gave defendants what they wanted, and the government should get something in return (i.e.: their confession)this evidence is usable against the defendants. In exchange for letting the women go, we will confess (in effect, you have engaged in a waiver of the privilege under the rules). 2) Taking the second view, defendants were trying to make a bargain for the third party but did not enter a plea, so the attempt to a plea failed. If the plea failed the K was not agreed to, therefore the privilege applies and has not been waived. In this instance, the government released the women because

Evidence Outline

Page 23

it was convinced of their innocence, and is not entitled to any "quid pro quo" for doing what it ought to do on making such a discovery. They never reached an agreement and the waiver has not been completed. (Because of this, prosecutors are making you sign a waiver waiving the inadmissibility of plea bargaining before the agreement is entered in to) Conclusion: We think that the two characterizations are almost equally plausible: The presence of the lawyers (prosecutor Kendall and counsel for each defendant) suggests that all parties were interested in a deal, so the ensuing talk was plea bargaining. But the presence of the agents, and the fact that they obtained a written waiver from the women points toward the fact that the men confessed and waived there right to exclude it from evidence.. You must be focused on the accuseds perceptions of the discussion (did they believe it was a plea or a confession? A court must apply a two-tiered analysis and determine (1) whether the accused exhibited an actual subjective expectation to negotiate a plea at the time of the discussion and (2) whether the accused expectation was reasonable given the totality of the circumstances. V. WRITINGS RECORDED STATEMENTS

AND

FRE 106Remainder of or Related Writings or Recorded Statements When a writing or recorded statement or part thereof is introduced by a party, an adverse party may require the introduction at that time of any other part or any other writing or recorded statement which ought in fairness to be considered contemporaneously with it. If you admit a writing or recorded instrument at the other party can make reference to any other part of that writing or recorded statement. This rule can be used during cross, during interruption of the Ds case-inchief, or during the case in rebuttal.

Problem 2-HPower Rollback Caused the CrashThe Air Crash Case While serving as a Navy flight instructor, Erin died in a training session. Her husband (Jim Ranney) sued the plane company. Ranney wrote a letter to his commander saying that the Rollback caused the crash. During the Ds case-inchief, counsel for plane company calls P as an adverse witness and asks him about several comments that were made in a letter that P wrote. (Note the hearsay issue here) Comments in the letter contradicted the Ps theory of the crash which he testified to. The D said we never asked about these comments and therefore it is beyond the scope of direct. Answer: Does the part of Ps letter not mentioned on cross shed important light on the issue? YES! FRE 106: The common-law rule of completeness was designed to prevent exactly the type of prejudice of which Rainey complains. Rainey is complaining that the D cannot just selectively choose which parts of the letter he wants to talk about. If you are going to admit the letter, than anything in that letter is fair game. You would allow a misleading piece of evidence to go un-countered (and un-

Evidence Outline

Page 24

countered at that time) and this is unfair. (Malavet says he always tests on this issue.) VI. CHARACTER EVIDENCE
AND

RELEVANCE

FRE 404Character Evidence Not Admissible To Prove Conduct; Exceptions; Other Crimes (a) Character evidence generally Evidence of a person's character or a trait of character is not admissible for the purpose of proving action in conformity therewith on a particular occasion, except: (1) Character of accused - In a criminal case, evidence of a pertinent trait of character offered by an accused, or by the prosecution to rebut the same, or if evidence of a trait of character of the alleged victim of the crime is offered by an accused and admitted under Rule 404 (a)(2), evidence of the same trait of character of the accused offered by the prosecution; (2) Character of alleged victim - In a criminal case, and subject to the limitations imposed by Rule 412, evidence of a pertinent trait of character of the alleged victim of the crime offered by an accused, or by the prosecution to rebut the same, or evidence of a character trait of peacefulness of the alleged victim offered by the prosecution in a homicide case to rebut evidence that the alleged victim was the first aggressor; (3) Character of witness - Evidence of the character of a witness, as provided in rules 607, 608, and 609. (b) Other crimes, wrongs, or acts Evidence of other crimes, wrongs, or acts is not admissible to prove the character of a person in order to show action in conformity therewith. It may, however, be admissible for other purposes, such as proof of motive, opportunity, intent, preparation, plan, knowledge, identity, or absence of mistake or accident, provided that upon request by the accused, the prosecution in a criminal case shall provide reasonable notice in advance of trial, or during trial if the court excuses pretrial notice on good cause shown, of the general nature of any such evidence it intends to introduce at trial. Four preliminary questions 1. For what purpose do you seek to prove character? (What is the relevance?) Any one of three purposes can be used: Because the character of a witness itself relates to a material issue in the case (specific element of the crime) o There are limited times this occurs in the civil context, and it NEVER happens in criminal contextsthe book came up with one example. Character as circumstantial evidence to infer conduct at the time of the litigated event: the character is not itself a material evidence of the case, but you want to use this evidence as way to infer how this party acted at the time of the event. o Cannot be used in civil cases, but there are times this can be used in criminal cases.

Evidence Outline

Page 25

Character to impeach the credibility of a witness (used when D is a witnessthis is discussed in impeachment) Any one of these purposes cannot be used: Bad person inference (he is a bad person and therefore likely committed this crime) Action in conformity therewith (he did it before, and therefore he is likely to have done it this time) 2. What method/technique can you use to prove character? 3 possibilities: Specific Acts of conduct Opinion Testimony Reputation Evidence 3. What kind of case is it? Civil Criminal 4. Character for what trait? (pertinent character trait) There is no such thing as general character. There must be some specific pertinent trait or character. This trait must be substantively at issue in the case. This depends on the nature of the case. If it is a larceny case the trait would be honesty/dishonesty If it is an assault casethe trait would be peacefulness/violence If it is a perjury casethe trait would be truthfulness or the lack of it. FRE 405Methods of Proving Character (a) Reputation or opinion. In all cases in which evidence of character or a trait of character of a person is admissible, proof may be made by testimony as to reputation or by testimony in the form of an opinion. On cross-examination, inquiry is allowable into relevant specific instances of conduct. (b) Specific instances of conduct. In cases in which character or a trait of character of a person is an essential element of a charge, claim, or defense, proof may also be made of specific instances of that person's conduct. FRE authorize both reputation and opinion evidence, but sharply restrict evidence of specific instances. Under the old common law, only reputation was allowed for the purpose of proving character as circumstantial evidence of conduct on a particular occasion. Opinion- Allowed by defense and by Prosecution to rebut. Reputation- Allowed by defense and by Prosecution to rebut (FRE 803 has a hearsay exception for reputation evidence). Prior Specific Acts- Can only be used o where character or a trait of character is an essential element of a charge (criminal), claim (civil), or defense [Rule 405(b)] or

Evidence Outline

Page 26

o MIMIC Rule: to show motive, intent, plan, identity, opportunity, lack of mistake, etc. [Rule 404(b)] (See Examples Below) o On Cross-Examination of a character witness [Rule 405(a)] (See Example Below) Note, in character questions, always first make reference to FRE 404(a) and 405! Then you may move on to other rules! (such as the impeachment rules FRE 607, FRE 608, FRE 609) Character of Defendant in Civil Cases Generally there is no character evidence allowed in civil cases. Most of the time, evidence in civil cases is offered for the purpose of showing circumstantial evidence to infer conduct in a civil case. There is no character as circumstantial evidence to infer conduct in a civil case. It is disposition evidence, in other words and it is not allowed in a civil case. Character is admissible in a civil case if it is a material issue in the case. If the character of a witness is itself directly a material issue in the case than you need to be able to show it. This situation is rare. There are situations when this happens: (1) defamation when truth is a defense (2) negligent entrustment (bus co. hired the town drunk as their bus driver, and the company should have known it), and (3) wrongful death (if husband was a horrible husband, the fact that he is dead may make wife better off and she wont get much money) (4) child custody (parental fitness of mother and father is assessed in order to serve the best interest of the child. Here being a good parent is an essential issue in the case.) Character of Defendant in Criminal Cases Again, this is almost never used to prove a material element of the crime. However, there are circumstances where it can be admitted as circumstantial evidence to infer conduct. Basic Rule: no bad character evidence at the initiative of the prosecution if the purpose is to show criminal disposition unless and until the D first opens the door. A defendant can put forth evidence of a pertinent character trait, (i.e.: showing good things about his character showing he would not commit a crime like this) but in doing so he opens the door to rebuttal evidence concerning the same trait by the prosecution (i.e.: showing bad things about his character showing he would likely commit a crime like this) (See FRE 404(a)(1)). The one exception to this: sexual assault and child molestation cases. FRE allows the Ds prior acts of sexual assault or child molestation to be shown by P. The D need not open any doors. This can be part of the Ps case in chief. Although this is disposition evidencehe did it before, therefore, he probably did it this timeit is still allowed. Also, there does not need to be a charge or a conviction.

Evidence Outline

Page 27

Definition of Pertinent Character Trait (FRE 404(a)(1))- traits that bear on the specific charge usually are considered pertinent, while those that dont bear directly on the crime, or that are too general (i.e. defendant is a good person or law abiding) are not. It is important to distinguish the difference between a character trait and a pertinent character trait. (ex: if someone robs you, this has nothing to do with whether they are an honest person or not.) A defendant cannot be acquitted based solely on character evidence, but they can create reasonable doubt based on the character evidence. Jury Instruction- The Eleventh Circuit has said that you can create reasonable doubt through the use of character evidence. Some courts have held that a jury should be told not to base their decision solely on character evidence, or that character does not excuse a crime, and to view character evidence along with other evidence. Problem 5-E Shes a known Thief Gretta is charged with shoplifting. Upon getting caught, she claimed the clothes were hers. She pleaded the same defense at trial. The P (during its case in chief) wants to offer a witness to testify that Gretta is a known thiefshe had a reputation of being a thief. (Reputation) P also wants to offer evidence that Gretta had been convicted of shoplifting at other stores 4 other times. (Specific Instances of Conduct) Answer (going through the forest) Preliminary: Whether Gretta shoplifted will be determined by the jury. (FRE 104(b)), but the Judge has to decide whether a there is a legitimate purpose to admit the evidence. Relevance: Possible evidentiary hypothesizes: AICT: she has stolen before and therefore she is likely to do it again. (not admissible 404(a)) Bad Person: she is a bad person and is therefore likely to steal. (not admissible 404(a)) Also, as it stands, there is no exception that applies, therefore it is not admissible. How the evidence would become admissible: If Gretta got on the stand and made a comment about her character, than the Prosecution could contradict her testimony with character evidence related to specific facts. Problem 5-A Fight in the Red Dog Saloon Facts: Defendant is accused of assault against victim in a bar fight. He claims self-defense. Prosecution calls Coach Jones, who says D is a violent person. Answer: This would be relevant (because it can help the finder of fact determine who the aggressor of the fight was), but under FRE 404(a) the first sentence bars this and none of the three listed exceptions apply. (We prohibit its use because of the AICT argument) Facts: D is going to call Reverend Graham to say he is peaceful. Answer: This would be relevant (because it makes it less likely that he is the initiator of the violence). This is forbidden by the first sentence of 404(a) because of AICT, but it fits into one of the exceptions under 404(a)(1). It is important to

Evidence Outline

Page 28

note, the defense has now opened the door to the prosecution attacking his character in their case-in-rebuttal. For instance, in rebuttal, the P could now call Coach Jones and offer his testimony. Character Evidence of Crime Victim If the victims character is relevant to a defense by the D, (i.e.: is a pertinent character trait) then the D can take the initiative to show the bad character of the victim in order to help the defense. Note this will open the door for rebuttal by the prosecution. (See FRE 404(a)(2)) Character of Victim Opens Door for Character of Defendant-If the D attacks the character of a victim, not only is the door opened to rebut this character evidence, but it also opens the door to evidence of the same trait in the defendant, even if the defense has not approached the issue of defendants character [See FRE 404(a)(1)]. Self-Defense Homicide Exception to Character and NON-character evidence (404(a)(2)(c)- Where a defendant is charged with homicide and has introduced evidence (including non-character evidence) that the crime was committed in self-defense, the prosecution is allowed to rebut that with character evidence. Ex: In Ds murder trial, if D presents an eyewitness that testifies the victim struck him first (this is non-character evidence) than the prosecution can introduce character evidence tending to show the trait of Victims peacefulness. Note, this only applies in homicide cases. If Ds witness offered this testimony in a battery case, Ps character evidence about Vs peacefulness would not be admissible. (Because we would be applying 404(a)(2)(b) and not 404(a)(2)(c).) Problem 5-B Red Dog Saloon-Part II Facts: Now Defendant offers testimony of Ernie who says that the victim is belligerent, aggressive, fight-picking fellow. The Prosecution objects for irrelevance and barred by the character evidence exception. Answer: This is relevant because it makes it more probable that Don started the fight and Vince acted in self defense. It does run into the prohibition of FRE 404(a) because it is AICT. But it fits into the exception in 404(a)(2). Because Defendant is claiming self defense, Ernies statements do show a pertinent trait that he should be allowed to prove. He can only prove this trait through reputation or opinion testimony (FRE 405(a)), however, because the victims aggressiveness is not an essential element of the self-defense defense. The offering party only can offer opinion and reputation to support their evidentiary hypothesis concerning the victim. Only on counter, may a party inquire into specific instances of conduct. FRE 404(a)(1)(2): If as a defense lawyer, you offer evidence of your own clients peaceful traits, then the prosecutor can rebut that same character trait in the client. If the accused in offers character evidence about the victim, the prosecution can rebut this evidence concerning the victim. Furthermore, if the accused attacks the victim, then the Prosecutor can attack the accused with character evidence. (Using Opinion or Reputation)

Evidence Outline

Page 29

In a homicide case, when the accused attacks the character trait of the victim, the prosecutor can not only rebut to show the victim was peaceful/nice/good, etc., but he can even attack the witness (with anything to rehabilitate the character of the victim). (This allows admitting things beyond character evidence!any attack or suggestion that a homicide victim is mean/bad/etc. you can show any evidence to rehabilitate your client.)

Problem 5-C Red Dog SaloonPart III Facts: If the judge lets Ernie testify about Victims character, should Ernie testify as described above? What kind of testimony is that? Ernie says Victim is a belligerent, fight-picking fellow with a real short fuse. Answer: As offered, the opinion testimony by Reverend Gram, Coach Jones, and Ernie all squarely fit FRE 405(a), where the basic message is that if character evidence is admissible at all, it may take the form of "opinion" or "reputation" testimony. Note: Once the D admits this testimony, the P may now rehabilitate the character of the Victim and attack the character of the Defendant. In other words, the attack on the character of the victim opens two doors for the prosecution: (1) rehabilitating Vinces reputation and (2) attacking Defendants reputation. (FRE 404(a)(1)(c)) Using FRE 404(a)(1)(c) and FRE 611(b)(2), the P might ask Ernie about Ds peacefulness (or lack thereof.) This cross examination would be allowed to occur even though technically it is direct questioning of the witness (sooner rather than later). Under FRE 611(b)(2), this would be subject to a foundation and only limited to opinion and reputation testimony. Once Ernie (the Ds witness), asserts that D is a peaceful guy, 405(a)(2) also kicks in to allow specific acts of conduct relative to D provided that Ernie is considered to be associated with D (FRE 611 (C)(3-C) or hostile to the P (FRE 611(c)(3-A)). Hostility towards P is not automatic, because it could just be that E knows V and not D. E could just avoid testifying about D simply by testifying that he knows V but not D. Also note, once the D admits this testimony, the P can encounter about specific acts which may rebut the character of VICTIM when they are cross examining the witness. (404(a)(2)) On redirect of E, Ds counsel would be allowed to ask about specific bad acts of Victim if the P asked E about specific good acts regarding Victim. Also, in order to admit this type of testimony, the proponent must lay a proper foundation. Laying the Foundation For Opinion/Reputation Testimony: It is important to determine whether the testimony is being offered as opinion or reputation because there is a different foundation that you have to lay based on whether the testimony you are offering is opinion or reputation. For opinion, you need to be acquainted with the person. The context matters. The pertinent character trait would need to be likely to be displayed

Evidence Outline

Page 30

in the given context. You would want to show the witnesses are personally acquainted with the person whose character they are prepared to assess. For reputation, you need to show a pertinent period of time. (i.e.: you need to have met/ known them before the underlying facts took place.) It is possible in this case that Jones, Gram and Ernie could also give reputation testimony, which would require them to testify that they talked with people in the community about Vince or Don. Here the foundation should indicate acquaintance with reputation in a pertinent period of time -- for testimony about either Don or Vince, that should mean acquaintance with their reputations before the fight became widely reported and the charges were brought against Don. **In both cases, the relevant community is important: in this instance the relevant community would be drinking buddies or neighbors, etc. Maybe even coworkers or class mates (anyone who can provide testimony about the opinion or reputation about the accused). It has to be that your knowledge was acquired in the regular course of human operation (i.e: you were classmates, neighbors, went to church together, etc.) Problem 5-D- What Price Truth? Facts: Same fight as above. At trial, after the Reverend testifies that the defendant is peaceable, the prosecutor asks the following question on cross-examination: Did you know that the defendant beat his wife? Answer: This is admissible. This is a specific act/specific incidence of conduct. FRE 405(a) allows on cross examination inquiry...into relevant specific instances of conduct. Therefore the Prosecution is allowed to ask Gram (who has already testified to the accused peacefulness) if he knows of specific acts which the accused has done which may show otherwise (i.e.: that he beat his wife). Good Faith Requirement: The lawyer must have to have a good faith basis for asking such a question. (this is not expressed in the rules). Asking a question for which you do not have a good faith basis is definitely unfair prejudice! (Ex of sources of good faith basis: convictions, medical records, police complaints, etc.) The good faith basis becomes important with specific actions, because there is likely other evidence relating to the specific actions to formulate the good faith basis. Under FRE 405, this other evidence (medical records, etc.) called extrinsic evidence would not be admissible, therefore in order to get this in at trial, you have to ask the question. Remember rule FRE 405 only allows inquiry. Anything other than that (eye witness, documentary) is not allowed under the language of this rule. Note, the judge may require you to show the evidence upon which your good faith is based not in front of the jury. What if in the hypo, the question was whether the witness knew the D had been charged with embezzlement Would this question be allowed? Answer: The P can argue this goes to how well the witness knows the D and how qualified he is to testify to the Ds reputation. However, this evidence would be so damaging that courts usually restrict them to the subject area of directi.e. to the pertinent character trait. Violence and embezzlement are not in the same subject matter.

Evidence Outline

Page 31

What if D was not the D and was instead the witness? In this case the questions might be allowed, but only if G had testified to Ds truthful character, which would only have been allowed if D testified and the prosecution had attacked him under FRE 608. Would questions concerning whether he beat his wife be admissible? Start with 404, or 405 then go to 607, 608 and 609 (conduct as they relate to credibility). BarBri Example: The D (Hector) is charged with assault or murder and claims self defense saying the victim was aggressive toward the D. The D can attack the reputation of the victim saying the victim has a reputation of being a violent person in the community. Then the P may rebut by either showing the good character of the victim or by attacking the character of the D by the use of opinion/reputation evidence. Ex: 5 possible scenarios: 1. Scenario 1: H is on trial and calls a witness that wants to testify V is known in the community as a violent person. (This is permissible) 2. Scenario 2: In response, P wants to call a witness to testify that V had a good reputation and was a peaceful person. (This is permissible) 3. Scenario 3: Also, P wants to call a witness that says H (the D) has a bad reputation of being a violent person. (This is permissible even though H did not take the initiative to show his own good character. By attacking the character of V, the D opens the door for the P to attack the Ds character.) 4. Scenario 4: H calls a witness that testifies that he was present and saw V use a beer bottle to almost kill 3 other bar patrons in 3 other bar room brawls (This is not permissible because it is a specific act of conduct and does not fall into one of the exceptions for the use of specific acts of conduct to show character evidence!) 5. Scenario 5: H calls a witness that testifies like he did in scenario 4, but the witness adds, I told H about it before the incident in this case. (If this is offered to show the character of the victim for violence, then this is not permissible.) But it has another relevant argument which would make it admissible to show the state of the mind of the accused. (one of the MIMIC exceptions when you can show a specific act of conduct) This shows H knew V almost killed 3 other people, so it was reasonable for him to be afraid for his life. This fear was reasonable because of what he had been previously told. The limitations of Specific Prior Acts Remember Prior Specific Acts can only be used where character or a trait of character is an essential element of a charge (criminal), claim (civil), or defense [Rule 405(b)] or On Cross-Examination of a character witness [Rule 405(a)] (See Example Below) MIMIC Rule: to show motive, intent, plan, identity, opportunity, lack of mistake, etc. [Rule 404(b)] (See Examples Below) 1. The first major exception is where the character trait is an element of the crimethis RARELY happens. The book cites one example for criminal cases:

Evidence Outline

Page 32

reputation for chastity as an element of the crime of seduction and four examples for civil cases. (See above) 2. The second major exception for when you can admit a Ds specific prior acts is on Cross-Examination of a character witness (FRE 405(a)). When the accused calls a character witness, the prosecution can cross examine that witness on specific instances from the defendants past that would not be admissible otherwise. Possibility of Prejudice- Although the mere asking of these questions might prejudice the defendant, they are seen as having waived protection against questions by calling a character witness in the first place. Extrinsic Evidence- If the prosecutor does not like the answer he gets on cross-examination he cannot bring in extrinsic evidence of the specific acts like calling the actual victims of those acts as witnesses or admitting a document which proves those acts into evidence. Ex where prior specific acts allowed on cross-examination of a character witness: Ds witness is testifying to a specific character trait of the D (such as the peacefulness of the D in an assault case.) In order to do this, the Ds character witness cannot give specific instances of prior acts, he can only give opinion and reputation evidence. If the Ds witness does this, the P can then respond by bringing up the Ds prior specific bad acts on cross-examination of the Ds character witness testing the witnesses knowledge of the Ds reputation. Logic behind the admissibility: This question is admissible only to test the witnesss knowledge of the Ds reputation. (ex question: Did you know D has been arrested 6 times? OR Have you heard of any specific act which would tarnish the reputation of the D?) In asking these questions, the P is not trying to prove the truth of these prior acts. He is only saying, if the Ds witness is qualified to testify to the Ds reputation, then he should know about these prior acts. See Michaelson case Jury Instructions: In instances like this, it is important to give the jury instructions which clearly state these questions are permitted only to test the standards of character evidence that these character witnesses seemed to have. There is nothing before you on the truth of any of these questions. You only use them in weighing the evidence of the character witnesses. The P cannot just make up questions to raise the stench of misconduct. Michaelson requires the cross examiner to demonstrate privately to the court the fact of the question they are asking. Note: If the witness says No, Ive never heard of that, then the P is screwed. He cannot then admit documents or call other witnesses which prove the D has been charged with those other crimes, because that is outside the scope of cross examination. If the P chooses not to cross-examine the Ds character witness, he can always call in his own character witness of the D to rebut what the Ds character witness had said. However, this witnesss testimony would be limited to reputation and opinion testimony.

Evidence Outline

Page 33

3. The third main time you can admit a Ds specific prior acts is if there is some other relevance apart from showing criminal disposition (FRE 404(b)).(i.e.: a reason other than an AICT) MIMIC RULE--Motive, Intent, Mistake (or lack of), Identity, Common plan and scheme (not an exhaustive list) Four Pronged approach to the MIMIC RULE: Many courts use a four prong approach when admitting evidence under Rule 404(b) in which the judge does the following: 1) Decides whether the evidence is offered for a proper purpose (FRE 104) 2) Decides whether the evidence it is relevant for that purpose (FRE 402) 3) Decides whether its probative value is outweighed by the risk of unfair prejudice, (FRE 403) and 4) Gives a limiting instruction on request. Notes Regarding FRE 404(b) Remember, provided that upon request by the accused, the prosecution in a criminal case shall provide reasonable notice in advance of trial, of the evidence to be presented. The court can excuse pretrial notice if good cause is shown. Also Remember FRE 403 still applies to the MIMIC rule!! So even though some particular prior act would be relevant to show one of these things, the judge can still exclude it under FRE 403. Possible unfair prejudice dangers: o That the jury will use the evidence to believe that D is a bad person, and thus deserving of conviction or a finding against them in a civil case. This is always an impermissible use. o That the jury will conclude a general bald inference that D acted in conformity therewith on a particular occasion. Also remember that MIMIC rule applies to both CIVIL and CRIMINAL cases. Independent instances which would allow in prior acts of misconduct: 1. To show motive: Ex: the D is charged with killing a detective. Can the P show the Ds wife went missing three years prior to this current charge? If the P is admitting this evidence to show the D killed the detective because the detective was about to arrest the D for killing his wife, this goes to the Ds motive and it admissible. Therefore, in Ds murder case of the detective, the details of the disappearance of Ds wife would be admissible to show motive of killing the detective. 2. To show intent/state of mind: (such as knowledge): Ex: D is charged with receiving stolen goods. D claims he didnt know the goods were stolen. In this case the P could offer evidence showing the D has received stolen goods on 4 prior occasions and knew the source of the shipments. This shows the D knew these goods were stolen. Problem 5-E Drug Sale or Scam

Evidence Outline

Page 34

The defendant meets with an undercover cop to sell coke. Defendant gets nervous when he sees dust on the bills and walks out on the deal. They arrest him and he has no Coke on him. He says he was just going to rip the guy off and not actually sell him the coke. (i.e.: I am a thief, not a drug dealer.) They try to get in evidence of past drug dealing crimes under Rule 404(b) to show that he did intend on this occasion to sell the drugs. Stipulation- Recall that in Old Chief the Court held that as a matter of Federal law, the prosecution cannot admit evidence of a prior conviction (where the criminal conviction is an element of the current charge) if the defense offers to stipulate that the element is satisfied. This holding does not extend to use of prior acts to prove intent, but some courts have held that where intent is not an issue in controversy the evidence should be excluded under a 403 analysis (e.g. the defendant claims mistaken identity but offers to stipulate to intent if the jury finds he was the perpetrator). Answer: Relevance: AICT: he sold drugs in the past so he is likely to have been selling them here (prohibited 404(a)(b)) Bad Person: he is a bad guy and therefore he sells drugs (prohibited 404(a) (b)) Hyper-specific: the evidence is being used to show intent, because the D is denying he intended to sell drugs. FRE 403 analysis: The risk of unfair prejudice is not as prevalent here because, the D is claiming he is a thief, not a drug dealer. So we dont have to worry about the jury getting a negative impression of him. They probably already have one based on his own admissions. Jury Instruction: Generally, when you allow such evidence to be used, the judge should instruct the jury they are to use the character evidence only in a certain way. (FRE 105) The jury should never use character evidence alone to convict someone. 3. To show Mistake of Lack of Mistake This is often used in child abuse cases. The evidence shouldnt really be used to identify the parent as the source of injuries (but if the parent is the sole caregiver it is pretty obvious on its own; and often extrinsic evidence will link one parent to the injuries), but several prior injuries in a short period of time may be used to show that these ones were not accidental. While expert testimony by a doctor concerning the nature of the injuries may be helpful, if there are so many similar trips to the hospital over a short period of time common sense alone would lead one to infer that the instant injuries were no accident. Problem 5-I It Was an Accident A boy is brought to the hospital with many injuries. The mom says he fell down the stairs but he has been to the hospital many times recently with similar injuries. The Prosecution wants to admit these prior instances into court. Relevance hypothesis: Because the child has been in the hospital with severe injuries in the past, we want the jury to infer the mother has recklessly or intentionally injured the child and this was not an accident or mistake.

Evidence Outline

Page 35

The unfair prejudice is not outweighed by the probative value. It is important to note, we are still very concerned about the possibility of unfair prejudice here. With child abuse, we often allow stuff like this to be admitted out of necessity. These things happen behind closed doors and there are no witnesses usually. We are not trying to show her character, but we are trying to show that the pattern of injuries shows the intent or absence of mistake which is part of our list of excepted uses for character evidence under 404(b). The jury will decide whether this conduct was in fact abuse under 104(b). Uncharged-non-conviction bad acts are left to the jury to decide. Testimony by an expert witness should not be required It is common sense a child who has been to the hospital in one year three times is not normal. However, it should be allowed to assist the jurys understanding of the case. Note: Too many accidents may tend to be unfairly prejudicial because it would increase the likelihood that the jury will think the D is a bad person. Often in these cases there is more than one potential villain because more than one person provides care for the child. Arguably, it should be enough to show the injuries happened at home, even if the D is one of two parents. Two important cases for 104(a) review Bourjaily and 104(b) review Huddleston.

4. To show identity: D is claiming it was not him. Therefore, any prior conduct that connects the D to this crime is admissible. Ex: if H robbed As house 3 years ago. He is convicted and sent to prison and then released. Then there was a murder with As gun. A says that gun is one of the things H stole when he robbed As house. At Hs trial for murder , P can admit Ds conviction of robbery because it tends to show that it was H who committed the murder not A, because H stole As gun. Modus operandi evidence: where a defendants past crime or act contained sufficiently distinctive qualities that match those of the current crime, the evidence of the past crime can come in to link the defendant to the crime, rather than to show his propensity for committing it. Prior act must bear singular strong resemblance to the charge offense, and similarities must be sufficiently idiosyncric to permit inference of pattern. Keys to modus operandi: o Close resemblance o Distinctive nature of prior charged misdeeds Ex: The D is charged with a crime which is distinctive and unusual in its nature. Prior instances in which a person was killed in this same unusual and distinctive nature can be admitted. Ex: in 2005, Hs wife dies drowning in a bathtub. In 2000, 1996, and 1994 the same thing happened to Hs wives. These prior instances will be admissible when D is prosecuted for the 2005 killing. In order for this to apply, there has to be a distinctive element to the crime charged and the prior crime(s). (A trademark)

Evidence Outline

Page 36

Problem 5-G He Came in Running All Hunched Over: Two men are accused of robbing a bank. Eyewitnesses testify that one of the men ran in hunched over, that two men were seen arriving in separate light and dark cars, in the morning. One of the men is identified by an eyewitness, and the other confesses to eight other distinctively similar robberies using the same modus operandi. Issue: Whether the confession of the other man is admissible. Answer: Yes. The fact that defendant committed 8 distinctive robberies, and we know someone committed a similarly distinctive robbery here containing the same element is admissible. The fact that defendant committed 8 distinctive robberies makes it likely that he committed other similarly distinctive robberies, and we know we have a similarly distinctive robbery here, so it makes it likely that it was he who committed the offense. Here there is a close parallel between the charged crime and other acts. As an attorney, you need to convince the judge that this evidence is a consistent/common conduct that can link the D to the crime. The conduct must be very specific facts that are unique and very similar to the crime. Reverse Modus Operandi- A defendant can also show prior crimes or acts of a third person to show mistaken identity provided the other persons crimes are sufficiently signature like to link them to the instant crime.

5. To show common plan and scheme: the D is charged with bank robbery. The P would be able to show that the day before the D stole a truck if the truck was used in the bank robbery, because it is all part of the same transaction/preparation. Problem 5-H The Corrupt Judge A Judge is charged with several acts of extortion for taking bribes from lawyers for ruling one way or another. In addition to the individual charges, he is charged under the RICO act which makes it illegal to operate an enterprise in interstate commerce through a pattern (i.e. more than one act) of racketeering. The prosecution wants to admit testimony by a lawyer that he bribed the Judge several times over a long period of time, but the lawyer is unable to name even one case in which this occurred. Should this be let in? Answer: Testimony that defendant engaged in innumerable but unspecified instances of bribery tends to tar the judge with a broad brush and put him in a disadvantage to defending. (As to RICO): It does not matter here that the P prove any one particular bribe, for Ds activities in the aggregate indicate the plan or design. RICO only charges people for taking bribes as part of an overall and ongoing effort to exploit the criminal court system. Therefore, specific instances do not have to be annunciated under RICO. (As to extortion): The testimony will less likely be admitted to show intent to take a bribe in connection with the specific instances, since taking numerous bribes in the single professional setting of the courthouse may not be strong enough to show intent. If this argument failed, D would at least be entitled to

Evidence Outline

Page 37

a limiting instruction that the testimony is admitted only to show D operated an illegal enterprise, and cannot be used to prove bribery. Common Objective- The prior acts must be integral components of an overarching or common objective and cannot just be unlinked acts that are similar in nature and close in time to one another. Convictions verses Non-Convictions in Criminal Cases Two types of convictions: 1. Un-charged non-conviction bad acts (such as in it was an accident) and 2. Actual convictions Huddleston test: THE JUDGE, 1. Decides whether the evidence is offered for a proper purpose, [404(b)[2]] 2. Decides whether it is relevant for that purpose [FRE 401&402] 3. Decides whether its probative worth is outweighed by the risk of unfair prejudice, [FRE 403] and 4. Gives a limiting instruction on request. [FRE 105] Explanation of RELEVANCE ANALYSIS (Under Huddleston): There are in effect TWO required relevance findings. 1. The JUDGE's finding of whether evidence fits within one of the established exceptions to the general prohibition against character evidence. Malavet thinks this is a 104(a) exercise, but whether the court is acting under FRE 104(a) or FRE 104(b), as to this part of the analysis was not that important to the Supreme Court. 2. Under FRE 104(b), the judge must admit the relevant prior acts evidence if the government adduces enough proof so that a reasonable jury could find by a preponderance of the evidence that such acts did in fact occur. (The judge has already decided what the permissible inferences that the jury can draw from the facts are). 3. If you get this far, i.e., if the evidence may be admitted you still must pass the final hurdle, FRE 403. NOW THEN comes the balancing of the legitimate uses of prior acts evidence against those prohibited by the general character rule of 404(a) (supplemented by 405(a)) and by the specific prohibition of 404(b)[1]. 4. Finally, if the evidence is admitted, upon request, a rule 105 instruction should be given on the proper use of the evidence. How do we go about proving a non-conviction act? (such as an acquittal, not guilty, confession that someone has committed acts where they have not been tried, etc.) The rules do not require a preliminary finding by the court that the government must prove the prior act actually happened. It is up to a jury to find by a preponderance of the evidence that the act actually occurred.

Evidence Outline

Page 38

The judge plays a screening role, asking only whether there is sufficient evidence to enable the jury to find that the defendant committed the prior act.

Huddleston is the Federal standard. That is, proof of a prior crime is relevant if the jury can reasonably conclude: by a preponderance that the act occurred and that defendant was the actor. Some states such as the state in Bourjaily, require the judge to find by a preponderance of the evidence that the act actually occurred before he can admit it. Footnote 6: In a remarkable footnote [footnote 6], Huddleston agreed "that the strength of the evidence establishing the similar act is one of the factors the court may consider when conducting the Rule 403 balancing." Does it follow that courts can, if they want, reintroduce the clear-and-convincing standard? The Exception of Sexual Misconduct of a Victim: Rule 412. Sex Offense Cases; Relevance of Alleged Victim's Past Sexual Behavior or Alleged Sexual Predisposition (a) Evidence generally inadmissible. The following evidence is not admissible in any civil or criminal proceeding involving alleged sexual misconduct except as provided in subdivisions (b) and (c): (1) Evidence offered to prove that any alleged victim engaged in other sexual behavior. (2) Evidence offered to prove any alleged victim's sexual predisposition. (b) Exceptions. (1) In a criminal case, the following evidence is admissible, if otherwise admissible under these rules: (A) evidence of specific instances of sexual behavior by the alleged victim offered to prove that a person other than the accused was the source of semen, injury, or other physical evidence; (B) evidence of specific instances of sexual behavior by the alleged victim with respect to the person accused of the sexual misconduct offered by the accused to prove consent or by the prosecution; and (C) evidence the exclusion of which would violate the constitutional rights of the defendant. (2) In a civil case, evidence offered to prove the sexual behavior or sexual predisposition of any alleged victim is admissible if it is otherwise admissible under these rules and its probative value substantially outweighs the danger of harm to any victim and of unfair prejudice to any party. Evidence of an alleged victim's reputation is admissible only if it has been placed in controversy by the alleged victim. In general, you cannot talk about a victims sexual predisposition in order to show something like the V consented in a sexual misconduct case.

Evidence Outline

Page 39

RAPE SHIELD STATTUES: if the accused is charged with sexual misconduct, there can generally be no evidence offered to prove the alleged victim engaged in other sexual behavior or no evidence to prove sexual disposition of the alleged victim. (FRE 412) Generally, prior sexual acts are not going to be admissible under the FREs unless there is a specific prohibition in FRE 412, The Sixth Amendment or FRE 404(b). o EXCEPTIONS: there are three situations where specific instances of sexual behavior may be shown: 1. Explaining Physical Presence/To show a third person was the source of seaman, or injury (i.e.: The D says it wasnt him who caused the injury or seaman it was another party) FRE 412(b)(1)(A) 2. Past Sex Between Victim and Defendant/Prior acts of consensual sexual activity between the alleged V and the D himself. (That is, we did it before, and therefore it was consensual this time.) FRE 412(b)(1)(B) 3. Constitutional Rights/If the constitutional rights of the accused require this sexual conduct evidence to be admitted. This exception is very vague, but some courts have allowed sexual history in where it shows a motive for victim alleging rape or where it shows a bias against the defendant. Sixth Amendment confrontation clause: in all criminal prosecutions, the accused shall enjoy the rightto be confronted with the witnesses against him. [412(b)(1)(C)] Note: FRE 403 is still in play here. There still must be notice to the complainant in advance of trial. There must be an in camera closed hearing and the judge must find the probative value outweighs the value of harm or unfair prejudice to the victim or any other party.

Reasoning behind Rape shield Statutes: To avoid embarrassing or humiliating complainants in rape cases To encourage (avoid encouraging) victims to report sexual assaults The conviction that judges could not be trusted wisely to exercise discretion to weed out proper from improper questioning of complainants in rape cases. Problem 5-K. Ordeal of Leslie or Fred Leslie comes home, acting funny, and tells her friends she was raped. She goes to the clinic where she has bruises and semen within. She said that Fred did it. She saw him at the luau and they left together, went to a lonely spot and his advances went too far. Fred is charged with rape. He claims that Leslie consented and that (3) they had sex once the previous summer. He also offers testimony from his friend Greg who says (1) Leslie is an easy mark/sexually active and testimony from (2) Thomas who says he had sex with her that same night. The prosecution objects. The physical evidence is fairly strong. Do you let this evidence in? Evidentiary Hypothesis: Leslies disposition is relevant (whether proved by opinion or reputation) in suggesting that she engages in sex voluntarily, hence that she did so with Fred. Response: a womans sexual disposition is irrelevant.

Evidence Outline

Page 40

(1) Not admissible. This is exactly what the rape shield is trying to exclude. This is an AICT and is specifically prohibited under 404a and 404b[1]. Go to 404(a)(2) because it is a criminal case dealing with character of the alleged victims. 404(a)(2) directs you to FRE 412. (2) Probably Admissible. May explain the semen/bruises, admit under 412(b)(1)(A). D admits they had sex so it is probably more probative with respect to the bruises, which deal with consent more than the semen. But if the prosecution never brings up the bruises, it is probably not admissible. (3) Admissible. Admit under 412(b)(1)(B). Note: under 403, this does not have a very high probative value because just because they had sex 1 year ago does not mean she consented now. In any case, it seems doubtful on these facts that a court would fund such proof of specific conduct with the defendant so lacking in probative worth as to warrant exclusion on that ground. Note: What about where there is a say no, but mean yes? There must be prior evidence of a conversation that illustrates that no would mean yes. How the civil context differs from the criminal context: Civil Cases- 404(b)(2) only allows evidence of victims sexual disposition if its probative value substantially outweighs any harm to the victim or prejudice to any party (reverse FRE 403 test). Reputation evidence (against victim) is only admissible if put in controversy by the victim. Problem 5K Acting Out in the Assembly Line An employee is charging an employer with sexual harassment. The Prosecutor wants to argue that she is a stripper and is therefore sexual harassment proof. Being a stripper is considered character evidence. (this brings in the prohibition scheme of 404(a) and 412). Now we have to see if there are exceptions: There are no exceptions in 404. Although 404(a) applies in civil and criminal cases, the exceptions apply only in the criminal context. Rule 412(b)(2) is the exceptions in civil cases. Here we have a presumption of inadmissibility. This presumption is rebutted only if the probative value substantially outweighs the danger of harm. It is likely here the probative value would not outweigh the danger of harm. Note: out of work behavior is not relevant. Out of work behavior known by people in work is also not relevant. But, if there is co-participation between co-workers and plaintiffs outside the place of work (like an orgy), this would be relevant. (ex: Provocative Dressing, Conduct outside of work (even if known by co-workers) is not going to be relevant.) Sex Crimes Continued (Prior Offenses/Character of Defendant) Rule 413. Evidence of Similar Crimes in Sexual Assault Cases (a) In a criminal case in which the defendant is accused of an offense of sexual assault, evidence of the defendant's commission of another offense or offenses of sexual assault is admissible, and may be considered for its bearing on any matter to which it is relevant. Rule 414. Evidence of Similar Crimes in Child Molestation Cases

Evidence Outline

Page 41

(a) In a criminal case in which the defendant is accused of an offense of child molestation, evidence of the defendant's commission of another offense or offenses of child molestation is admissible, and may be considered for its bearing on any matter to which it is relevant. Rule 415. Evidence of Similar Acts in Civil Cases Concerning Sexual Assault or Child Molestation (a) In a civil case in which a claim for damages or other relief is predicated on a party's alleged commission of conduct constituting an offense of sexual assault or child molestation, evidence of that party's commission of another offense or offenses of sexual assault or child molestation is admissible and may be considered as provided in Rule 413 and Rule 414 of these rules. FRE 413, 414, 415 invite prosecutors to prove sexual assault or child molestation by means of evidence that the D assaulted/molested others. An actual conviction for a sexual offense is not necessary according to Congressional comments. BIG HUDDLESTON DEBATE: Under Huddleston the jury should make the determination under 104(b) if there is any question as to whether the prior act occurred or was a sexual offense. However, may State courts do not like Huddleston (i.e.: the Bourjey cases) would probably use 104(a) (let the judge decide if the prior act occurred) before admitting the evidence. Basic rundown: 1) FRE 413 where a D is accused of sexual assault, evidence of other sexual assault offenses is admissible and may be considered 2) FRE 414 trials for child molestation, evidence of other child molestation offenses is admissible and may be considered 3) FRE 415 extends 413 an 414 to civil cases FRE 413 through 415 say that the evidence can be considered for any matter to which it is relevant. Therefore, in these cases, we look to the points of similarity between the prior act and the current act to determine if the prior act is similar enough to be relevant. Depending on the court, you may not even need to go through a FRE 403 analysis. (i.e.: some courts look at FRE 413 as being automatic admissibility rules, however Malavet does not.) To make up for a little unfair prejudice, the judge can always instruct the jury accordingly to avoid the unfair prejudice that may occur.

Problem 5-L I Told Him to Stop Craig is on trial for raping Karin. She says it was rape and he says she consented. The prosecution wants to introduce (1) Laura, Craigs previous girlfriend, who will testify that he once tried to rape her and (2) a judgment of conviction against Craig from another state three years earlier for fondling and sodomizing the 13 year old daughter of the woman he was living with after they drank beer and played strip poker. Can these two things be admitted? Answer: (1) Prior instance of conduct that did not result in conviction.

Evidence Outline

Page 42

Because this was never proven in court, the Court must allow the jury to determine if the prior instance is true under 104(b). Under Huddleston, deciding whether the specific instance of conduct occurred should be left to the jury. The judge under 104(a) only determines whether the testimony is admissible. In this case, Lauras statements are a similar scenario to this charge so it is highly relevant under 413 and should be admissible. (i.e.: initial intimacy, escalating advances, verbal refusal, physical resistance leading to assault.) It would seem that by believing Laura, it would support the conclusion that Craig is by disposition prone to press for intimacy once he and his mate have started down that road, even when she says no and offers physical resistance. In other words, under 413, we are asserting an AICT. This is way broader than 404(a) and (b) standard. If you were the prosecution in this case, this is what you should argue to admit the evidence: 1. There is need for this evidence 2. Rule 413 allows me to use this for this purpose, 3. Rule 403 is designed to favor admissibility 4. I have a legislative finding that I am allowed to do it. 5. Any unfair prejudice will be guarded by the judge through FRE 105, 106 (jury instructions.) Argument against admitting the testimony: One prior misstep does not establish this trait, and all a single act can show is something like modus operandi. In which case, this does not really go to modus operandi because there is not enough significant parallels to support this inference from one act. You could also argue there are not enough similarities for this to be admitted under FRE 413. Eventually, the P would probably win this one because under 413, evidence of similar sex crimes is admissible. It is important to notice the very different arguments that are being used under 404 verses 413. Under 413, the prosecution can argue, because he did it before, he is likely to do it again. However, under 404(a) and (b) this is not allowed, because it is an action in conformity therewith (which is specifically prohibited). Under 404(a) and (b) you would need to come up with another probative value. The key under 413 is to find as many similarities as you can (or dissimilarities depending on which side you are on) to determine if the prior situation is similar enough to be relevant to admit. A 403 balancing act still occurs. The judge must make sure the jury will not be overwhelmed by unfairly prejudiced. The case book suggests it is possible to read 413 as automatic admissibility (i.e.: not going through the 403 test). Most courts hold this is not an automatic admissibility ruleFRE 403 still has to be addressed. (2) Specific Instance of Conduct that Resulted in Conviction. Points of similarity: (ways this prior act is relevant): He used alcohol to sexually molest the child and lower her inhibitions, he initiated in a friendly way of contact before he raped the girl, he seeks sex in a socially inappropriate manner. This is relevant. Skip 404 because we are acting under 413. Remember 413 is still subject to the 403 balancing.

Evidence Outline

Page 43

Defense Counsel should try and argue the unfair prejudice far outweighs the probative value. In this case there is a real risk of prejudice. The guy is a child molester. People hate child molesters. Therefore, this probably has less probative value and a higher risk of prejudice. The assault against Laura is more likely to be admitted than this is. Side Note: because the D intends to testify, FRE 608 and 609 will be available. Although sexual assault itself is not a crime involving false statements, the circumstances may support such a finding for 608(b) or 609(a)(2) purposes. Even under these rules, the conviction is still probably going to be weighed by the unfair prejudice. Furthermore, under 609, no mention of the underlying facts would be allowed, yet under 404 and 413, those facts could be well developed for the jury. One more side note: the attacks on the victim of rape allowed by FRE 412 will probably not trigger FRE 404(a)(1)(C). VII. HABIT AND ROUTINE PRACTICE Rule 406. Habit; Routine Practice Evidence of the habit of a person or of the routine practice of an organization, whether corroborated or not and regardless of the presence of eyewitnesses, is relevant to prove that the conduct of the person or organization on a particular occasion was in conformity with the habit or routine practice. Rule 406 broadly states that habit of a person or routine of an organization is relevant to prove conduct in conformity on a particular occasion. The rule states that regardless of the presence of eyewitness or other corroboration of the evidence is unnecessary. Proof of habit must be very specific and must be regular and almost automatic behavior. Habit is relevant to prove that the conduct of person on a particular occasion was in conformity with the habit or routine practice.

Difference Between Character and Habit: Character -Evidence is a generalized description in respect to a general character trait such as honesty, temperance, or carefulness. -Character is more general (a tendency applicable to the varying situations of life) -Involves moral overtones that are not part of our habit. Habit -Is more specific, designating a regular practice of meeting a particular kind of situation with a certain type of conduct -Constitutes non-volitional behavior (reflex, semi-automatic, regular) -Specific situations and particular kinds of situations

Habit or not Habit? The wearing of seatbelt: habit Previous Speeding tickets: non habit (specific instances of conduct character)

Evidence Outline

Page 44

37 counts of drunken aggressive behavior: non habit (violent when angry character) Particularized Behavior

Problem 5-M Death on the Highway Teel and Finney got in a car accident in which both parties died and there were no witnesses. The police report indicated that the accident could only have occurred where one of the parties was at fault. The Plaintiff (Teels estate) wants to introduce testimony that Teel was always a good and safe driver. This is not habit because it is too general a type of behavior (it is more like character evidence than habit). Evidenec of carefulness or carelessness is not considered habit. If he had testified that the driver travels Route 46 everyday, that they regularly drive with him, and that Teel always uses a signal when turning that would be habit because it is more specific. It is important the testimony shows more than a general disposition to be careful, but it must show a regular practice of meeting a particular kind of situation with a specific type of conduct. In this case, we are not too concerned with the unfair prejudice because Mr. Teal is dead. Regularity Problem 5-N The Exploding Can A mechanic (Halleck) was trying to heat up a can with water and Freon in it with a heating coil and the can exploded. He sues the Freon manufacturer. The manufacturer says he ignored warning labels on the can (i.e.: that he heated up the can past 130 degrees). They want to introduce testimony by Newsome, a fellow worker, that says Ive often seen Halleck use a coil to heat water and thus to heat cans of Freon. Newsomes testimony describes behavior that is specific and if not sub conscious, at lease semi-automatic. Thus it qualifies as habit under 406. However is often enough to show habit? The answer will turn on how many times Newsome saw Halleck use the coil. Alternative Argument: You could potentially classify this as character evidence under 405(a)an eyewitness opinion classifying specific instances of conduct. If it is character evidence, it will not be admissible. When does something go out of the realm of 404(b) and into the realm of 406? There needs to me enough to establish a mode of behavior that has become nearly or completely involuntary. Proof of a deliberate repetitive act by one in complete control of the circumstances should be admissible because it is highly probative. Questions to ask to establish a habit: o How long have you worked together? o How many times have you observed him install Freon systems? Judges, under 104(b) determine whether a foundation has been laid to establish habit or routine practice and determine if it is reasonable to classify the conduct as habit or routine practice rather than character evidence.

Evidence Outline

Page 45

Practices of an organization/Habit of an organization Problem 5-O Was He Served? Manuel is charged will illegal entry after previous deportation. At trial, the government must prove Manuel was served with the warrant of deportation. As part of proof, government calls an INS officer to describe the procedures of what happens in the ordinary course of deportation procedures claiming part of the procedure is to hand the immigrant a copy of the warrant. The officer has never executed such a warrant but has been told this is how it works from one of his detention officers. Answer: A person who does not himself follow the routine has the necessary knowledge to testify to habit. This is as long as he has observed it firsthand and understands how it works, meaning he has seen people or machines performing their functions and understands the underlying purposes and relationships. But the officer apparently has not observed service of the sort of papers involved here. Therefore the situation is troubling. If he has heard "what goes on" from several people in a position to know and has corroborative firsthand knowledge about organizational practices, circumstantial knowledge might be all right. It generally suffices when a party offers business records under FRE 803(6). On these facts, the officer seems to fall below a reasonable minimum. If he had some firsthand corroborative information, then he would be an adequate witness. The government should produce a witness who serves warrants to describe how it is done, or at least one who has observed warrants being served. However, the actual court thought it was sufficient that the agent had extensive experience in the normal rule of things and the normal processes of deportation and was thus familiar with the procedure. Remember, you still need to establish facts that show a pattern of company habit.

HEARSAY
I. HEARSAY
IN

GENERAL

Rule 801. Definitions The following definitions apply under this article: (a) Statement. A "statement" is (1) an oral or written assertion or (2) nonverbal conduct of a person, if it is intended by the person as an assertion. (b) Declarant. A "declarant" is a person who makes a statement. (c) Hearsay.

Evidence Outline

Page 46

"Hearsay" is a statement, other than one made by the declarant while testifying at the trial or hearing, offered in evidence to prove the truth of the matter asserted. Rule 802. Hearsay Rule Hearsay is not admissible except as provided by these rules or by other rules prescribed by the Supreme Court pursuant to statutory authority or by Act of Congress. Definitions: o Declarant= the one who makes the out of court statement o Witness=The one who hears it who comes into court to report what the declarant said. o Writing= Declarants own writing to report on what the declarant wrote earlier. Hearsay is not admissible unless it fits an exception or exclusion Whether something is hearsay or not depends on the purpose for which it is being offered. Rationale/policy behind the hearsay rule: we exclude hearsay because it denies to the opponent the opportunity to cross examine the one whose perception, memory, and sincerity is really important. The opponent cannot be denied the opportunity to cross-examine the person who said the statement. Excluding hearsay is a superior kind of truth seeking contentbetter than the alternative: Although excluding hearsay does not ensure the truth, excluding hearsay and forcing a live witness in front of the jury, are superior methods than the alternative (allowing the completely uninterrogated statement, unviewed testimony and not underoath testimony).

Why exclude hearsay? Absence of cross examination Absence of demeanor evidence (the jury cannot see how the declalarant reactsthe jury misses clues, impression, inflection, expression and appearance to convey) Absence of the oath There is a clear preference for live testimony over out of court statements Why do we distrust hearsay? Risk of Faulty Memory Risk of Misperception Risk of Ambiguity or Faulty Narration Risk of Distortion

Forest for Hearsay: 1. Is the offered evidence relevant? (FRE 401, 402)

Evidence Outline

Page 47

You need to establish the evidential hypothesis that favors admissibility or the irrelevant evidence will be excluded under FRE 402 and the analysis of admissibility stops here. 2. Is the offered evidence hearsay? (FRE 801): If the offered evidence is (1) within the definition of hearsay, and is not saved by an (2) exemption or (3) exception, it must be excluded under FRE 802, and the admissibility would stop here. a. Does the evidence fit within the definition of hearsay of FRE 801(a)(b)(c)? If no, than it is not hearsay and move on to the FRE 403 question. (Remember, you may have to explain here how the evidence is hearsay for one purpose but not hearsay for another.) If the answer is Yes you move on to FRE 801(d) and other sections of chapter 8 to see if there is an applicable exemption or exception. b. Even if it fits within the FRE 801 definition of hearsay, is it nevertheless within some exemption that expressly defines it as not-hearsay or non-hearsay? (FRE 801(d)) If the answer is yes, move on to FRE 403. If the answer is no, move on to chapter 8 of the FRE. c. Even though it fits the FRE 801 definition of hearsay and despite its failing to be exempted by FRE 801(d), is it within some exception found in the rules, especially in FRE 804 and FRE 804? 3. Should the offered evidence be excluded, despite being relevant, and regardless of the answer to the hearsay question (FRE 403)? If the probative value is substantially outweighed by the danger of unfair prejudice, relevant and non-hearsay evidence will still be excluded. 4. Always keep in mind the limiting instruction if you allow evidence to be admitted. Problem 3-A Three See A Robbery 1. Listener is testifying that he heard Plaintalk say Higgens did it. This is the direct statement of the point to be provedit is plain hearsay, fitting FRE 801(a)-(c). If the it you are referring to is the crime (then this goes directly to the truth of the matter asserted, i.e.: that Higgens committed the crime) and therefore is hearsay. Therefore this is not admissible (unless there is an exception). 2. Lissner said I heard Serchev say Higgens went out of here carrying money bags. This tends to show on the day this bank was robbed, and the time this bank was robbed, Higgens was observed carrying money out of the bank, therefore he robbed the bank. It tends to prove Higgens robbed the bank. This is hearsay and not admissible unless there is an exception. 3. Lissner said he heard Oblique say they ought to put Higgens in Jail for this. This amounts to an indirect assertion that Higgens robbed the bank. By any reasonable assertion, Obliquw inteds to assert that point, even though his words dont necessarily say it Answer: All three statements are hearsay. II. NON-VERBAL ASSERTIONS: ASSERTIVE CONDUCT

Evidence Outline

Page 48

Any reasonable definition must embrace assertive conduct when offered to prove the truth of the matter asserted. The most common instance of such conduct involves one of the standard nonverbal cuesnodding or shaking the head or shrugging the shoulders in an answer to a question, pointing as a means of identifying or selecting. Problem 3-Kenworth and Maserati A truck to the side of the Masarati making a turn begins to move forward and the Masarati shoots forward into the intersection, where it crashed into a car. A lawsuit ensues. As proof that he had the light in his favor, D wants to offer evidence of a truck moving forward as being evidence that the light turned green. Advance the Ps argument that evidence of the truck moving forward is hearsay. Answer: On first glance this is clearly not hearsay. It fails by the very definition of hearsay. This is a nonverbal conduct not intending to be an assertion. The truck moving forward was most likely not an assertion saying the light was green. However, you can argue that this conduct implies that the light turned green, suggesting the fact that the jury could reasonably draw this inference from the proof. If you could convince the judge that the trucker was being assertive, then she could call it hearsay. Whatever you can convince a judge was intended by the actor as an assertion is assertive conduct to prove the truth of the matter asserted will be hearsay and excluded (unless there is an exception). The Drag Racing Scenario Two cars (Phillip and Hillary) are stopped at the traffic light, directly in front of you. They appear to look at each other and they both revved their engines loudly. When the light turns green they both "burn rubber" in front of you. 1. Evidential Hypothesis or Fact-pattern: Through their conduct, Phillip and Hillary intended to assert that the traffic light was green (i.e., this was assertive conduct). Alternately, a court can simply offer testimony that they went into intersection as evidence that the light was green (i.e., it is offered as non-assertive conduct). 2. Relevance: Moving into the intersection, after having been stopped at it for a red light, makes it more probable that the light had changed to green in their favor. 3. Probative Value is substantially outweighed by the danger of any of the six (6) reasons stated in FRE 403. FRE 403 is probably not a problem, although the likelihood of unfair prejudice becomes higher if the evidence is admitted as an assertion. 4. Is it Hearsay: FRE 801(a), (b) & (c)? This is only an issue if it is offered as assertive conduct OR if the opposing party objects to its admission by arguing that it is assertive conduct (theoretically possible but quite odd). If it is not offered or objected as assertive conduct, you need not go any further than item 3. 5. Is it a statement, FRE 801(a)(2)? (a) the revving was intended to be assertive of the call/intent to race AND/OR (b) accelerating into the intersection ("burning rubber") was intended to be assertive of the fact that the light was green. Only "b" is admissible for the proposition indicated above, and, if intended as an assertion, it is subject to the hearsay exclusion. 6. Declarants FRE 801(b)?

Evidence Outline

Page 49

Phillip and Hillary 7. Truth of the matter asserted FRE 801(c)? If the proponent is seeking its admission as a statement to prove that the light was green, then yes. The opposing party would argue that any other reason is a mere pretext. 8. FRE 802 The "burning rubber"-as-statement that the light was green is inadmissible hearsay, unless you can find an exception. 9. FRE 803(1) The exception. If it was a statement, it was a contemporaneous statement regarding the condition of the intersection made upon personal observation thereof by Phillip and Hillary. A judge would probably allow the revving evidence to constitute assertive conduct showing Phillip's and Hillary's intent to race, in a criminal trial charging them with racing or injuring someone while racing. The same goes for a tort trial in which they were parties (to show intent to commit a tort requiring specific intent). Thus, the evidence becomes technically subject to the language of FRE 801(a)-(c), but specifically defined as not hearsay by 801(d)(2). (Admission by a party opponent) If we were to classify it as hearsay, and neither Hillary nor Phillip are a party, the best chance of admission is 803(1) present sense impression. That is, Hillary and Phillip intended their conduct to assert that the light was green. They are not parties to the case, so you are describing their conduct in your trial against the person that broad-sided you after you went into the intersection behind Phillip and Hillary. The assertive conduct would be a hearsay statement, but the statement would be admissible because it was their present-sense impression of the situation (i.e., the light changed from red to green). Cain v. George Parents are suing the hotel, alleging their son died of Carbon Minoxide from the AC vent. Owners of the hotel introduce evidence claiming there were never any complaints from customers in the hotel. The Ps contend the trial court erred in allowing in evidence the testimony of the hotel owners. Relevance of Testimony/Evidentiary Hypothesis: People would have complained if there was a problem in the room. People would have been able to smell the carbon monoxide. This EH requires the assumption that the noncomplaints can in fact prove the defect did not exist, because if it had existed then people would have complained. Plaintiff tried to say this is hearsay because the hotel owners implied the assertion of the hotel visitors (i.e.: their silence communicated there was no problem.) Court said this was not hearsay because it derived its value from the credit to be given to the witnesses themselves and was not dependant on the veracity or competency of other persons. This is clearly the best available evidence to support their position that carbon monoxide did not come from the heater. It is admissible to show how the heater acted in the past.

Evidence Outline

Page 50

Indirect Hearsay Consider the following testimony Q: What is your name? A: Edith Harris Q: Where were you born? A: Bangor, Maine Q: Who are your parents? A: Mike and Sue Q: What is your mothers maiden name? A: Smith Q: What is your date of birth? A: November 14, 1983 Edith Harris cannot have firsthand knowledge of any of the facts set out in her answers, and it would appear that every one of them was in substance. These statements are hearsay pure and simplethe source of her knowledge is hearsay, she does not have personal knowledge. In short, the system tolerates testimony of the sort shown above even though the witness lacks personal knowledge and is, in substance, testifying to hearsay. Indirect hearsay, such as this, cannot be used to prove substantial points in a case. Ex: in a paternity case, this would be hearsay. However, in another case where this is irrelevant/background info, this would not be hearsay. United States v. Check Checkkey witness (Cali)Spinelli Check, a police man is convicted of possessing cocaine. Cali, the key witness, was officer Spinellis informant who had a conversation with Check, that pretty much proved his guilt. In the conversation Check offered cocaine to Cali. However, Cali would not testify. Although Spinellie was close by, he was inside the restaurant and did not hear the contents of the conversation. P put Spinelli on the stand to get to what Check had said. They did this creatively by only asking him to say what he (Spinelli) talked to Cali about following the conversation. By not telling us what Cali said to you, what did you say to him? Court said Spinelli was obviously serving as a transparent conduit for the introduction of inadmissible hearsay information obviously supplied by and emanating from the key witness. These statements were being offered for the truth of the matters asserted in them. Government tried to argue they were Spinellis own out-of-court statements that he was testifying to and could be cross-examined on them The testimony presents triple hearsaythe major problem is that Cali is refusing to testify! 1. The statements by Check to Cali: these statements would be admissible if they were offered using Calis testimony (because Cali would

Evidence Outline

Page 51

have personal knowledge of themFRE 602 and they were admissions under FRE 801(d)(2)(a)) However, Spinelli had a lack of personal knowledge of what Check said. It is ultimately the declarants knowledge of what was said. Spinelli does not know what was said between Check and the key witness. The only person who has personal knowledge of what was said during that conversation is Cali. 2. The statements by Cali, as relayed by Spinelli, regarding what Check told him: here the Ps are trying to prove the truth of what Check said to Cali. The P was trying to get Calis description of his conversations with Check into evidence through Spinelli. In effect, Spinelli was becoming a second witness (Cali) to confirm his own testimony about conversations with Check. This is not allowed. They are statements being used to prove the truth of the matter asserted. No exception is available and they are inadmissible under FRE 802. 3. The statements by Spinelli during his conversations with Cali: (i.e.: his statements in the form of I said would be hearsay under FRE 801(a), (b), & (c). No exception is available. He could testify as to his own observations, but not as to his statements at the time he saw Cali and Check talk. Note: The prior consistent statement exception does not apply because that can only be used for rehabilitative purposes, and only if the witness lacked reason to fabricate at the time he made the hearsay statements. The co-conspirator exception (FRE 801(d)(2)(E) does not apply here. Cali, as an informant, was a government agent, and thus was not a true co-conspirator for purposes of exemption. III. NOT
HEARSAYNON-TRUTH

USES

Non-Hearsay: As was mentioned above, an out of court statement is hearsay when offered to prove the truth of the matter asserted. Therefore it is not hearsay when offered for any other purpose. The following are several commonly recognized alternative purposes: (1) impeachment; (2) verbal acts; (3) effect on listener or reader; (4) verbal objects; (5) circumstantial evidence of state of mind; and (6) circumstantial evidence of memory or belief. These categories are not listed in the rules, but are well accepted by the courts throughout the country as nonhearsay uses. With these uses, they do not fit within the definition of FRE 801(a),(b) & (c) most likely within (c) (it fails to fit within the truth of the matter asserted language) You always must balance the possible legitimate value of its nontruth use against the unfair prejudice of its truth use. We are making a very purposeful choice by classifying something as a truth use or non truth use and then are making a judgment as to which one should prevail. Non-Truth Uses (1) impeachment [FRE 607] [FRE 613] [FRE 801(d)(1)(A)] [FRE 803(3)] [FRE 401] [FRE 403] (2) verbal acts (or parts of acts) [FRE 801(a),(b) & (c)]

Evidence Outline

Page 52

(3) (4) (5) (6)

effect on listener or reader verbal objects circumstantial evidence of state of mind, and circumstantial evidence of memory or belief.

1. Impeachment(flip side of impeachment is rehabilitation) A statement that would be hearsay if offered for its truth is not hearsay if used solely to cast doubt on the credibility of a witness current testimony. Prior inconsistent statements are not hearsay when offered to impeach. Rehabilitation is a response to the impeachment. You dont need to rehabilitate a witness unless that witness has been impeached. (FRE 801(d) (1)(a). Problem 3-C: The Blue Car Ran a Red Light Facts: Witness testifies at trial the blue car ran the red light. However, the attorney (on cross) attempts to get the witness to testify that in a previous statement made to an insurance adjuster, he said that the blue car had a green light. The attorney states that he is offering it to impeach the testimony of the witness and not to prove that the blue car had a green light. Issue: Whether the previous statement to the insurance adjuster is admissible or is hearsay. Answer: Admissible to impeach. The statement is being used to cast doubt on the credibility of Bystanders current testimony. Burton offers the statement not to prove what it asserts (the blue car had the light), but to prove that Bystander vacillated. Bystander made a prior inconsistent statement conflicting with his testimony, and the change in his story is contradictory behavior raising doubt about credibility Also, the policy behind the hearsay rule does not seem to be at risk here because the very person who made the out of court statement is available to testify (and be cross-examined) to it. Abbys counter argument: The evidence goes straight to the matter asserted. Abby can argue that Burton's purpose is pretextual, that he really wants the jury to take the prior statement as proof that Burton had the light, hence that it should be excluded under FRE 403 as posing too great a risk of "unfair prejudice." This shows how, with non-truth uses, you can always argue its truth use is too prejudicial. NOTE: If the statement had been offered to prove the truth of the matter asserted, it might fit within the "not hearsay" exception of FRE 801(d)(1)(A) (prior inconsistent statement), BUT ONLY IF GIVEN UNDER OATH IN A HEARING OR AT A DEPOSITION. Since this one was given to the insurance adjuster, it does NOT meet the requirements for the exemption. The net result is that it is only admissible for impeachment purposes under FRE 607, 611, 613, but not to prove the truth of the matter asserted. Steps to take with this problem: Preliminary: The color of the light is crucial to determining negligence.

Evidence Outline

Page 53

Two alternative evidential hypothesis that apply to the out of court statement: 1. Possible truth use (truth of the matter asserted): The light was green for the blue car. 2. Non-truth use: The testimony will impeach the testimony given by Bystander in court. (prior inconsistent statement). Specific Rule (e.g.: 801Hearsay) FRE 801 does not say anything specifically about impeachment. It is important to note the words are hearsay because the words are still a statement (oral assertion) capable of being used to prove the truth of the matter asserted. However, we are not going to use the statements to prove the truth of the matter asserted. We will exclude the statements from the definition of hearsay. Probative Value These statements do have a legitimate probative value for impeachment, and therefore it is a non-truth use for the statement and it should be admissible. We choose to show the inconsistency of the witness is a legitimate probative value for impeachment. We are engaging in a balance. 2. Verbal ActsWords (Spoken or Written) having logical or legal significance independent of their assertive value. Where a persons words have independent logical or legal significance, they can be offered into evidence. This could be the element of an offense or claim (such as an offer or acceptance of a contract, or libel and slander). This is the classic situation where the substantive law gives legal significance to certain words. The statement constitutes the legal act in itself. All we care about is if the Declarant said certain words, because it is a relevant legal act in this case. We dont care about the perception, memory and sincerity of D (the ability to cross examine him). All we want to know is if he said it Ex: A witness testifies I heard the declarant say I accept your offer. This is not hearsay because we just want to know if he said those words, because if he said those words, it constitutes legal significance of acceptance. The same goes for defamation, words of conspiracy (the words constitute the legal act of conspiracy), words of bribery (making offers and seeking acceptances), words of cancellation, words of misrepresentation in a misrepresentation action, words of waiver. Verbal Acts-Element of the Crime Problem 3-DAnyway You Like While at a massage parlor, an undercover cop is propositioned by a masseuse (Debra) who is likely a prostitute. In the middle of the massage Debra asked if the cop was interested in a good timeI can give it to you any way you want. Issue: Whether the prostitute and the undercover cops out of court statements are admissible to show that an act of soliciting prostitution occurred. Answer: Yes. The conversation itself is the issue. (the only way you can show solicitation is through words, so the words spoken would have had to be out of courtthis is a crime that is committed through words.) The act of

Evidence Outline

Page 54

stating the words is the crime of solicitation, and is part of the requirement of the substantive law. It is being offered to show that it was spoken (whether her statement was true is irrelevant, they are not trying to prove that the cop could really have it any way he liked it). Thus, it is admissible. As a matter of substantive law, the words have "independent legal significance," which is to say that the law assigns importance to the fact that they were spoken. They have relevance independent of their assertive aspect. Solicitation for prostitution is the very fact at issue. The truth of the girls statements is not important. The fact that they were made was. The Forest: Relevant: These words are relevant because they are material as to the crime that is committed. Two Alternative Evidentiary Hypothesis: 1. Truth of the matter asserted: She actually gave him sexual favors or she intended to give them sexual favors. 2. Verbal Act: Has legitimitate non-truth probative value. Because it proves an element of the offense, it is given independent legal significance. This is a crime committed through words. This is an offer of sex for money and has legitimate probative matter. Prejudice This statement is still potentially unfair prejudice because the jury could take it to mean she actually gave him sexual favors. Again, we have a balancing. FRE 403 favors admission. Unfair prejudice has to substantially outweigh the probative value. We really need this to prove an element of the crime. Verbal Acts-Element of a Contract Problem 3-E: Whose Corn? Facts: Lord is in a descrepency with Bank and Prager. Both assert they own 40% of Cartwrights corn. As proof, Lord offers his own testimony that he and Cartwright had gone to the field, where Cartwright "pointed out the corn in the double crib and said, 'Mr. Lord, this double crib of corn is your share for this year, and it belongs to you, sir.' " (This statement solidified the contract. ) As proof that the corn was the banks, the bank offers testimony by its loan officer that "when we came out to see about selling the corn, Cartwright told us that the corn in the double crib was his." (This statement occurred after the contract had been made between Lord and Prager.) Issue: First, is Cartwright's statement to Lord hearsay? Second, is Cartwright's statement to the bank officer hearsay? Answer: (1) Lord's Statements are NOT hearsay. (2) Bank Officer's statements ARE hearsay. Reasoning: (1) The statement has its own legal significance beyond its assertive aspects. The statement has legal significance (conveying corn, performing the contract) independent of the assertive aspect, so it is properly viewed as a verbal act. (2) Cartwrights statement to the loan officer did not perform a contract or effect a conveyance to the bank, for there is no indication that Cartwright was conceding default and telling the officer that the bank could claim and sell the collateral. Hence the statement is hearsay.

Evidence Outline

Page 55

The Forest: Relevant: These statements establish who would be the lawful owner of the corn. Alternative Hypothesis: 1. Truth of the matter asserted: The Plaintiff owns the corn 2. Non-Truth use: Delivery of the corn was legally accomplished in the barn when the statement was made. Specific Rules This is a statement (an oral assertion (this corn is your share) and a nonverbal conduct (pointing)) made out of court that can possibly be offered to prove the truth of the matter asserted. So it is hearsay under FRE 801. However, there is a possible non-truth use. Probative Value: The probative value that delivery of the corn was delivered, and a contract was effected is legitimate and therefore passes muster under FRE 403. 3. An out of court statement offered to who the effect on the listener Where a statement (written or spoken) is used to show the effect it had on its intended addressee, it is not hearsay. This often comes up in cases where the reasonableness of someones actions is in controversy, such as loan denials or termination of employment. In the following case, the gas mans statement is offered to show the effect it had on the plaintiff in order to show that his actions were reasonable under the circumstances. Think of it as an out of court statement that shows why a third person acted or didnt act in a certain way. Ex: Victor (the Victim) goes to the police officer and says Spano mugged me. PO arrests Spano. Spano sues the PO for false arrest. The PO wants to show he had probable cause to arrest Spano, therefore he will be allowed to testify to what V said to him because this is relevant to show the effect that it had on the police officer. It gave the PO probable cause to arrest Spano. This statement is not being offered to prove the truth of the matter asserted (it is being offered to show the effect on the police officer) Problem 3-FI am from the Gas Company Facts: Plaintiff smells a gas leak. Agent appears and says that he is from the gas company, and asks Plaintiff to show him the leak. Plaintiff takes Agent around to the gas leak, and Agent lights a cigarette which ignites the gas and injures Plaintiff. At trial, the gas company asserts that Plaintiff is contributory negligent for going so close to a gas leak. Issue: Whether Plaintiff's testimony that Agent stated he was from the gas company is admissible as non-hearsay. Answer: We are not trying to prove the gas man is actually the agent, we are trying to prove the reasonableness of Plaintiffs conduct (the effect on Plaintiff-- that he was not contributory negligent because he believed that he was accompanied by someone who knew what they were doing) We still need to balance the probative worth to ensure the jury does not misuse the statement for proof of agency. (FRE 403)

Evidence Outline

Page 56

Two alternative hypothesis: 1. Truth of the matter asserted: The agent is actually an agent of the Gas Company 2. Non-truth use/Effect on Listener: Alfred was reasonable in approaching the gas leak with someone he believed to be an employee of the Gas Company who was instructing him to go therehis reaction was reasonable upon hearing the statement. Could Forrester take the stand and testify he was an agent in order to prove agency? Yes. Forrester can give persuasive evidence in court that he is employed by the gas company. Although it standing alone; if there is evidence sufficient to show the fact of agency or employment prima facie, declarations or statements of the alleged agent are admissible in corroboration. Note that the gas company could then offer counterproof showing Forest is not an agent. 4. Circumstantial Evidence of State of Mind Where a statement is used to prove what someone thought or how they felt, but not to prove that what they thought or felt was actually accurate, it is admissible. FRE 803(3) creates a hearsay exception for a statement describing a state of mind. In the following example, the wifes statements about her husbands cruelty are being offered to show that she didnt like him, they are not being offered to prove that he was actually cruel. Problem 3-HAnna Soffers Will Facts: Husband tries to collect in a wrongful death suit for future income of dead wife. Only a few weeks earlier, wife executed a will in which she limited his inheritance to $1, and made several disparaging comments about him. Answer: The will and the comment about Ira are not hearsay, when offered to prove how Anna felt and would likely have treated him. As a dispositive document, the will fits the verbal act doctrine (words having legal significance) and it disinherits Ira. The comment about Ira (he is cruel and selfish) is nonhearsay because shows the state of mind of the wife-- showing trouble in the relationship, regardless what kind of person Anna thought Ira was. It is not being offered as proof that the husband was actually bad, but only that the wife and the husband were not getting along and so the husband was not likely to get future income and companionship. Forest Relevance: The will suggests that she deeply resented him, and supports the contention that she would not have shared with him much of her expected "significant income" and would not have been much of a companion (a loving spouse). Arguably, because of this, the damages will be reduced because it shows that she would not have shared much of her income with him or much of a companion. Hearsay:

Evidence Outline

Page 57

1. Will as Dispositive Document: It is right to say that insofar as the will leaves Ira only $1, we have a verbal act with a legal significance (aka that Ira is disinherited and that Anna would not have supported Ira if she had lived.) Truth Use: Ira was a bad person Non Truth Use: Ana would not have supported Ira if she would have lived. Ira would have had no reasonable expectation of future financial benefit if Ana survived. 2. Annas Comments: Anna was highly upset because it amounts to an act of public disclosure revealing a problem in the relationship, and probative worth does not depend on truth content. When one spouse says in a public setting (or one likely to be made public) that her spouse is cruel and selfish, that statement indicates that something has gone sour in the relationship no matter what the declarant actually thinks about her husband's qualities. Truth Use: Ira beat Ana Non Truth Use: The quality of companionship between the two was not what might have been expected between husband and wife-statement raising doubt in the closeness of the relationshiphusband and wife were not getting along so husband would likely not be supported by Ana. FRE 403 Balancing: her disclosure has probative worth simply because she said it; it is behavior raising doubt about the closeness of the relationship, and probative worth is independent of truth content. This content is highly probative of the reasonable expectancy the husband would get if the wife died. The fact that such a declaration was made by the wife, whether true or false, is compelling evidence of her feelings toward him and are not excluded under hearsay, but are admissible as her state of mind and verbal act. 5. Circumstantial Evidence of State of MemoryA persons recollection can be used in conjunction with non-hearsay evidence to connect a person to a place or crime. In the following example, the girls statement does not in itself prove that the place she was assaulted was the defendants, but the fact that her memory matches the description offered by the arresting officer helps to prove she was there. Problem 3-I: A Paper Mache Man Facts: A child is molested by a man in a room. The room is described by the child to a police officer who testifies to what the girl told her about the appearance of the room. The arresting officer then gives a separate detailed account of the defendant's room, and the accounts match. Issue: Whether the description by the police officer of the girls description of the room is hearsay. Answer: No, it may be let in as a then existing mental impression of the room. This is not used to prove that the room was as the girl described it, but rather that the girl had a mental impression of being in the room that could only have come from being in the room.

Evidence Outline

Page 58

The officers testimony as to her own observations does not raise any hearsay problems because the officer is simply testifying to what she observed in the room at the time of the Ds arrest. But this testimony is essential to make the childs testimony relevant. Truth Use: The young girl was assaulted in the Ds room. Non-Truth Use: Because she can describe the room in detail and she was assaulted somewhere, the assault occurred in the room. This is a great example of conditional admissibility: if the actual appearance of the defendant's room can be established by direct testimonial evidence of the police officer, then the young girl's statements to the officers indicating what she believed the room to look like can be used not to prove what the room looked like, but rather to prove that she believed that the room looked thus because she had been there. These statements would NOT have been inadmissible if the purpose had been to establish that there were in fact the stated articles in the room, or that they were located as stated, or that the exterior features or surroundings of the house were as the child stated. However, it was admissible to show she had knowledge as to articles and descriptive features which, as was proved by other evidence, were in fact in or about that room and house. IV. NONHEARSAY
AND

NONASSERTIVE CONDUCT

Rule 801. Definitions (d) Statements which are not hearsay. A statement is not hearsay if-(1) Prior statement by witness. The declarant testifies at the trial or hearing and is subject to cross-examination concerning the statement, and the statement is (A) inconsistent with the declarant's testimony, and was given under oath subject to the penalty of perjury at a trial, hearing, or other proceeding, or in a deposition, or (B) consistent with the declarant's testimony and is offered to rebut an express or implied charge against the declarant of recent fabrication or improper influence or motive, or (C) one of identification of a person made after perceiving the person; or (2)Admission by party-opponent. The statement is offered against a party and is (A) the party's own statement, in either an individual or a representative capacity or (B) a statement of which the party has manifested an adoption or belief in its truth, or (C) a statement by a person authorized by the party to make a statement concerning the subject, or (D) a statement by the party's agent or servant concerning a matter within the scope of the agency or employment, made during the existence of the relationship, or (E) a statement by a coconspirator of a party during the course and in furtherance of the conspiracy. The contents of the statement shall be considered but are not alone sufficient to establish the declarant's authority under subdivision (C), the agency or employment relationship and scope thereof under subdivision (D), or the existence of the conspiracy and the participation therein of the declarant and the party against whom the statement is offered under subdivision (E).

Evidence Outline

Page 59

The nonhearsay uses have nothing to do with the not hearsay uses set up by 801(d)(1). We must distinguish between statements that are nonhearsay because they are used in a way that puts them beyond the reach of the hearsay definition in FRE 801(a)-(c) and statements that are not hearsay because they fit into the special categories in FRE 801(d). For the purposes of this class, nonhearsay exceptions will be classified as follows. o NOT FALLING UDNER FRE 801(a)-(c): nonhearsay or not hearsay o FRE 801(d): hearsay exemptions, "nonhearsay" and "not hearsay" o Other FREs (like FRE 803): hearsay exceptions

FRE 801(d) Not hearsay exceptions Prior statements by witness A witness testifying to the witnesses own prior statement: This is hearsay. A witnesss own prior statement if it is being offered to show the truth is hearsay. Generally, if the declarant testifies, he or she must testify to their observations, not about their prior statements. Ex: The D in a criminal case takes the stand. The Ds lawyer asks his client, when you were arrested what did you tell the police? He said I told them I was innocent. The answer to this is inadmissible hearsay because it is being offered to show the D is innocent. The D could just say I am innocent. However there are certain prior witness statements that are not hearsay and are therefore admissible and subject to cross examination. 1. Prior inconsistent statement which was given under oath and as part of a formal proceeding. (such as a deposition or a hearing)FRE 801(d)(1)(A) 2. Prior consistent statement which is offered to rebut a charge of recent fabrication or improper motive. 3. The Prior statement of identification made by a witness. In this situation most of the hearsay dangers are abrogated by the fact that the witness is now present and subject to cross-examination. Note, the rules adopt a partial approach: 801(d)(1)(A) allows admission to prove the truth of the matter asserted but only if the inconsistent statement was given under oath at a hearing or in a deposition. Nevertheless, FRE 607, 611(b), and 613 make it clear that any statement can be used for impeachment. Admission by party-opponent: A declaration by a party offered against the party. In order to be relevant, usually the admission would have to be inconsistent with the partys present position at trial. These can be in the form of legal conclusions. Ex: I was negligent. The statement is offered against a party and is: The partys own statement

Evidence Outline

Page 60

A statement of which the party has manifested an adoption or belief of its truth A statement by a person authorized by the party to make a statement A statement by the parties agent or servant concerning a matter within the scope of the relationship A statement of a conspirator made during the course and in furtherance of the conspiracy.

United States v. Singer, (1983); pg. 136 Facts: Sazenski and Izquierdo were suspected of smuggling marijuana from Florida to Minnesota. Sazenski had a residence in Minnesota. A letter was sent from Sazenskis landlord to Sazenski and notifying them of their eviction. The prosecution sought to enter the letter into evidence in order to show that Sazenski and Izquierdo lived together. The defense objected on the grounds that the content of the letter was hearsay (writing made out of court by other than the witness). Answer: The court said sending the letter was conduct not intended to be an assertion. In other words, the LLs behavior is an act (LLs conduct of mailing that letter which is being introduced to demonstrate his belief that the D lived there. We can infer from the Landlords act the Defendant lived there. Although this is going straight to the truth of the matter asserted, it is an act, not a statement and therefore does not fit under FRE 801(a). The inferential syllogism starts with an act (not a statement). The court chooses to look at the entire scene and the action of the landlord and therefore it is not a statement and therefore not hearsay. Remember it is all about how we classify this: The court chooses to classify the LLs behavior as an ACT therefore avoiding the problem of classifying it as a written statement. It is the implied truth of the action we are implying from the LLs behavior that D lived there. Counterargument: If this was a written statement submitted to imply the truth of the matter asserted, this is hearsay. Lying and Hearsay: Most courts allow out-of-court lies by a declarant under the somewhat over-simplified reasoning that because they are lies they are not being offered to prove the truth of the matter asserted. A lie cannot possibly be offered for a truth purpose and cannot trigger the definition of hearsay. Instead we are trying to show what the declarant was attempting to achieve through her lies and perhaps what the lies show about her state of mind. Where a defendants lies are offered into evidence, the hearsay doctrine is not implicated because what a party to a suit says may always be used against him. Problem 3-J My Husband is in Denver Facts: FBI agents issue an arrest warrant for Greg. When they get to his house the wife said he was in Denver. The government wants to enter this testimony. D objects claiming that what she said or thought was hearsay. Issue: Whether lies made by a declarant out of court are admissible as non-hearsay if used to show the state of mind of the declarant. Answer: Yes. Here, the out of court statements are not being offered to prove their contents. In fact, just the opposite is true. Her statement appears to

Evidence Outline

Page 61

be a deliberate lie. The government argues the lie is being introduced to show Gregs wifes lied because of her belief in his guilt. If she were trying to lie to police, that would be evidence that she felt he was guilty, which is probative on whether Greg was guilty. Nobody could be convicted on such proof alone, and there may be innocent explanations for what the wife said. Her statement is also a performative act because it is an attempt to interfere with the investigation, which also goes to show Gregs guilt. Counterargument: This is an oral assertion being offered to prove the truth of the matter asserted. The actual assertion (truth of the matter asserted) here is: I believe my husband to be guilty and that is why I am providing him with an alibi. In which case, this would be hearsay. You could also argue admitting this statement produces risks (FRE 403) that unfairly prejudice the jury. Therefore we should not give a lot of probative weight to her statements. Barbara may have misspoken, intending to say only that Greg had been or would be going to Denver (narration) she may have forgotten, wrongly believing he had gone (memory) she may have misunderstood the arrangement (misperception) Significance of Disclosure- Sometimes a defendant might make a disclosure that would seem unlikely of a person with a guilty conscience. This can be offered because of its performative aspects and as circumstantial evidence of state of mind. With these types of questions, you may be able to cross reference FRE 801(d)(1)(B). Problem 3-K: King Air YC-437-CP Facts: Bruno is charged with importation of drugs. Bruno let a drug plane land on his property. At trial, Bruno denies any involvement with the smugglers, claiming that he let the plane land because it was an emergency. He also offers witness testimony saying that she heard him in public say that he was storing a King Air aircraft on his airstrip. The testimony is offered to show that he is innocent because a guilty person would not advertise that a stolen plane was on his airstrip. Issue: Whether a disclosure made out of court by a declarant which would not ordinarily be made by a guilty person is hearsay. Answer: No. The important issue here is did Bruno know there was drugs on the plane. The statement was not offered to prove the airplane was stored on the property. It was offered to support an inference of innocence: a man with a guilty knowledge is not likely to advertise his possession of stolen property. This statement is not offered to prove what Bruno knew, but what he was willing to tell others that he knew. It is being offered to show Brunos state of mind that he didnt know. Nobody intentionally harbors stolen property and talks about it. You are not offering the statement for its truth, but as a state of mind that no one would talk about any stolen property. This conduct by Bruno tends to prove he wasnt knowingly involved in the theft of the plane. It was offered to support an inference of innocence; a man with guilty knowledge is not likely to advertise his possession of stolen property. Pacelli:

Evidence Outline

Page 62

Pacellli was charged with conspiracy to interfere with the constitutional rights of others. In essence, the government was claiming he killed Parks (a witness who was testifying against Pacelli in a drug dealing case). In other words, Pacelli murdered Parks to prevent her from testifying. Lipsky testifies that he was at Paselis house in which there were statements made that Paseli was the one who murdered Parks. Pacelli argues that Lipsky should not have been able to testify as to the conduct and statements of his wife, Beverly, of his uncle, Frank Bassi, and of his friends Perez and Bracer on February 10, 1972, at the Bassis' apartment. The Court agreed (a.k.a.: these statements were hearsay). These statements are verbal assertions by multiple declarants used to prove the truth of the matter asserted. This was an assertion that implied the guilt of Pacelli. Truth of the Matter Asserted: Paseli Killed Parks. Defense knows that the statement is going straight to the matter asserted. However, Defense was trying to say they wanted to admit the evidence to inferentially show guilt through acts. Court said no! Counterargument: this was non-assertive conduct/conduct (actions) from which we can infer Pacellis guilt, in which case it is not hearsay. This is conduct from which we can circumstantially infer Paseli killed Parks. These are acts from which we infer the guilt of the defendant. This is performative conduct intended to assert that they believe defendant to be guilty. Remember, if it is an act, it is non-assertive conduct, thus NOT HEARSAY under 801(a)(2). Therefore, we can use it to prove any inference we want. But the Pacelli court did not buy that prosecutorial argument. Also, if we can prove that they committed a crime in engaging in the cover-up, then they clearly become Verbal Acts and would be admissible, just as in the "My husband is in Denver" problem. (The wife was committing a crime independent of everythinglying to the Feds) Accordingly, they would be admissible to prove something other than the truth of the matter asserted for 801(c) purposes. Pacelli Under the Rules: (1) FRE 401: It is more likely Paceill committed conspiracy if he killed a key witness that was going to testify against him. (i.e.: Pacelli is guilty-- the speakers assumed him to be guilty because he told them he was guilty.) (2) FRE 801(a): The statements and assertions were intended to be statements, thus the former fit under 801(a)(1) and the latter under 801(a)(2). (4) FRE 801(b): The statements were made by persons. (5) FRE 801(c): (what the Plaintiffs would argue) The statements are NOT being offered to prove the truth of the matter asserted, (a) because they do not state that Pacelli is guilty, they infer that he is guilty and the rule does not adopt the inference view; or (b) because they are verbal acts constituting obstruction (illegal act, ex: cover-up) (6) Since they are not hearsay, 803(3) is not needed, but note that for the same reason they probably do not fit within the 803(3) exception. If you find the statements are hearsay:

Evidence Outline

Page 63

You can argue that the statements are offered to prove mental impressions based on knowledge acquired from Pacelli regarding the crime (803(3)), but they would not be admissible to prove the truth of the matter asserted. However, when you couple them with the co-defendant's confession that he helped Pacelli commit the crime, they might become admissible, just as the child's statements in the Papier Mache Man case became admissible once the other police officer testified to her personal knowledge of what the appartment looked like.

Betts v. Betts There is a child custody proceeding going on. The foster mother saw an item in the paper relative to the remarriage of the child's mother and with reference to it, testified as follows: A. So I told her that her mama and Mr. Ray Caporale had got married, and she started crying. She said: --she ran and put her arms around me and her head in my lap and started crying real bad and hard and said, "He killed my brother and he'll kill my mommy too,"-- and she doesn't seem to ever get that out of her mind. The use of this testimony does not violate the hearsay evidence rule. Court deems this evidence to be circumstantial assertions of fear on behalf of the child. We are offering this statement to show the childs state of mind. (nontruth use) This testimony offers circumstantial evidence of the childs fear of her step father. (Be careful not to say the child knows or believes, because this may implicate the truth of the matter asserted.) Reasoning: The statements of the child were not admitted to prove the truth of the assertions she made, but merely to indirectly and inferentially show the mental state of the child at the time of the child custody proceedings. o In this case, if we were offering this evidence to prove the father actually did kill the childs brother, then it would definitely be hearsay, as it would go straight to the truth of the matter asserted. o However, here we are only offering the evidence to show the child would not be in a good environment (evidentiary hypothesis for the non-truth use) o In this case, verbal communication is being used for a nonverbal assertion (fear) Possible truth uses, the court could have found: (in which case, the statements would have been hearsay) 1. The Step-dad killed my brother. 2. The Step-brother is a bad guy. FRE 403: We allow this because the dangers or the truth use is NOT IMPORTANT in this case. I am Napolean Bonaparte

Evidence Outline

Page 64

This is an obvious example of an out-of-court non-hearsay statement which circumstantially indicates a state of mind regardless of the truth of the statement. This would be relevant in a sanity hearing. A Note on Reliability The Reliability that we deal with in non-truth uses is the reliability/strength of the evidentiary hypothesis. We do not deal in terms of the reliability of the statement. Reliability of the statement/witness is only important when we are dealing with the truth of the matter asserted. (i.e.: for truth purposes.) We will deal with reliability with the hearsay exceptions and the 801(d) Inferences verses Implications/Assumptions Inferences Betts deals with inferences (i.e.: circumstantially inferring the state of mind of the child) Implications/Assumptions Pacelli deals with that which is implicit/asserted by a statement.

A Note on Possibility of Truth Use: In civil cases, the possibility of truth uses is a lot less. Where in criminal cases, as in Pacelli, the possibility of truth use (i.e.: that Pacelli killed the victims) could be used to impact the jury (i.e.: that Pacelli was guilty.) In Bettes, the fact that the step dad killed the son was not as harmful because it wasnt his criminal trial! Pacelli was a criminal cases that implicated Pacellis guilt. Bettes was a civil case that dealt with circumstantial inferences. Hearsay Quiz: (page 150) We are only determining whether it is or is not hearsay (FRE 801(a)-FRE801(c). Elements: 1. Non-hearsay: circumstantial evidence of a state of mind. If you identify yourself as someone you cannot be, and your capacity is being challenged, this is circumstantial evidence of a state of mind. Because of the Duck Soup Argument, you do not have to go to 803(3). If this was an identity theft case, this could be labeled as a verbal act because this would be an element of the fraud. 2. Non-hearsay: possible non-truth useeffect on listener (attempting to show C was negligent by getting in the car) 3. Hearsay: this goes straight to the truth of the matter asserted! This is different than the gas company, because the testimony in the gas company example was being offered to prove something else. In the gas case, there was independent legal significance. The man was reasonable in following the agent to the gas tank. 4. Non-Hearsay: Verbal Act (if we knew that lying to the police was a crime) or circumstantial actions of the state of mind. Her lie circumstantially proves her belief that G was guilty. Cross reference Ana Sofers will.

Evidence Outline

Page 65

5. Non-hearsay: non assertive conduct-it was an unavoidable natural response and therefore we should trust it. We may be able to argue it was assertive, if we can argue, the sweating and shaking was fake. IV. HEARSAY EXCEPTIONS Here, we know the evidence is hearsay. We are now more concerned with deciding whether the statement is admissible, even though it is hearsay. There are 37 express exceptions allowing the use of hearsay for truth purposes within the rules, not to mention the six "non-truth use" categories. With hearsay exceptions, you still need to consider: o the FRE 403 balancing test and o the Sixth Amendment Confrontation clause. At the heart of the confrontation clause is the right to cross-examine, which is diminished or denied when hearsay is used. Deconstructing Hearsay: Preliminarily: who decides admissibility/foundation, judge (FRE 104(a)) or jury (104(b))? (1) Is the offered evidence relevant? [FRE 401] [FRE 402] (2) Is the offered evidence hearsay? [FRE 801, et seq.] [Presumptively Excludable, FRE 802] (a) Does the evidence fit within the definition of hearsay of FRE 801(a), (b)&(c)? (Remember the non-truth uses) (b) Even though it fits the 801(a),(b),(c) definition of hearsay, is it nevertheless within some exemption that expressly defines it as "nothearsay" or "nonhearsay." (FRE 801(d))? (c) Even though it fits the 801(a),(b),(c) definition of hearsay, AND despite it failing to be exempted by 801(d), is it nevertheless within some exception found in the rules, especially in FRE 803 and FRE 804? (Hearsay Exceptions) (3) Does the 6th amendment prohibit its use? Should the D be able to cross examine the witnesses against him? (4) Should the offered evidence be excluded, despite being relevant, and regardless of the answer to the hearsay question? [FRE 403] (5) Should there be a limiting instruction? Hearsay Exceptions Come in FOUR main groups: 1. Statements by declarants who testify: Three exceptions that apply to certain prior statements by testifying witnesses. Rule 801(d)(1). These statements are called "not hearsay" even though they fit the basic hearsay definition set out in Rule 801(a) through (c). In substance, these are hearsay exceptions created by statutory magic. 2. Admissions: The second group contains five exceptions that together make up the "admissions doctrine." These are party admissions. These five exceptions are set out in Rule 801(d)(2). 3. Unrestricted Exceptions: The third, and by far the largest group, is comprised of the 24 [actually 23] 'unrestricted' exceptions listed in Rule

Evidence Outline

Page 66

803. Statements that fit these exceptions may be offered to prove what they assert regardless whether the declarant testifies, and regardless whether or not he could be produced at trial to give testimony. Declaration of Existing State of Mind when existing state of mind of declarant is an issue in the case. Ex: Did Spano act with malice on January 2? A witness can testify that he heard Spano say I feel malicious today. Declaration of Intent: declaration of present intent to do something in the not too distant future is admissible to show what was intended was done. Ex: police find declarants dead body. The witness said he heard the decalrant say I want to kill myself a few days earlier. Excited Utterance: (1) there must be a startling event; (2) the statement must be made under the stress of excitement under the startling event; (3) statement must concern the effects of the starling event o Startling event could be a fire, bank robbery, etc. There can be a time lapse between the event and the utterance. The issue becomes has the excitement worn off? o Look at the verbs to determine if it is an excited inferencethe declarant screamed, showed, etc. Present sense impression-precise contemporaneousness. Mom calls up V. V tells Mom someone is at the door. 20 seconds later V comes back on the phone says Spano is here. Shorltly thereefater, V is found dead. Mom wants to testify Spano is here. This is present sense impressionv is describing something that is in existence at the very instance he is speaking. A declaration of present physical postion: A declarants exclamation of present pain, suffering, bodily condition etc. is admissible. A declaration of past pain, suffering physical condition: medical records made for the purpose of diagnosis or treatment is admissible. (You arent going to lie to a Dr. who needs to give you a diagnosis or treatment.) Ex: a dr. is hired to give testimony that the P was disabled as a result of the accident. He basis this on the medical history he got from the P. The previous medical history is admitted for its truth. 4. Statements by unavailable declarants: The fourth and final group is comprised of five more exceptions set out in FRE 804(b), but these may be invoked only if the declarant is ''unavailable as a witness" under FRE 804(a). Former Testimony by an unavailable declarant The Statement Against Interest a declaration of a person now unavailable as a witness against that persons money interest or property or penal interest. A statement which is offered to help the accused is not admissible without cooboration of the trustworthiness of the out of court statements against penal interests. Ex: a third party confessed to the crime of which someone is on trial for. This only comes in if there is corboration of the trustworthiness of the

Evidence Outline

Page 67

statement. (Dont confuse this with the admission of a party) It is different because (1) the statement of interest must be against interest at the time it is made; (2) it can be by anyoneit doesnt have to be a party; (3) personal knowledge is requiredthe declarant must know what he is talking about and know what he is saying is against his interest; (4) the person who made the statement must be unavailable. The Dying Declaration: the statement made under a sense of impending death. Rationale: a person will not lie at the dying hour of death. There is a psychological compulsion not to die with a lie on your lips. Four things to note: (1) the statement must be made under a sense of impending deathvictim must know they are going to die; (2) the declarant need not actually die, they can survive, but they must be unavailable at the time of trial; (3) available only in homicide case or in any civil case; (4) the subject matter must concern the cause or circumstances of the impending death.

FRE 807: (Residual Exception gives another possibility for admissibility with hearsay)-- In extraordinary circumstances where you have an out of court statement that does not fit in a categorical exceptions, if it has equivalent ground of trustworthiness with one of the exceptions the judge can use its discretion to admit it. Sixth Amendment and Testimonial Evidence In hearsay questions, you must always consider the Ds right to confront the witness. The accused has a 6th amendment right in a criminal case to confront the witnesses against him. Even if the out of court statement fits a hearsay exceptioneven if the hearsay is normally admissible, it should still be excluded by the 6th amendment if: (1) the statement is offered against the accused in a criminal case (2) there was no opportunity to cross examine when the statement was made and (3) there is no opportunity to cross examine at the time of trial; and (4) the statement was testimonial in nature (a statement which is made to police officers for use in the investigation or prosecution of a crime) In Crawford v. Washington and Davis v. Washington, the S.C. changed the understanding of the 6th Amendment. For a long time, if one of the hearsay exceptions applied, you met the Ds 6th amendment right. However, the recent S.C. has said there is a distinction between what the rules accept and allow to be admitted and what the 6th amendment accepts and allows to be admitted. The 6th amendment does protects a Ds right to confront testimonial statementsstatements made to police that are gathered for the purpose of prosecuting someone. o Not every statement to law enforcement is testimonial, however. Sometimes the police officer is merely trying to provide for a persons health and safety and is not gathering evidence for prosecutorial concerns.

Evidence Outline

Page 68

o Subjective intent of the governmentas long as the police are trying to help you and prevent the health and safety of the general public, this is not testimonial. This is trying to get control for an emergency situation. However when they are gathering information to prosecute criminals, this is testimonial. A. Prior Inconsistent StatementFRE 801(d)(1)(A) A statement is not hearsay if-(1) Prior statement by witness. The declarant testifies at the trial or hearing and is subject to cross-examination concerning the statement, and the statement is (A) inconsistent with the declarant's testimony, and was given under oath subject to the penalty of perjury at a trial, hearing, or other proceeding, or in a deposition, or (B) consistent with the declarant's testimony and is offered to rebut an express or implied charge against the declarant of recent fabrication or improper influence or motive, or (C) one of identification of a person made after perceiving the person; or State v. Smith Assault victim, Rachael Conlin, wrote out a statement on a form supplied by a detective of the Pasco Police Department, which contained Miranda warnings, in which she named Nova Smith (defendant) as her assailant. She signed under oath with penalty of perjury before a notary. At Smith's trial a month later, she named another man as her attacker. The prosecuting attorney introduced the written statement at issue for impeachment purposes. (FRE 607, 613) The State then moved to have it admitted as substantive evidence also, as it was apparently the only evidence that identified defendant as the perpetrator of the assault. [FRE 801(d)(1)(A)]. They want to use this for a proof purposethey want the jury to believe the content of the statementthat Nova assaulted her. NOTE: if the P would have just used this for a non-truth purpose (impeachment) then you cannot prove the substantive matter of the statement. The witnesses testimony would simply have been thrown out. In that case there is no evidence to prosecute Nova!) The trial court allowed her prior inconsistent statement to be used as substantive evidence ruling it was not hearsay under Rule of Evidence 801(d) (1)(i). The jury found Smith guilty of assault in the second degree. Note that the brutal nature of the assault, combined with the subsequent attack by the same person, creates a strong inference that the witness has changed her version of the events due to fear, rather than because what she is now saying is the truth. Analysis: The statement meets all the requirements of FRE 801(d)(1)(A) except the "prior proceeding" which is the object of the Court's analysis. While every statement given in a police station is not a proceeding the court said all of the factors in this case would suggest this is a proceeding.

Evidence Outline

Page 69

This decision stretches the term "proceeding" beyond recognizable bounds. o Clearly, nothing here is a hearing, grand jury proceeding, deposition or trial. All of these things entail adversarialness (lawyers from both parties) and often a neutral undetached magistrate or factfinder/jurrors who preside over the proceeding. o Giving a notarized statement to the police is questionable as to whether it constitutes a proceedingbut the court said a proceeding must establish that the statement is sufficiently reliable to constitute other proceeding. o Prevailing standard: Some police interrogations may fit in this other proceeding language. Some federal courts would exclude all affidavits made to investigating officials as substantive evidence under FRE 801(d)(1)(A)this is the prevailing standard. Florida also uses this standardyou have to look for adversarialness or the presence of a neutral and detached fact finder in order to get it admitted. o Exceptional state rule: (which Smith uses) you can admit an affidavit sighed by a witness any time it is taken under oath before an official who is authorized to hear evidence and administer oathsthis is the exception to the rule! In determining whether to consider something a proceeding and therefore admitted as substantive evidence under 801(d), the reliability of the statement is the key. o The court says there is no bright line test for reliability. You have to do a balance--look at reliability under a totality of the circumstances test. o Here, there was no question that the statement was made since Ms. Conlin testified to that fact. o The inconsistent statement is more likely to be true than the testimony at trial as it was made nearer in time to the matter to which it relates and is less likely to be influenced by factors such as fear or forgetfulness. o Minimal guaranties of truthfulness were met since the statement was attested to before a notary, under oath and subject to penalty for perjury. Additionally, the witness wrote the statement in her own words. o The jury, seeing Rachael Conlin on the stand, under oath, and hearing her explanation of the inconsistent statement while subject to cross examination, was in a position to determine which statement was true. Each case depends on its facts with reliability as the key. Here, the complaining witness-victim voluntarily wrote the statement herself, swore to it under oath with penalty of perjury before a notary, admitted at trial she had made the statement and gave an inconsistent statement at trial where she was subject to cross examination. FRR 801(d)(1)(i) is satisfied under the totality of these circumstances. Note, FRE 804, would have been a better way to getting this prior statement admitted. In the federal system it would be the ONLY way to

Evidence Outline

Page 70

get something like this admitted. Because this balancing test of reliability would not have passed muster in federal court! Furthermore, something like this will not pass Crawford and would be exempt under the Sixth Amendment.

Problem 4-A: "I Got Amnesia" In June 2007, Paul Barlow is tried on charges of racketeering and disrupting interstate commerce, arising out of an armed robbery of Hilshire Farms. At a grand jury proceeding, Peter testified that he and Paul and Zigler cased Hilshire farms a few days prior to the robbery. The government calls Peter as its star witness, but he suddenly proves unhelpful. He says he cannot remember making the statements. He claims he had amnesia and that he was the victim of federal police abuse. The trial court admits the grand jury testimony. Paul is convicted and he appeals. Did the court properly admit the grand jury testimony? The prosecution wants to classify these statements as being inconsistent. The defense makes two objections: 1. Lack of memory (even feigned) is not the standard form of inconsistency However, courts have held that inconsistent does not require actual conflict between present testimony and an earlier statement. It is enough that one contains a point of detail absent from the other-- any change between what was said before and what is said now that might bear on the case in some way seems inconsistency enough. Does "inconsistent" reach only differences that seem the product of deliberate change of heart by the witness, or also differences resulting from non-volitional changes of opinion, perhaps brought on by changing recollection? o If inconsistent refers to deliberate change of heart, then genuine memory lapse does not pave the way for a prior positive statement, though a lying claim of lack of memory does. o In other words, the rule is not as concerned with genuine memory lapses as it is with a lying witness. We only want to resort to using prior statements when we are trying to ward off the most serious evils, such as lying. o Here Pauls claim of lack of memory seems hard to credit, which makes it easier to conclude that the present stance of the witness is inconsistent with his earlier stance. o The real case: A professed memory lapse . . . must often be considered as inconsistent testimony," or FRE 801(d)(1)(A) would fail in its purpose to protect against the turncoat witness. 2. The un-remembering witness is hard to test by cross-questions. This argument is more substantial than the first. The main justification for FRE 801(d)(1)(A) is that the speaker is now subject to cross about what he said before. FRE 801(d)(1)(A) requires the witness to be cross-examinable about his prior statement, not necessarily to be cross-examinable about the events reported. Arguably the Rule is satisfied if Paul simply remembers going to the grand jury, not necessarily what he told the grand jury.

Evidence Outline

Page 71

However, Pauls lack of memory (or refusal to answer) is so complete that he was not really cross-examinable. However, the court says the statements are reliability enough even though he technically is not being cross, examined, the jury has all the elements it needs because this entire bizarre scene has played out in front of them and the jury can determine if he is lying This is sufficient for reliability because a jury can evaluate which statement to believe. Therefore even though he is not able to answer questions, the jury in observing this scene has all the elements to decide what it needs to decide whether he is lying about not remembering his prior testimony. We do not know how this would fair under the 6th amendment analysis. In many cases in which the witness suffers a total memory loss concerning both the prior statement and its contents, the witness cannot be considered subject to cross examination. Now, we only evaluate reliability under the rules. Back in the day, you combined the reliability analysis under both. In other words, if it was reliable under the rules, it was reliable under the Sixth Amendment. Today, you not only need to pass the reliability standard under the rules, but also need to pass the standard under the Confrontation Clause. The SC has yet to tell us what this standard is. Any reliability flexibility is suspect as to the Sixth Amendment requirements because of the SCs conservative approach to the Sixth Amendment. Reliability is now not enough to satisfy our constitutional concerns. We need the SC to tell us what they mean on this aspect of the Sixth Amendment. If you take a strict view of the rule, this should not cut it. It is highly likely the SC will need to re-evaluate this, what it means to be cross-examinable. The real case: Lack of memory about "the subject matter" of the statement (robbery of Halsted Foods) "does not alone render him not subject to crossexamination." But the fact that Paul "claimed no memory of giving the testimony" to the grand jury and no memory of "circumstances surrounding" that testimony could pose a problem.

General Credibility Attack The only headway Barlow makes is that Breen answers questions relating to general credibility: He admits he's a lifelong burglar; he's out of the racket and dependent on the government; he's still in trouble with the law; he's done soft drugs (valium) in the past while in a correctional facility. All this goes to general credibility, and presumably suggests that his prior statement can be discounted in some measure. The Confrontation Clause and the Ability to Cross Examine In Owens, the Supreme court rejected a constitutional confrontation challenge. The Supreme Court places a great deal of trust on (a) the jury's ability to see through the claims of memory loss and (b) the effect of the questions during crossexamination. Now the issue of construing FRE 801(d)(1): The more natural reading of "subject to cross-examination concerning the statement" includes: o Ordinarily a witness is regarded as "subject to cross-examination" when he is placed on the stand, under oath, and responds willingly to questions.

Evidence Outline

Page 72

The witness may undermine the process to such a degree that meaningful cross-examination within the intent of the rule no longer exists. But that effect is not produced by the witness's assertion of memory loss -- which . . . is often the very result sought to be produced by cross-examination, and can be effective in destroying the force of the prior statement.

B. Prior Consistent Statement (FRE 801(d)(1)(B) Tome v. U.S. At issue is the interpretation of [FRE 801(d)(1)(B)] bearing upon the admissibility of statement, made by a declarant who testifies as a witness, that are consistent with the testimony and are offered to rebut a charge of a "recent fabrication or improper influence or motive." The question is whether out-of-court consistent statements made after the alleged fabrication, or after the alleged improper influence or motive arose, are admissible under the Rule. Here there is a (1) simple non-truth purpose of rehabilitation: consistency of her statements which bolster her credibility and counters the argument that she is fabricating and (2) a substantive usethe jury should believe the content of the statements (that the Dad beat the girl) Impeachment by charging that the testimony is a recent fabrication or results from an improper influence or motive is, as a general matter, capable of direct and forceful refutation through introduction of out-of-court consistent statements that predate the alleged fabrication, influence or motive. o A consistent statement that predates the motive is a square rebuttal of the charge that the testimony was contrived as a consequence of that motive. By contrast, prior consistent statements carry little rebuttal force when most other types of impeachment are involved. o This has a high probative value and is highly reliable and therefore we should allow it to be admitted under the exception. Common Law rule: The rebuttal is confined to those statements made before the fabrication or improper influence or motive arose. But this is considered an element under 801(d)(2)(B). o You have to look at when the motivation to lie arises. If the statements were made after her motivation to lie arises, they are useless as to whether she is fabricating. Therefore, the statements must predate the motive. These statements passed muster under the rule because they met the express requirements of the rulethe statements were made prior to the formulation of a motive and were consistent with her testimony in court. They were offered to rebut an express charge against the declarant of recent fabrication. The court can still exclude the admittance under FRE 403 (which it did). It said the evidence was too unfairly prejudicial. The fact that a jury will use the statements for their substantive value is very unfairly prejudicial. This is the way out for courts.

Evidence Outline

Page 73

Two ways to interpret Tome: 1. A consistent statement must satisfy the premotive requirement if it is offered to refute a claim of improper influence or fabrication, and then it may be used to both rehabilitate and as substantive evidence under FRE 801(2)(1) (B). 2. A consistent statement must satisfy the premotive requirement if offered to refute a claim of influence or fabrication, but only if it is to be used as substantive evidence too. Note: Tome (and the rule) is limited to using prior consistent statements to refute claims of improper motive or recent fabrication. Therefore you cannot use a prior consistent statement to refute claims of lack of memory or on attacks of character and veracity, or biasness. You cannot use it for the general purpose of repairing credibility! C. Admission by party-opponent (FRE 801(d)(2)(A) The statement made by a party being offered against that party by their opponent to prove the truth of the matter asserted. FRE 801(d)(2)(A) The statement is offered against a party and is the party's own statement in either an individual or a representative capacity or a statement of which the party has manifested an adoption or belief in its truth, or a statement by a person authorized by the party to make a statement concerning the subject, or a statement by the party's agent or servant concerning a matter within the scope of the agency or employment, made during the existence of the relationship, or a statement by a coconspirator of a party during the course and in furtherance of the conspiracy. Problem 4-B: Fire in the Warehouse Martin left his truck at Carters auto repair. While it was in the shop, a fire consumed the premises and destroyed Martins truck. Martin sues Carter. As proof that Carter is negligent (because his employee Dugan caused the fire), Martin calls Esher, the insurance adjuster. Carter spoke to Esher after the fire and told him The fire started in the shed when Dugan put a flaming welding torch on the ground too close to the fumes. Carter raises a hearsay objection and Martin invokes the admissions doctrine. Should Carters statement come in? Forest: 1. Relevance: Carter as owner of the automotive shop makes a damaging statement to his insurance adjuster Esher, saying that the fire started in the paint shed when Dugan put a flaming torch on the ground too close to the fumes. Thus Carter acknowledges both cause and negligence. 2. This is hearsay because it is an out of court statement made by Carter (the declarant) to prove the truth of the matter asserted. (i.e.: that Martin caused

Evidence Outline

Page 74

the fire which destroyed the truck) Because it is hearsay, you need an exception. 3. This is an admission by a party opponent because it is the partys own statement being offered against him by the Plaintiff. The party who made the statement does have the right to be able to explain the statement. 4. The facts suggest four arguments for exclusion (besides it being hearsay), and they should all be rejected (separately and together). The Speaker Lacks Personal Knowledge: [FRE 602] Problem is, Carter has no personal knowledgehe didnt see it happen. However, the ACN rejects this objection: It endorses common law tradition, noting that admissions are not subject to "the rule requiring firsthand knowledge. This has been common law tradition foreveranything that comes out of the mouth of a party and is relevant to the case may be offered against them at trial for substantive legal process. No guaranty of trustworthiness is required in an admission. The Statement was self-serving: (he wanted to collect insurance proceeds). This argument won't fly: a "self-serving" statement fits the exception (the speaker cannot himself exclude what he said on ground that when he spoke he was trying to further his own interests). You cant have it both wayswe dont care why you made the statement. The statement gives an opinion/is too conclusory ("too close to the fumes"): Admissions are free of "the restrictive influences of the opinion rule." This is lay opinion and generally lay opinion is not allowed to be conclusory. However, because of the nature of the admissions doctrine, it does not matter if an admission is opinion. The speaker did not anticipate use of the statement against him: on this point, FRE 801(d)(2)(A) is silent, and here is a place where legislative silence is significant. This concern simply doesn't count. A failure to anticipate the use to which a statement might later be put affects our sense of fairness, but on facts such as these it is hard to imagine that the declarant was oblivious to the possibility of being sued. You have the opportunity to explain yourself regarding anything relevant to the case you may have said. What the court actually did: It held that the statement by the defendant was admissible, and rejected his argument that it should have been excluded because he lacked personal knowledge. Personal knowledge is not required, and that statements "against interest" are admissible against a party when "inconsistent" with his position at trial (the apparent assumption being that the statement here was against interest because it hurt declarant's position at trial). Cross Reference FRE 804(b)(3)Statement Against Interest (however this is only available when the declarant is unavailable)

Evidence Outline

Page 75

Problem 4-C: An Encounter Gone Bad While on a vacation in Co, a wealthy man named Ken had a convo in a bar with a waitress named Sally. They eventually went to Kens room and two days later Sally told police Ken sexually assaulted her. If Ken pleas guilty to the sexual assault charges will this be a party admission in a civil suit resulting out of the same manner for damages? A plea of guilty to criminal charges is an admission by the defendant that fits FRE 801(d)(2)(A) to the extent that defendant personally enters the plea or FRE 801(d)(2)(C) if the plea is entered on the defendants behalf by his attorney. On the question whether such a plea and the accompanying statements should be admitted in a later civil suit, there are three serious issues and one nonissue. This is potentially an admission that can be used in the civil case, and you must be concerned about this as defense counsel. Here are your concerns: 1. Plea: Sufficient overlap? Does the Ds act fit the language of the torts statute? Here you need to look at the statutory language. Do the elements of the criminal offense to which he is pleading guilty and the elements of the tort crime in the civil case overlap? You need to see what statute he pleaded guilty to and compare the substantive language of the tort statute. Many times, the tort statute is broad enough to include the acts to which the D pleaded guilty under the criminal statutes, the admission in entering the plea of guilty is admissible in the civil case. Two pleas that are not admissible under 801(d)(2)(a) as admissions: No lo contender pleas: I admit no liability. It does not constitute an admission. Alfred pleas: the D says I didnt do it, but because the government can prove me guilty I will enter a plea of guilty so that I dont get the worse penalty. However variations of the Alfred plea (where the D is offering another explanation) should be treated as an admission under 801(2)(2)(a), however you can still argue that it should be thrown out at the FRE 403 stage and if that doesnt work, you can argue that it is too weak to be applied in summary judgment and it must go to the jury. 2. Plea: Undermined by statements? A defendant who pleads guilty must understand what he is doing. The judge will likely ask the defendant if he understands that pleading guilty paves the way for conviction and waives his right to a trial by jury and will lead to sentencing, and the judge will likely ask questions directing defendants attention to the occasion of the charged crime and will ask whether defendant engaged in sexual touching of the complaining witness without her consent. Assuming that the defendant acquiesces on all these points, his plea of guilty is supported and will likely be admissible in the later civil suit. The question of admitting or excluding the plea raises issues similar to those in Problem 2-E (The Exploding Gas Tank), in which the concern is over confusing the issues or misleading the jury, which are grounds of exclusion under FRE 403. Cross Reference 609 (and the Liscomb case) 3. Plea: Failing to advise? In the setting of a well-healed defendant like Kenneth Brixton, who is represented from the beginning by a lawyer that he hires, it is inconceivable that a court would worry about the

Evidence Outline

Page 76

defendants awareness of the possible risks of using the criminal plea in the civil case. The lawyer must be concerned about his possible malpractice for failing so to advice the client about the implications of how this plea may be used in a civil case. The court is not required to warn a D that their plea may be used against them in a civil case. 4. Are the statements in court excludable as plea bargaining? The answer is no, at least if the judge accepts the plea and it sticks. FRE 410 only protects a FAILED plea bargain. Therefore if a guilty plea is entered and accepted by the judge, it is no longer protected by the FRE 410 privilege. It is fully admissible under FRE801(d)(2)(a). Note: In traffic cases, guilty pleas are generally not admissible. They are not considered an admission of fault. Confession by a co-defendant: Confessions are unique in their devastating impact; If a co-defendant admission is admitted into evidence, there may not be harmful error if there is overwhelming independent evidence of guilt Remember, if one partys admission may be inadmissible because of the unfair prejudice to the other party, the prosecutor has the option of trying codefendants separately Bruton v. U.S. George Bruton and William Evans were on trial for armed robbery. A postal inspector testified that Evans made an oral confession saying in effect that Bruton committed the robbery. The trial court admitted the confession against Evans but said that it could not be admitted against Bruton. The court said that limiting instructions suffice to protect one defendant from anothers confession. General Rule: Jurors are presumed to be able and willing to follow their instructions. However, both Bruton and Evans were convicted. The appeals court reversed: There are some contexts in which the risk that the jury will not, or cannot , follow instructions is so great, and the consequences of failure so vital to the defendant that the practical and human limitations of the jury system cannot be ignored. Here, the powerfully incriminating extrajudicial statements of a codefendant, who stands accused side-by-side with the defendant are deliberately spread before the jury in a joint trial. Not only are the incriminations devastating to the defendant, but their credibility is inevitably suspect. It is very important in explaining that Evans' confession was not admissible against Bruton under any other exception. The authors explain specifically that the co-conspirator exception (FRE 801(d)(2) (E)) does not apply. o The statement was made after the arrest. A statement made after the D is arrested is not in furtherance of the conspiracy and therefore it is not admissible as a co-conspirator admission under 801(d)(2)(e) New Vision About Limiting Instructions "The fact of the matter is that too often such admonition against misuse is

Evidence Outline

Page 77

intrinsically ineffective in that the effect of such a nonadmissible declaration cannot be wiped from the brains of the jurors. The admonition therefore becomes a futile collocation of words and fails of its purpose as a legal protection to defendants against whom such a declaration should not tell." "The government should not have the windfall of having the jury be influenced by evidence against a defendant which, as a matter of law, they should not consider but which they cannot put out of their minds." This case is limited to this particular contextwhere a co-defendant admits on behalf of the other defendant. The general presumption is still that juries are able and willing to follow instructions. When annotating FRE 105must put the general rule found in Delli Paoli! Rule: A co-defendant admission may not be admitted in a trial. It is just way too prejudicial, and the limiting instruction does not protect against the 6th amendment rights of the defendant. The gender rule does not apply to co-conspirator statements. In co-defendant criminal trials you either sever the Ds or dont use the admission. Sixth Amendment Confrontation right: We still adhere to the rule that an accused is entitled to confrontation of the witnesses against him and the right to cross-examine them. Therefore, if the co-defendant does not get on the stand, than the 6th amendment rights of the non-confessing co-defendant is implicated. Are we destroying the Jury system? Are we assuming that jurors are incapable of following ANY instruction? As noted above, there are some contexts (ex: a co-defendant) in which the risk that the jury will not, or cannot, follow instructions is so great, and the consequences of failure so vital to the defendant, that the practical and human limitations of the jury system cannot be ignored. Possible Alternatives in Bruton: (1) Try Evans and Bruton separately (FRCP 14 authorizes a severance where it appears that a defendant might be prejudiced by a joint trial.) (2) Redact the confession: possible but highly unlikely (3) Hope the confessing defendant X testifies and is subject to cross-examination by codefendant Y (then Y's confrontation (6th amendment) rights are satisfied); (4) Impanel two juries (both to hear at the same time the evidence admissible against both defendants; each jury would separately hear the evidence admissible against only the defendant that it tries). Problem 4-D: His Master's Car Napton works for Ace Building supplies. While working one day, Napton negligently runs over OBrien. A month later Ace fires Napton for reasons unrelated to the accident. Six months later Napton tells OBrien the brakes on the truck failed and I was speeding at the time of the accident. OBrien sues Ace and Napton for personal injuries. At trial, OBrien offers Naptons statement invoking the admissions doctrine. Ace objects saying it is inadmissible because Napton was not an employee then.

Evidence Outline

Page 78

Constitutional Issue: Unlike Bruton, the Constitution has nothing to say because the confrontation clause applies only in criminal cases. There is no sixth amendment concerntherefore the whole jury instruction rule does not apply. And a court in this setting will not likely bifurcate the proceedings. But, don't forget good old Due Process!! Answer: 1. The statement regarding the brakes is likely to be fully EXCLUDED. It is IRRELEVANT, and thus inadmissible, when offered against the declarant NAPTON, assuming he owed no duty to the plaintiff regarding maintenance of the brakes. The statement would be RELEVANT against ACE, but it is NOT admissible because it IS hearsay and ACE did not make the statement (no 801(d)(2)(A) admission) and NAPTON was not acting within the scope of his employment (he had been fired when he made the statements) (no 801(d)(2) (D)). Note however that in real life NAPTON is likely to WANT to testify about the brake failure as an Affirmative Defense against the complaint, and perhaps a cross-claim against ACE, which would change the situation substantially). 2. The statement about speeding is relevant against Napton, and admissible against him as his own admission. The statement is relevant about ACE's respondeat superior liability, but it is hearsay and not subject to any of the 801(d)(2) exemptions. It is likely to be admitted against NAPTON accompanied by a limiting instruction in favor of ACE (upon request). You should not use the statement by Napton to establish Aces negligence. The statement may only be used to establish Naptons negligence, and if you find Napton is negligent, you can find Ace negligent under the doctrine of respondent superior. (In your answer you must state you are working under the assumption that the applicable substantive law accepts the doctrine of respondent superior) Forest: Relevance: The admission[s] by Napton tends to prove two points relevant to the liability of both himself and Ace First, the brakes failed suddenly, which suggests that Ace did not properly maintain the truck, hence that Ace is responsible and Napton is not (it looks as though Napton had no prior warning that the truck was unsafe); Second, Napton was speeding, which suggests directly that he is responsible and indirectly (through respondeat superior) that Ace is responsible. Admissibility: The first statement should probably be excluded, and the second admitted under instructions requiring the jury to consider it only against Napton. Napton: The brakes "just failed." Probably this statement should be excluded: It is inadmissible (though relevant) against Ace, and it does not advance O'Brien's case against Napton, thus being arguably irrelevant (though otherwise admissible) against him. A problem with this argument is that Napton's statement about the brakes seems relevant to the O'Brien/Napton dispute because it helps Napton: Still, it has that tendency only because it suggests that Ace failed properly to maintain the truck -- and it is inadmissible against Ace. It is incongruous to

Evidence Outline

Page 79

argue that a statement that is admissible (competent) against Napton but not against Ace should be admitted because it helps Napton by hurting Ace. Napton: "I was speeding." This statement by Napton is evidence that is relevant against both defendants (suggesting both Napton and Ace are liable to O'Brien). But again the statement is only admissible (competent) against Napton. No doubt it should be admitted against Napton, though Ace is entitled under FRE 105 to a limiting instruction telling the jury not to consider the statement as evidence against Ace. Severance? Possible but unlikely: Like criminal cases, civil suits too can be severed for trial (see FRCP 42(b)). But where respondeat superior applies, joinder of claims against agent and principal is commonplace, and severance for trial highly unusual. Exclusion under FRE 403: Again, possible, but unlikely. And of course Ace can argue under FRE 403 that limiting instructions will fail to work, so the statements by Napton should be excluded as posing too great a risk of jury misuse ("unfair prejudice"). But unless other evidence of negligence is overwhelming, Napton's statements seem so important to O'Brien's claim that exclusion is unlikely. Limiting instructions are the most that Ace can expect. There is prejudice, but the statement is highly probative as well, and 403 favors admissibility. Limiting Instruction under FRE 105 More Likely: Limiting instructions are looked on more favorably than not admitting the statements, even where it appears unlikely that the jury can follow their directive. Respondeat Superior: Because the doctrine of respondeat superior means Ace is liable for torts by its agent Napton (assuming he acts in scope of employment), it is possible to take the position that no limiting instruction is necessary for Napton's second statement ("I was speeding"). Even without an instruction, the jury will only take the statement as proof of how fast Napton was driving (that is all it says), hence as evidence of Napton's negligence. If the jury finds Napton negligent, then by operation of law (respondeat superior) Ace is responsible in damages to O'Brien. The statement is only evidence against Napton, and when he is found negligent Ace is on the hook. Admissions by Employees & Agents Mahlandt Deconstructing Hearsay under Mahlandt using the forest: 1. Preliminarily: is the question for the judge or for the jury? [FRE 104(a); FRE 104(b)] In Mahlandt the court rules that the issue of foundation (does it meet the requirements of FRE 801(d)(2)(D)?) is for the court to decide, along with the other matters. 2. Relevance: Is the offered evidence relevant? [FRE 401] [FRE 402] In Mahlandt statements regarding how the child's wounds were inflicted were highly relevant. Evidential Hypothesis: If the party thinks and or states that the wolf bit the child, that has a strong tendency to show that fact of consequence (the wolf,

Evidence Outline

Page 80

which was under the control of the declarant-party-agent, bit the child) more likely than not. Therefore, the evidence is relevant and thus admissible under FRE 402. 3. Is the offered evidence hearsay? [FRE 801, et seq.] [Presumptively Excludable FRE 802] (a) Does the evidence fit within the definition of hearsay of FRE 801(a),(b)&(c)? In Mahlandt two were written (the note and the minutes) and one was an oral statement intended by the parties to convey the message "[we think that] the wolf bit the child." FRE 801(a). The statements were made by persons. Mr. Poos is the declarant as to the first two, the author of the minutes, probably the corporate secretary, in any case, a person prepared the minutes. FRE 801(b). The statements are being offered by the plaintiffs to prove the truth of the matter asserted, i.e., that the wolf bit the kid. 801(c). (b) Even though it fits the 801(a),(b),(c) definition of hearsay, is it nevertheless within some exemption that expressly defines it as "not-hearsay" or "nonhearsay"? [FRE 801(d)] In Mahlandt the court rules that the two statements by Mr. Poos fit within the language of FRE 801(d)(2)(D): They were made by an employee because Poos is employed by defendant (this appears to be uncontroverted here); the statements regarded matters within the scope of the employment (he is the wolf-keeper and the wolf was actually in his care in his own home, for business-related reasons--the classroom visits); and they were made during a period when he was employed by defendant company. The minutes are authorized statements under FRE 801(d)(2)(C) but only admissible against the Center, not against Mr. Poos. (c) Even though it fits the FRE 801(a),(b),(c) definition of hearsay, AND despite it failing to be exempted by 801(d), is it nevertheless within some exception found in the rules, especially FRE 803 and 804? In Mahlandt you do not need to reach this question, because of the answer to the previous part. 4. Should the offered evidence be excluded, despite being relevant, and regardless of the answer to the hearsay question? [FRE 403] In Mahlandt the court notes that 403 exclusion is possible, but highly unlikely. Only as to the minutes, admissible against one party, but not against the other, does the level of unfair prejudice justify exclusion. However, the other reasons for exclusion under FRE 403 are available under appropriate circumstances (confusion of the issue, misleading, cumulative, etc.). But, speaking generally, forcing a party to explain its own words will almost NEVER rise to the level of unreasonable or unfair prejudice when you are applying the admissions doctrine. The effect of a limiting instruction [FRE 105] comes into play here to reduce unfairness. Government Employees: Traditionally statements by public employees have not been admissible against the government, on the grounds that (1) such people do not have the same sort of personal stake in the outcome of any dispute as private employees have, and (2) agents cannot bind the sovereign.

Evidence Outline

Page 81

However, there is a new line of cases that disagrees with this. However, the standard rule is that admissions by public employees should not be held against the government. Mahlandt v. Center A party to litigation should be required to explain thier own words in court.That is what FRE 801(d)(2) is about. Note that BOTH Kenneth Poos AND his employer are parties to this action. Therefore, you must distinguish admissibility against each of them. Facts: This is a civil action for damages arising out of an alleged attack by a wolf on a child. Mr. Poos is the Director for the Research Center and he was keeping the wolf at his house in order to take the wolf to schools to show kids. A kid walked by his house and allegedly got bit by the wolf. The child and the wolf were in the same enclosure and there are 2 theories of what occurredthe child dragged himself under the fence which caused the scratches or the wolf caused the scratch. The main issue is the cause of the lacerations on the kid. The jury brought in a verdict for the defense, however at trial the court excluded the following statements: Statement 1: Within an hour after he arrived home, Mr. Poos left a note on Sextons (the president of the Research center) that Sophie bit a child that came in our back yard. All has been taken care of. I need to convey what happened to you. o Written assertion by declarant Poos that is being used to prove the child was bitten by the wolf Statement 2: Later that day, Mr. Poos told Mr. Sexton what had happened that "Sophie had bit a child." o Oral assertion by declarant Poos that is being used to prove the child was bitten by the wolf. Statement 3: A company meeting was held, and Mr. Poos was not present at that meeting. The minutes of that meeting reflect that there was a "great deal of discussion . . . about the legal aspects of the incident of Sophie biting the child." o Written assertion by declarant Secretary of the organization who is acting on behalf of the corporation. Are Mr. Pooss statements admissible against Mr. Poos? Clearly the statements by Kenneth Poos are admissible against him. FRE 801(d)(2)(A) permits use of a statement against the person who made it regardless whether he acts in "his individual or a representative capacity," so it does not matter that Poos apparently spoke as agent for the Center. Is there a reliability/trustworthiness component required by the rule? NO the ACN tells us, the admissions doctrine is unburdened by the reliability trustworthiness requirements such as having personal knowledge. Are they admissible against the Research Center? Yes, because of FRE 801(d)(2)(B). It is a statement made by an agent during the scope of the agency. You must go to the common law of agency to determine that indeed, an employee is an agent. As an employee, Poos is an agent and therefore the statements are admissible against the company.

Evidence Outline

Page 82

The defense tried to argue the admission of a third party does not account for statements made in house. They should only apply to third parties. The court rejects this argument. What about the meeting minutes? The minutes are admissible against the Center, but they are not admissible as to Mr. Poos. Mr. Poos was not at the meeting and never said these statements. (801(D)(2)(C) The admissibility of admissions is a one way street! Although statements of an individual employee can be used against the company, a companys statements cannot be used against the employee Reliability/Trustworthiness and the Admissions Doctrine Judge Weinstein thought there should be an implied condition of reliability or trustworthiness that we should read into the admissions doctrine. AKA it should be mandated that when courts look at 801, they determine if the statements are reliable and trustworthy. However the court objects this reasoning. It is not required but you can use reliability factors at the FRE403 balancing stage. (See below.) Although it is not mandated, we may find the reliability and reasonableness in FRE403. It is possible to weigh this aka: it defeats its probative value if it is not highly reliable. FRE 403 Discretion There is left only the question of whether the trial court's rulings which excluded all three items of evidence are justified under Rule 403. Rule 403 does not warrant the exclusion of the evidence of Mr. Poos' statements as against himself or Wild Canid Survival and Research Center, Inc. But the limited admissibility of the corporate minutes, coupled with the repetitive nature of the evidence and the low probative value of the minute record, all justify supporting the judgment of the trial court under Rule 403. The minutes have a low probative value. The lack of personal knowledge matters here. Although this doesnt matter under 801, it certainly matters in determining the probative value. The combination of lack of personal knowelge, the limited admissibility (it is only admissible against the corporation) and the repetitive nature of the statements all lean in favor of not allowing its admission because it has very little probative value. Problem 4-G: I was on an errand for my boss: A truck driven by Rogers (an employee of Farm Right) got in an accident with Story and Rogers said Sorry, I was delivering for Farmright and I got distracted. The company is trying to say Rogers was not acting in the scope of his employee. Can the driver's statement prove agency? Yes. However, additional proof of agency is also required. (last statements) The content of the statement may be considered but is not alone sufficient to establish agency. Two Statements: In substance, Rogers says two things. First, he is employed for Farmright and was acting in the scope of his employment ("making a delivery") at the time of the accident. Second, he was probably negligent (he was "distracted" at the time). Rogers was acting in the scope of his employment:

Evidence Outline

Page 83

The Judge Decides: 104(a) The court has to find by a preponderance of the evidence that all the requirements imposed in the rule itself are met. Here, the predicate facts are that declarant was an agent of defendant speaking on matters within the scope of his duties. This is only to determine admissibility (not fault.) The judge determines if there is agency. o They can consider the statement itself and other statements that support it. Bootstrapping element: independent evidence that can establish the predicate facts. The independent facts must be beyond the statement itself which show agency. In this case, the truck Rogers was driving had a farm right sign on it, Rogers was wearing a farm right uniform, employee records show he was an employee. o The statement alone is not alone to establish agency and the statement will not be able to pass by the court. But almost all companys have some sort of outside records of their employees. It is almost undoubtedly so there will be independent facts to establish agency. The jury will have to resolve the same issues in deciding the case on the merits. If the judge decides the predicate facts against the plaintiff, the jury may still find the employee was not negligent and therefore the company is not negligent. Rogers was negligent This statement does not assert a point that must be proved to make the statement admissible. The negligence statement is admissible against the company both because of respondeat superior (which makes it relevant) and because of FRE 801(d)(2)(D), because there is enough evidence for the court to conclude that; (1) it was given by an employee, (2) about a matter within (he is a truck driver!), and (3) the driver was then still employed by the company (unlikely he had been fired in the intervening 30 minutes). Co-Conspirator Statements Co-conspirator statements are admissible if (1) declarant and defendant conspired ("coventurer" requirement), and the statement was made (2) during the course of the venture ("pendency" requirement) and (3) in furtherance thereof ("furtherance" requirement). Bourjaly: Clarance, an informant for the FBI arranged to sell cocaine to Lonardo. Lonrado agreed that he would find individuals to distribute the drug. The petitioner (Bourjaly) was charged with conspiring to distribute cocaine. They want to offer a phone conversation regarding Leonardo talking about the participation of a friend (who is Bourjaly) on the transaction. A conspiracy is an agreement between 2 or more people to commit a criminal offense. One agrees to this through words. We have to prove the declarants participation in the crime and then use that against others in the crime.

Evidence Outline

Page 84

Procedure for Co-Conspirator Statements Before admitting a co-conspirator's statement over an objection that it does not qualify under Rule 801(d)(2)(E), a court must be satisfied that the statement actually falls within the definition of the rule. There must be evidence that there was a conspiracy involving the declarant and the nonoffering party, and that the statement was made "in the course and in furtherance of the conspiracy." o A statement made after arrest is not a conspiracy o However, if a person confesses to an undercover agent, the declarant does not know this and this is considered part of the conspiracy. The existence of a conspiracy and petitioner's involvement in it are preliminary questions of fact that, under Rule 104(a), must be resolved by the court. The court applies a preponderance of the evidence standard. What Proof is Required to Meet the Preponderance Standard? (a) Independent evidence, or (b) The statements themselves (which raises the bootstrapping problem) BOTH are required! Statements themselves can be included, but you can also use independent evidence of them in order to establish whether there was a conspiracy. In determining whether a conspiracy existed and whether the D was a member of the conspiracy, the court can look at the independent evidence (evidence other than the statements sought to be admitted) to determine whether a conspiracy existed. The fact the petitioner went through all of the steps to obtain the cocainhe showed up to the hotel parking lot, at the pre-arranged time, he had $20,000, etc. The government certainly put forth enough facts for the trial judge to find by a preponderance of the evidence that there was a conspiracy and petitioner participated in it. A co-conspirator's statements could themselves be probative of the existence of a conspiracy and the participation of both the defendant and the declarant in the conspiracy. Statements AND other Evidence = Conspiracy = Admissibility Court did not decide in this case whether the courts below could have relied solely upon Lonardo's hearsay statements to determine that a conspiracy had been established by a preponderance of the evidence. Malavet: I seems the amended language of 104(a) makes it clear that you need more than just the statements. Reliability of Co-Conspirator Statements There is a rebuttable presumption of unreliability of co-conspirator statements, but if you can meet all the co-conspirator requirements, it is in and of itself reliable enough. By meeting the 801(e) prongs, the statements are already considered reliable. Even if out-of-court declarations by co-conspirators are presumptively unreliable, trial courts must be permitted to evaluate these statements for

Evidence Outline

Page 85

their evidentiary worth as revealed by the particular circumstances of the case. The coconspirator exception is firmly enough rooted in or jurisprudence so that the Confrintation clause does not require an independent inquiry into reliability. If the opposing party is unsuccessful in keeping the evidence from the factfinder, he still has the opportunity to attack the probative value of the evidence as it relates to the substantive issue in the case.

Problem 4-H: Drugs Across the Border Arlen and Bud decide to import Cocaine from Columbia. They determine Bud will fly there with Carol and aquire the stuff while Arlen lines up the customers. When they arrive at the airport the DEA stops Carol and follows her. The agents arrest Bud, Carrol and Arlen. Carrol then says Bud acquired the cocaine and she was just helping him import it Three Conversations: testimony by Connie describing what Bud told her in the bar (Arlen "fronted us the buy money"), testimony by Don describing what Arlen said (Bud's "gone south to make the buy") testimony by the DEA agent describing what Carol told him ("Bud made the buy") Each conspirator is responsible for the statements of each other conspirator because those statements are admissible against all. Connie's statement in the bar: would likely be termed "mere narrative" because it does not advance the conspiracy. The fact that it is uttered in the presence of an outsider (Connie) in casual conversation in a bar both point toward this conclusion. DEA Agent Don's Account of Arlen's Statements: Don's account of his conversation with Arlen (Bud's "gone south to make the buy") satisfies the furtherance requirement even though the statement destroyed the venture. The furtherance requirement means that the statement seems to further the venture, presumably from the perspective of the declarant, Carol's Post-Arrest Admission: Carol's admission to arresting agents (implicating herself and claiming "Bud made the buy") raises the Bruton problem. The statement is obviously admissible against her under FRE 801(d)(2)(A), but its admitting it would violate Bud's confrontation rights under Bruton. The coconspirator exception does not cover post-arrest confessions by one of several co-offenders. Procedure: The judge is likely to admit statements, subject to a motion to strike if the prosecutor fails to adduce independent evidence of the conspiracy. He is likely to make a preliminary finding that there is prima facie evidence of conspiracy (perhaps on the basis of a James hearing in which agents testify to the observed movements of the three defendants, perhaps simply on the

Evidence Outline

Page 86

basis of the prosecutor's assurance that evidence of that behavior will be adduced). If asked by the defense, the judge will likely make a final decision under FRE 104(a) that Arlen, Bud, and Carol did indeed conspire in the drug business, and that Arlen's statement satisfies the pendency and furtherance requirements. Again, the judge will base his decision on the "independent evidence" of the behavior of the three during the time, and will conclude that this evidence establishes the predicate facts by a preponderance of the evidence. All of this occurs without the jury being present. FRE 104(c). V. Unavailability of Declarant

FRE 804 Unavailability as a witness does not mean the declarant must be physically unobtainable -hiding or beyond reach of subpoena. The requirement is satisfied if his testimony is unobtainable. This includes refusing to testify, or properly invoking a privilege. Procedure: The trial judge determines whether the declarant is unavailable under FRE 804(a), meaning that the question is one of "admissibility" under FRE 104(a). 1. Claim of privilege. Under FRE 804(a)(1) a declarant is unavailable if exempted from testifying by court order on ground of privilege. In criminal cases, often witnesses invoke the Fifth Amendment privilege against selfincrimination. 2. Refusal to testify. FRE 804(a)(2) contemplates actual refusal: On the stand, declarant declines to answer and does not cooperate when ordered to answer. 3. Lack of memory. A declarant who testifies that he does not remember "the subject matter" of his prior statement is unavailable under FRE 804(a)(3). 4. Death, illness, infirmity. Under FRE 804(a)(4), determining unavailability due to death has not posed problems, but the same is not always true of "illness or infirmity." A minor ailment from which speedy recovery is expected should not satisfy the requirement, even though the declarant cannot attend trial on a given day. In this situation it should be possible to adjourn the proceedings to allow time for recovery. But a serious illness of uncertain prognosis is likely to be enough. Context is important: 5. In some settings, mental condition makes a witness unavailable to testify even though the modern view is that insanity does not disqualify one from giving evidence. *** 6. Unavoidable absence. Oversimplifying for the moment, a declarant is unavailable under FRE 804(a)(5) if her presence cannot be obtained at trial by subpoena or "other reasonable means." o Even a witness beyond reach of subpoena is not necessarily unavailable, for "other reasonable means" may secure her presence. Occasionally courts expect parties simply to invite her to, attend, and

Evidence Outline

Page 87

in the case of the government in criminal cases, to offer to pay travel expenses. 7. Procurement or wrongdoing. A party who procures the absence of a declarant should not be allowed to invoke one of the exceptions that absence normally brings into play. The last sentence of FRE 804(a) so provides, though it is seldom invoked. Problem 4-L The government let her go Rick Masters is charged with importing cocaine when he and a 17 year old Australian named Snell are arrested as they arrive in Puerto Rico. Snell was arrested and detained in an adult prison facility, and she gave a statement that lead to the indictment of Masters. Several weeks later the US attorney sought permission to take Shells deposition, suggesting that she might return to Australia and it would be impossible to bring her back to testify against Masters. The court permitted the deposition over defense objection, and defense counsel attended but did not question Shell. She incriminated Masters, testifying that he hired me to carry the cocain which I was to turn over to him in exchange for $500 and my airline ticket to Australia when we arrived in Puerto Rico. Several weeks later at trial, the government offers the deposition testimony because Snell refused to come back. The defense argues she is only unavailable because the government let her go. FRE 804 cannot be used because she is not unavailable. She is a crucial witness, you need her to be cross examined and her unavailability is only the responsibility of the government. The government could have detained her or incarcerated her as a material witness so they could produce her at trial. Government tries to argue, they called the embassy and she refuses to return, that she is a minor, she is not a citizen, etc. It is unlikely the government could have held her for a long time because it is very costly, she wasnt charged with the crime, and it would have been difficult because she was a minor. The deposition should probably be excluded. Although she may be unavailable under the rule, at the 403 balancing stage, it seems the girl is too central, too crucial of a witness to go forward without her. In criminal cases, depositions for use in trial are to be taken only in exceptional cases. You have to make reasonable effortsuse the counselors in the foreign countries, offer to cover their expenses, promise they will not prosecute them, talk to the parents if it is a minor. If you give up and do not give reasonable efforts then the court will not allow you to claim they are unavailable. Barber v. Page Defendant claimed that the use of a transcript of a codefendant's preliminary hearing testimony in defendant's state trial deprived him of his federal constitutional right to confrontation. The confrontation clauses object was to prevent depositions or ex parte proceedings from being used against prisoners in lieu of a personal examination and cross-examination of the witness in which the accused had an opportunity not only of testing the recollection and sifting the conscience of the witness, but also of compelling him to stand face to face with the jury in order that they could look at him and judge his demeanor upon the stand.

Evidence Outline

Page 88

The prosecution has an obligation to make good faith efforts to product the witness. The right of the defense to cross examine, preferably at trial. Was the Defendant unavailable? The state made absolutely no effort to obtain the presence of Woods (the D) other than to ascertain he was in prison 225 miles away. Many commentators have held the mere absence of a witness from the jurisdiction is sufficient ground for dispensing with confrontation on the theory that it is impossible to compel his attendance, because the process of the trial court is of no force without the jurisdiction, and the party desiring his testimony is therefore helpless. With modern technology, this theory is no longer valid. In this case, the state authorities made no efforts to avail themselves of available means to secure Woods presence at trial. A witness is not unavailable for purposes of the confrontation requirement unless the prosecutorial authorities have made a good faith effort to obtain his presence at trial. The state made no such efforts here. Confrontation clause and Rule 804(a)(5) both require the good faith effort to find someone. Waiver: The confrontation right can be waived if you chose not to cross examine when you have the opportunity to cross examine. If there was a prior opportunity to cross examine and the attorneys failed to cross examine or chose not to, then it may be waived. But a waiver has to be an intentional relinquishment or abandonment of a known right or privilege. NOTE: even if the P would have cross examined Woods at the preliminary hearing, this still would not have met 6th amendment muster. The right to confrontation is a trial right. It includes both the opportunity to cross examine and the occasion for the jury to weigh the demeanor of the witness. The Barber court says the constitutional confrontation right is to cross examine the witness in front of a live jury at TRIAL. Confrontation entails live, in court testimony. If we have a showing of unavailability (which in this case we dont) the court might be willing to consider testimony at a pre-trial hearing acceptable. If the witness is not constitutionally shown to be unavailable, the court will apply a strict definition of what the confrontation right is. Technology and the Confrontation Clause In narrow child assault cases, the court will allow closed-circuit television or possibly the use of satellite dishes to cross examine the witness (and the confrontation clause will be met) In sexual assault situations, the courts have applied a very narrowly tailored exception to the confrontation clause that allows the victims to testify without seeing the defendenat. There is also a case in Florida which allows the use of closed-circuit television. Other courts have also allowed the use of technological devises as an exception to the confrontation clause on a case-by-case basis. An important public policy of protecting a witness (such as children) may justify the Ds right to face-to-face confrontation.

Evidence Outline

Page 89

VI. The Constitution as a Bar to the Admission of Hearsay The Confrontation Clause entitles the accused to be there when witnesses testify against him, and to cross-examine. In Coy, the Court held that the Clause entitles the defendant not only to be present, and to see and hear the witnesses against him, but also to be in view of them. In the modern era, two decisions arrived o California v Green suggesting two kinds of statements pass muster [1] those subject to prior cross-examination because they were made in proceedings where defendant had a lawyer who tested them, and [2] those subject to deferred cross-examination at trial because defendant can then question the witness about what he said before. o Dutton v Evans suggesting the Confrontation Clause is satisfied if a statement possesses "indicia of reliability," similar to the factors underlying traditional hearsay exceptions. 6th Amendment/Hearsay Theories: o Production of Declarant (Roberts, Barber, Crawford) Requires the prosecutor to produce an available declarant in preference to his out-of-court statement, but has nothing to say about statements by people who are unavailable-whose presence or testimony the prosecutor cannot obtain. o Reliability of Statement (Roberts) The Clause sets a constitutional standard of reliability for hearsay offered against the accused, although concerns over reliability may be satisfied by circumstances similar to the ones associated with hearsay exceptions, and reliability is unimportant (or less so) if the accused can crossexamine. o Minimalist: the clause speaks only to live testimony and has nothing to say about out-of court statements. It entitles the accused to be present and cross-examine witnesses who testify but does not stop the prosecutor from offering testimonial accounts of what others have said. o Testimonial (Crawford) Clause applies to "testimonial" statements (at least where the declarant cannot be cross-examined), and "testimonial" refers at least to statements to law enforcement officers describing crimes, where the purpose is to aid in prosecuting or trying the alleged culprit. ***Ohio v. Roberts and Barber adopted the Production and Reliability theories. Crawford retains Production and rejects the Reliability prong from Ohio v. Roberts. It also adopts the Testimonial theory. Ohio v. Roberts The Court has emphasized that the Confrontation Clause reflects [1] a preference for face-to-face confrontation at trial, and [2] that "a primary interest secured by [the provision] is the right of cross-examination." If the Declarant is unavailable, (not present for cross-examination at trial) the Confrontation Clause normally requires a showing that he is

Evidence Outline

Page 90

unavailable. Even then, his statement is admissible only if it bears adequate "indicia of reliability." Reliability can be inferred without more in a case where the evidence falls within a firmly rooted hearsay exception. In other cases, the evidence must be excluded, at least absent a showing of particularized guarantees of trustworthiness. Exceptions labeled as "firmly rooted," by the Supreme Court or other appellate courts: Coconspirator statements, excited utterances, statements for medical diagnosis or treatment, business records, dying declarations, agent's admissions and public records. Crawford v. Washington The court rejected the view that the Confrontation Clause applies only to in-court testimony. Testimonial statements: An accuser who makes a formal statement to government officers bears testimony in a sense that a person who makes a casual remark to an acquaintance does not. Statements taken by police officers in the course of interrogations are also testimonial under even a narrow standard. Malavet thinks you should focus on the gathering authorities (government agent) intent in order to determine if the statement is testimonial. This is not to say you should not take into account the intent of the declarant. However, the majority of people look to the government agents intent. Production Theory: The admission of testimonial statements of a witness who did not appear at trial are not allowed unless [1] he was unavailable to testify, and [2] the defendant had had a prior opportunity for cross-examination. When Declarant is Unavailable: Testimonial statements of witnesses absent from trial have been admitted only where the declarant is unavailable, and only where the defendant has had a prior opportunity to cross-examine. When Declarant is Available: When the declarant appears for cross-examination at trial, the Confrontation Clause places no constraints at all on the use of his prior testimonial statements. The Clause does not bar admission of a statement so long as the declarant is present at trial to defend or explain it. The Clause also does not bar the use of testimonial statements for purposes other than establishing the truth of the matter asserted. Gets rid of reliability requirement: The Constitution prescribes a procedure for determining the reliability of testimony in criminal trials, and we, no less than the state courts, lack authority to replace it with one of our own devising. Where testimonial statements are at issue, the only indicium of reliability sufficient to satisfy constitutional demands is the one the Constitution actually prescribes: confrontation. The only time the Confrontation Clause applies is when you are dealing with testimonial statements. If the declarant is available for trial and the statement is testimonial, then the 6th amendment right is met. If the

Evidence Outline

Page 91

declarant is unavailable and the statement is testimonial, the D must have an ability to cross-examine the statements in order for the 6th amendment right to be met. Statements Subject to Cross-Examination Davis v. Washington The two cases required a determination of when statements made to law enforcement personnel during a 911 call or at a crime scene were "testimonial" and thus subject to the requirements of the Sixth Amendment's Confrontation Clause. 1. The statement identifying defendant during the 911 call was not "testimonial." A 911 call was not designed to establish or prove past facts, but to describe circumstances requiring police assistance. The caller spoke about events as they were actually occurring while facing an ongoing emergency, rather than describing past events. The elicited statements were necessary to resolve the emergency NOT to investigate events. 2. In the other case, the statements of the alleged victim were made in response to an officer's questions in a room away from defendant when there was no immediate threat to her person. The purpose of the interrogation was investigatory. The statements recounted past events; they did precisely what a witness did on direct examination and were inherently "testimonial." The Court declined to relax the requirements in domestic violence cases but pointed out that the right to confrontation could be forfeited by wrongdoing. Davis is the first important attempt by the Supreme Court to define the limits of Crawford. Davis does several important things: 1. First, Davis rules that the Sixth Amendment bar against the use of hearsay statements applies only to testimonial statements. This completely overrules Ohio v. Roberts as a constitutional test for hearsay. 2. Second, Davis illustrates the definition of "testimonial" statements in the context of police intervention by creating a continuum that goes from the initial management of an emergency, which does not produce testimonial statements (Davis v. Washington), to the interrogation of witnesses of a crime once law enforcement is in effective control of the scene, which produces testimonial statements (Hammon v. Indiana). 3. In this context, the court also unanimously treats 911 operators as agents of law enforcement for sixth amendment purposes. Sixth Amendment provides: "In all criminal prosecutions, the accused shall enjoy the right . . . to be confronted with the witnesses against him. Crawford v. Washington: this provision bars "admission of testimonial statements of a witness who did not appear at trial unless he was unavailable to testify, and the defendant had had a prior opportunity for cross-examination." It is the testimonial character of the statement that separates it from other hearsay that, while subject to

Evidence Outline

Page 92

traditional limitations upon hearsay evidence, is not subject to the Confrontation Clause. Two steps 1. Is this admissible? 2. Is it constitutional? Is it Admissible? Davis: Yes, her hearsay statements are admissible under FRE 803(1) or (2) - excited utterance and present sense impression. Hanna: Yes, her hearsay statements are admissible under 803(1) or (2)excited utterance and present sense impression. Also, both statements are relevantthey identify the assailant. Is it Constitutional? (Do these pieces of evidence comply with the 6th amendment) Davis: Sixth Amendment does not guarantee them anything--statements are not testimonial and Sixth Amendment doesnt apply. Reliability ONLY SURVIVES AS a requirement of the rules not as a requirement of the sixth amendment. Hanna: Statements are testimonial Nontestimonial Statements: Statements made in the course of police interrogation under circumstances objectively indicating that the primary purpose of the interrogation is to enable police assistance to meet an ongoing emergency. A 911 call, and the initial interrogation conduicted in connection with a 911 call is ordinarily not designed primarily to establish or prove some past fact, but to describe current circumstances requiring police assistance. Testimonial Statements: Statements when the circumstances objectively indicate that there is no such ongoing emergency, and that the primary purpose of the interrogation is to establish or prove past events potentially relevant to later criminal prosecution. The court creates a continuum for the emergency doctrinenot every conversation between the police and a declarant will be considered testimonial. Examples interrogations soley directed at establishing the facts of a past crime, in order to identify the perpetrator. A solemn declarion or affirmation made for the purpose of establishing or proving some fact. There is no emergency in progress. When the officer arrived, things were fine, there was no immediate threat to anyone. Malavet says the only critical difference is that the officers had not yet arrived. Until law enforcement officers arrive on the scene, the polices main concern is the safety. However once the police have the situation under control, it is likely those statements will move out of the emergency exception. The judge seems to have a lot of discretion determining when something is non-testimonial and when something is testimonial. WHAT IS HAPPENING-Not testimonial WHAT HAPPENED-testimonial

Evidence Outline

Page 93

Health and statement of the victim and health and safety of the police officers and first responders. Most of the time, whenever you are talking to the police, you are going to be making testimonial statements. However, the emergency is an exception. AKA you have to show EXIGENT CIRCUMSTANCES (emergency situations) Forfeiture Doctrine (FRE 804(b)(6) When defendants seek to undermine the judicial process by procuring or coercing silence from witnesses and victims, the Sixth Amendment does not require courts to acquiesce. Ds have a duty to refrain from acting in ways that destroy the integrity of the criminal-trial system. One who obtains the absence of a witness by wrongdoing forfeits the constitutional right to confrontation. Forfetire is now controlled by the 2008 Giles caseif the prosecuter can show that the D or the objecting party is responsible for the absence of an absent witness AND the actions of the D were intended to prevent the testimony, then there is FOREFEITRE and that constitutes a waiver of your 6th amendment objection. This is a 104(a) question. Courts use a preponderance of the evidence standard to determine if the D has forfeited their 6th amendment confrontation right. Children and testifying Coy: Scalia says there is no blanket exception for children. You cannot exclude a child unless you have cause. In Craig Scalia says there is cause to make special accommodations for the children. Where the court comes up with specific findings of traumatic events for the child, you can provide exceptions for the child when they are testifying (such as closed-circuit television). Problem 4-Q: "Your Witness" Does the defense have an opportunity for full and effective cross-examination under Green if the prosecutor calls the witness and does not broach with her either the prior statements that the prosecutor intends to use or the acts, events, or conditions described in them? NOthe defense did not waive their right to cross examination! 1. First, prosecutor did not carry burdens. Calling T.E. and running through the preliminaries of age and school grade and knowing truth from falsehood is not an attempt to get her testimony, but rather an attempt to pass the buck so that the defense must play the heavy and broach the subject that nobody wants to touch, which is whether T.E. was abused. The prosecutors polite litany of questions does not even show in any convincing way that T.E. has forgotten what happened to her on the earlier occasion. 2. Second, there must be something to test or challenge. When a witness like T.E. gives no substantive evidence, the defense has nothing to test or challenge. When T.E. is excused, she has said nothing about the offense and, at least as a formal matter, she has given no evidence against the defendant. Defendant literally has nothing that he can test or challenge, and there is no occasion for the process of cross-examination to operate.

Evidence Outline

Page 94

3. Third, defendants cannot afford to initiate challenges. In cases like this one in which the witness/declarant has said nothing that goes to the merits, defendants cannot afford to challenge what has yet to be offered. It makes a real difference whether defendants are left to do all the heavy lifting, and a nonopportunity cannot be viewed as an opportunity. VII. Unrestricted Exceptions

Analysis for 803 Rules Relevance Specific Rule of prohibitionFRE 801 (Hearsay) What was said? (break up the statementswas there a 911 call? Then the police arrive, etc.) Discuss each statement separately and note the following: o Who was the declarantnote it does not matter if the declarant was available. o Are the elements of the FRE 803 rule met to see whether it falls within the exceptionlook to the subjective intent of the declarant o Is there independent evidence that supports the statements? Although this is not required, it makes the statements more reliable and is highly encouraged. (Malavet thinks there should be independent evidence) There are independent facts which will not be admissible under 403, but the judge can still look at in determinging whether to admit. Are the statements useful? Answer will always be Yes Constitutional Analysisfocus on the subjective intent of the officer/gathering person to determine if statements are testimonial. If the statements are testimonial, then the D must have an option to cross examine. Is it admissible under FRE 403? Present-Sense Impressions (FRE803(1)): a statement describing or explaining an event or condition made while the declarant was perceiving the event or condition, or immediately thereafter. For present-sense impressions, the availability of the delcarant is immaterial Reliability as a Requirement of the Rules (Remember, reliability under the constitution no longer exists): 1. Is independent evidence required? Do we need evidence independent of the evidence itself (the bootstrapping evidence). We do not, but Malavet thinks the more independent evidence we have, the more probative (reliable) and less prejudicial it becomes. The independent evidence does not have to be admissible. But the independent evidence can be shown to the judge so the judge is more likely to admit the admissible statements. The inadmissible independent evidence can support the admissibility of other things that the judge will admit under 104(a). 2. Is the evidence useful?

Evidence Outline

Page 95

To the extent that we have independent evidence that confirms the content of the statement is that independent evidence useful/helpful? Generally the answer is yes, because this goes to the reliability of the statement. Nuttall v. Reading Co. Holding: Wife was permitted to testify to husbands statements about being ill after conversation with boss as evidence that he was forced to work Florence Nuttall argued the court erred in the exclusion of the evidence, including: 1. two affidavits (one by Fireman John O'Hara, the other by Conductor James Snyder, both of whom worked with Nuttall on the occasion in question), 2. her own testimonial account of her husband's phone conversation with the yardmaster, and 3. testimony by the fireman about remarks Nuttall made in the trainyard on the day in question. Wife's Testimony Regarding Telephone Conversation Nuttal witnesses a telephone conversation between her husband and her husbands employer and wishes to relay to the jury what she observed. This is hearsay. These are oral statements by Mr. Nuttal being offered to prove he was sick, the employer knew he was sick, and the employer forced him to come to work. Therefore we need an exception. This conversation took place in the presence of his wife and at the end there was an additional statement made to her after he had hung up the receiver. The allegation is that the employer forced the employee to go to work in spite of the employees illness where he was exposed to inclement weather which proximately caused his illness and eventually his death. The court found the conversation tends to show that Nuttall was being forced to do something by somebody. The "somebody" is identified without difficulty. (It was obviously the employer.) Nuttall's state of mind established that his state of mind was induced by something his employer did. (I dont want to go to work but I feel compelled to go because my employer is making me.) When a man talks as Nuttall did and acts as Nuttall did during and immediately following a conversation on the telephone with his boss, it has a tendency to show that the boss was requiring him to come to work against his will. These statements are reliable because they were made at the very moment he heard what Marquette had to say and immediately thereafter. Such characterizations, since made substantially at the time the event they described was perceived, are free from the possibility of lapse of memory on the part of the declarant. And this contemporaneousness lessens the likelihood of conscious misrepresentation. The statements were made in circumstances which assure their trustworthiness. It makes the truth purpose likely true. Here we also have a lot of independent evidence that contribute to the reliability of the statements. These statements are harmful error and are ground for reversal Mistakes on admissibility of evidence are almost inevitable during a hotly contested

Evidence Outline

Page 96

trial. Unless they seriously affect the case they are not a ground for reversal. But here the rejected evidence goes to the very heart of the plaintiffs case. Two exceptions apply to these statements: Nuttles own state of mind (I feel I am being compelled to go to work) under 803(3) and the source of his state of mind (the boss is compelling me to go to work) under 803(1). State of mind FRE 803(3) goes to what Nettle says he feels (it is the EXPRESSED assertion of the state of mind, not the implied one.) Here the assertion was I am being forced to go to work. As to the source of the state of mind/feeling compelled to go to work you need another rule. That is why you need FRE 803(1)that the boss was the source of the state of mind. Here the need to show the reliability of the statements is even higher. Excited Utterance (FRE803(2)): a statement relating to a startling event or condition made while the declarant was under the stress of excitement caused by the event or condition. Again availability of the declarant does not matter. NOTE: there is a growing body by law that says statements that are admissible under FRE803(a)(2) as excited utterances are inadmissible because they are testimonial and it is impossible to comply with the sixth amendment because they were received by the law enforcement or agents. U.S. v. Arnold Three statements being challenged: 911 callTamica called 911 and told the operator that Arnold had just threatened her with a gun. The statements to police upon police arrivalwhen officers arrived on the scene, they encountered a visibly shaken Tamica who explained she had just gotten in an argument with Arnold and that he threatened her with a gun. At this point we still didnt know where the D was and there was not effect control of the scene or the D. The statements to police after the D showed up on the sceneTamica yells Thats him, thats the one who pulled the gun on me. She obviously feels threatened at this point. The officers never questioned her. The police were not in effective control of the situation. There was still an unsecure scene. All three questions (1) fit under FRE 803(2)(excited utterance), (2) have independent evidence that support the reliability of the statements made, and (3) are not testimonial under the constitutional analysis. At the rule stage in determining whether they fit the exception/their reliability focus on the subjective intent of the declarant Ex: the declarant subjectively was under the stress an excitement of the situation and therefore uttered the words. At the constitutional state in determining if the statements were testimonial focus on the subjective intent of the information gatherer.

Evidence Outline

Page 97

Ex: when the police was asking what kind of gun the D had, the D was intending to gather information for trial. Under FRE 803(2), a court may admit out-of-court statements for the truth of the matter asserted when they "relat[e] to a startling event or condition made while the declarant was under the stress of excitement caused by the event or condition." To satisfy the exception, a party must show three things. The BOP is on the offering party to show the following: 1. First, there must be an event startling enough to cause nervous excitement. 2. Second, the statement must be made before there is time to contrive or misrepresent. 3. Third, the statement must be made while the person is under the stress of the excitement caused by the event. All three inquiries bear on "the ultimate question": Whether the statement was the result of reflective thought or whether it was a spontaneous reaction to the exciting event. There is no need to make a precise showing of the lapse of time between the startling event and the out-of-court statement. The exception maybe based solely on "[t]estimony that the declarant still appeared nervous or distraught and that there was a reasonable basis for continuing [to be] emotional[ly] upset," The dissent raises the concern that the uncorroborated content of an excited utterance should not be permitted by itself to establish the startling nature of an event. (Bootstrapping Issue) The court applies an abuse-of-discretion review to a district court's application of the rule. Bootstrapping Problem: Can a statement admitted as an excited utterance prove the happening of the event on which its own admissibility depends? You should be weary of not providing any independent evidence. The ACN endorses rulings admitting statements where "the only evidence" of a startling event is "the content of the statement itself." However, you should try and have independent confirmation. We think this outcome is right for two reasons: 1. First, denying recovery in possibly deserving cases where a job situation makes firsthand evidence scarce is simply intolerable. 2. Second, courts have applied the requirement badly. Usually we have independent proof of an exciting event -- clinical evidence of ailment or injury, or lay observations of the speaker acting "out of sorts." State of Mind (FRE 803(3)): then existing mental, emotional, or physical condition (such as intent, plan, motive, design, mental feeling, pain, and bodily health), but not including a statement of memory or belief to prove the fact remembered or believed unless it relates to the execution, revocation, identification, or terms of declarant's will.

Evidence Outline

Page 98

It is very easy to get this exception confused with a nontruth use that goes to the persons motive, memory or belief, etc. Ask yourselfIs the statement itself an assertion of the state of mind (ex: I feel happy)? If the content itself is an express assertion it is being offered to prove the truth of the matter asserted and therefore you need the FRE 803(3) assertion. The use of statements that prove state of mind to infer that state of mind verses the use of statements that assert one thing, but from which we can infer state of mind. As formulated in FRE 803(3), the exception has four distinct uses: To prove declarant's then-existing physical condition, his then-existing mental or emotional condition, his later conduct, and facts about his will. Some states require the trial judge to exclude these statements where circumstances suggest insincerity. Other states hold truth or falsity of the statements is for the jury to determine, and a statement that falls within the exception is admissible without any preliminary finding of credibility by the judge. Then-Existing Physical Condition In personal injury suits, the exception is regularly invoked for statements describing aches and pains. The words must be describing how he feels as he talks. Then-Existing Mental or Emotional Condition Evidential Hypothesis: The statement (a) Infers or (b) asserts mental or emotional condition, AND that condition is itself an ultimate issue in the case. Problem with Fact-Laden Statements: often utterances indicating mental state are wholly or partially factual in nature. In what we can call "fact-laden statements," people may purposefully disclose state of mind by speaking in factual terms. Even if the statement is probative of state of mind, the entire statement usually includes some objectionable material or may be used by the jury for an impermissible purpose, which raises substantial 403 concerns. Ex: I am afraid of Bill because he said he was going to kill me and my family. (underlined portion is the fact-laden statement.) The first part (I am afraid of Bill) could be admitted under 803(3), however the facts lay an important problem because under 803(3), an existing mental state cannot be used to prove facts remembered. Therefore the second part must find another exception to come in under. Sometimes this could be 803(1) or another rule. In the above example, the second clause could come in under the verbal acts doctrine if the act is an element of the crime such as extortion) Problem 4-J: He says he'll kill me The Declarant is dead. Consider the Statement:

Evidence Outline

Page 99

Neff is after me again. He says he'll kill me and my family if I don't pay protection. I've already paid him $5,000, and I'm trying to steer clear of him, and I need help but I just don't know what to do. What does this statement say? Two ways to view this statement: 1. We can INFER Quade's state of mind from the statement, but the statement itself does not assert what Quade felt, which means that this does NOT fall under FRE 803(3), but is rather a non-truth/ nonhearsay use. 2. We might read it as necessarily implying fear (trying to steer clear of him, I need help), which would mean that we ARE offering it to prove the assertion ("I am afraid of Neff, because he threatened me"), which would then fall under FRE 803(3).

We can circumstantially infer that the statement necessarily implies


403 Balance: Where a statement tends to prove both a relevant state of mind and damaging facts, the real question is whether the risk of unfair prejudice outweighs probative worth. In an extortion case, where fear is an element of the crime, this statement is highly probative, but in a murder case, where Neffs fear is not an element but only a fact having circumstantial relevance, the statement is not very probative. Note, if the D was raising a defense of self defense, fear would become very probative and would become admissible. If D claims self-defense, fear on the part of the victim would tend to refute defense claims that the victim committed suicide. Consider the steps Minnesota Courts take in deciding whether to admit these types of statements in murder cases 1. The victim's state of mind must be a relevant issue. The victim's state of mind is generally relevant only where the defendant raises the defense of accident, suicide, or self-defense 2. The trial court must weigh the probative value of the evidence against the risk of unfair prejudice to the defendant. 3. A proper limiting instruction must be given to the jury. Subsequent Conduct Evidential Hypothesis: The statement either (a) infers or (b) asserts intent to do something AND it is offered to support the INFERENCE that the declarant in fact did the "something". What a person said is often admitted as proof of what she thereafter did (or did not do). Mutual Life v. Hillmon Defense is trying to argue that Mr. Hillmon is committing fraud on the insurance company (by faking his death so his wife can collect the proceeds of the life insurance policy.) The question is, who diedMr. Hillmon or Mr. Walters? The insurance co. is arguing Hillmon killed Walters so that people would think it was Hillmon who died, so that Ms. Hillmon could collect the money.

Evidence Outline

Page 100

The defendants offered in evidence to show the dead person was Walters, not Hillmon. They present letters from Walters. In one, Walters wrote to his sister in Iowa that he intended "to leave Wichita on or about March 5th, with Mr. Hillmon. In the other, Walters wrote to his fiancee in Iowa indicating his intent to leave Wichita "to see a part of the country that I never expected to see when I left home, as I am going with a man by the name of Hillmon, who intends to start a sheep ranch [and has] promised me more wages than I could make at anything else." The letters are being offered to prove (a) that Walters had the intent to travel with Hillmon and (b) that Walters in fact travelled with Hillmon. The "intent" proof today would be allowed by FRE 803(3) and the legitimate INFERENCE that he "travelled" would likewise be allowed and are not problematic issues. However, under the express language of FRE 803(3), "the person with whom I intend to travel is Hillmon" is admissible to prove that Walters believed he was travelling with Hillmon, but not to prove that he was in fact travelling with Hillmon, which is the real issue here. However the court in Hillmon says it can be used to prove both! We cannot infer someone elses intent from the declarants statement. When you involve someone other than yourself in a statement of intent, that requires that you know or believe something that occurs prior to making the statement (which runs afoul to 803(3)). Note that FRE 803(3) clearly would apply here, if it had been effect in 1892! Evidential hypothesis: The letters show Waters had the intention of going with Hillmon, which made it more probable both that he did go and that he went with Hillmon. This makes it more likely Hillmon killed Waters in Crooked Creek. Holding: This case is troubling because the court states that the letters are evidence that Walters in fact left with Hillmon, rather than the narrower Walters left for on or about the date of the Hillmon/Walters death, and Walters believed his companion to be Hillmon. Hillmon Doctrine: when the performance of a particular act by an individual is an issue in a case, his intention (state of mind) to perform that act may be shown. From that intention, the trier of fact may draw the inference that the person carried out his intention and performed the act. It can also be used to show the intention of a third party. The statements can prove both the declrants intentions and the intentions of a third party (in this case, that HIllmon accompanied him). Today, under FRE 803, you cannot use intentions to show what a third party did! (Although the Pheester court would disagree!) U.S. v. Pheaster Larry disappears. Right before he disappears he has a conversation with Francine and Doug. Francine and Doug are permitted to testify to Larry's description of what he planned to do. Francine testifies that when Larry picked her up that evening he told her in substance that "he was going to meet Angelo at Sambo's North at 9:30 P.M." in order to "pick up a pound of marijuana that Angelo had promised him for free." Doug testifies that Larry "made similar statements to him in the afternoon

Evidence Outline

Page 101

and early evening" and that on leaving the table "to go into the parking lot" Larry said "he was going to meet Angelo and he'd be right back." Francine also testifies that while with Larry on an earlier occasion she met a man named Angelo, and she identifies defendant as that man. Evidential Hypothesis The Government claims according to the Hillmon doctrine, Larry Adell's statements can be used to prove that the meeting with Inciso did occur. Under FRE 803, using Larrys statement of intent that Larry met with Angelo is unproblematic, the problem is when you use the statement to prove what a third party (Angelo) did. The fact that Larrys statements can be used to prove that the meeting did occur raises a difficult interpretation of the rule. When the Declarant states intention to do something with another person, it is problematic to conclude the other person did that something. In reality, we are using larrys statements to imply Angelos intention. In order for this to occur, the declarant should be dead or unavailable and the testimony concerning the statements should be relevant and posess a high degree of trustworthiness. (such as independent evidence that Angelo was present at the meeting) Phester says Hillman part 2 at least in certain circumstances survives 803(3). That is, even under 803(3), Hillmons liberality in admitting statements proving intent to infer action by someone other than the declarant is still good law today. Under the right circumstance, a statement of intent can improve someone other than the involvement of the declarant. Malavet is skeptical of this! The nature of this case makes it particularly necessary to use hearsay. (1) Larry is dead (unavailable declarant); and (2) there is independent confirmation of this meeting occurring. In applying the Hillman Doctrine to involve a third party there are factors which should be considered 1. The declarant should be dead or otherwise unavailable 2. The testimony concerning the statements should be relevant and possess a high degree of trustworthiness (independent evidence of the content of the statement and the persons involvement in the crime) In this case, there was an overwhelming amount of independent evidence.. 3. Other evidence from which the defendants guilt could be inferred. NOTE: this interpretation of the rule seems to allow exactly what the rule prohibits using the state of mind to prove pre-statement facts. Cases like this in fact allow such facts to be used (how did Larry know he was meeting Angelothey would have had to have had a discussion prior to the statement). However, remember, the general ruleusing someones statement of intent to prove action in conformity with that intent is perfectly acceptable. However, using someones statement as intent to prove a third partys action is not allowed! Pheester is a narrowly crafted exception.

Evidence Outline

Page 102

FRE 803(3) is intended to overrule Hillmon, partially. However, even under FRE 803(3), Hillmon's liberality in admitting statements proving intent to infer action by someone other than the declarant is still good law today. Limiting Instruction: Larry's statements could be used to prove only what Larry did, but that the jury could infer from that that Inciso was there. So, the limiting instruction does come in to explain how the jury should use the statement only inferentially against Inciso. You should also give limiting instructions to as to proving pre-statement facts.

DIRECT AND CROSS EXAMINATION


I. MECHANICS
OF

DIRECT

AND

CROSS

Litigants ordinarily present testimony by means of direct examination. The lawyer: Develops background information about the witness; Places him at the scene and Establishes the witness has personal knowledge; And in the case of the expert witness: Develops the basis of his expertise Only then does the focus shift to substantive matters Rule 611. Mode and Order of Interrogation and Presentation (a) Control by court. The court shall exercise reasonable control over the mode and order of interrogating witnesses and presenting evidence so as to (1) make the interrogation and presentation effective for the ascertainment of the truth, (2) avoid needless consumption of time, and (3) protect witnesses from harassment or undue embarrassment. (b) Scope of cross-examination. Cross-examination should be limited to the subject matter of the direct examination and matters affecting the credibility of the witness. The court may, in the exercise of discretion, permit inquiry into additional matters as if on direct examination. (c) Leading questions. Leading questions should not be used on the direct examination of a witness except as may be necessary to develop the witness' testimony. Ordinarily leading questions should be permitted on cross-examination. When a party calls a hostile witness, an adverse party, or a witness identified with an adverse party, interrogation may be by leading questions. Direct examination means (for the most part) non-leading questions (FRE 611(c)). (EX: Isnt it true that you were driving 35 mph?)

Evidence Outline

Page 103

The aim is to bring out what the witness has to say, and the attorney should not put words in his mouth or testify for him. But the system is not rigid, and the bar against leading the witness on direct is not absolute. Exceptions to the rule against leading questions: 1. When necessary to develop testimony. (FRE 611(c)) This is normally used when a witness is (a) very young, hence apprehensive, uncomprehending, or confused, (b) timid, reticent, reluctant, or frightened, (c) ignorant, uncomprehending, or unresponsive, or (d) infirm. Of course the danger of leading seems especially great for people of such description, but where the choice is to run the risks posed by this form of examination or to do without the knowledge of such witnesses, the risks become acceptable. These people could be the people most susceptible to be mislead. 2. When the witness is uncooperative. FRE 611(c) also contemplates direct examination by leading questions when the witness is "hostile" or "an adverse party" or "identified with an adverse party." 3. When the Rule is more trouble than it is worth. Sometimes requiring non-leading questions on direct is simply not worth the trouble. On preliminary matters, for example, leading questions save time and are allowed. The same is true of matters that are not contested. 4. When memory seems exhausted. ("refresh his recollection") Usually that means that the lawyer gently reminds the witness of something he has said before, perhaps in a written statement or affidavit or in a deposition, and FRE 612 expressly recognizes this technique. You may allow a witness to look at a document that they would normally be able to see or review before testifying. This could include former statements, depositions, etc. When you refresh the witnesss recollection, you are reminding them of something they know (as opposed to putting words in their mouth). 5. Cross Examination: FRE 611(c)(2) On cross examination, leading questions are allowed, however not when the witness is the attorneys own client (or aligned with her client), as happens when (for example) plaintiff in a civil case calls the defendant to testify and defense counsel then "cross-examines." o It is important to note, that we cannot think of this in mechanical terms it is the attorneys relationship with the witness that determines if we should vary the ordinary rule that direct is by non-leading questions and cross is by leading questions.

Rule 612. Writing Used to Refresh Memory Except as otherwise provided in criminal proceedings by section 3500 of title 18, United States Code, if a witness uses a writing to refresh memory for the purpose of testifying, either-(1) while testifying, or

Evidence Outline

Page 104

(2) before testifying, if the court in its discretion determines it is necessary in the interests of justice, an adverse party is entitled to have the writing produced at the hearing, to inspect it, to cross-examine the witness thereon, and to introduce in evidence those portions which relate to the testimony of the witness. If it is claimed that the writing contains matters not related to the subject matter of the testimony the court shall examine the writing in camera, excise any portions not so related, and order delivery of the remainder to the party entitled thereto. Any portion withheld over objections shall be preserved and made available to the appellate court in the event of an appeal. If a writing is not produced or delivered pursuant to order under this rule, the court shall make any order justice requires, except that in criminal cases when the prosecution elects not to comply, the order shall be one striking the testimony or, if the court in its discretion determines that the interests of justice so require, declaring a mistrial. An adverse party is entitled to have the writing produced at the hearing, to inspect it, to cross-examine the witness thereon, and to introduce in evidence those portions which relate to the testimony of the witness. (One alternative to the FRE 612 argument would be to argue "necessity" under FRCP 26(b)(3).) The Rule is discretionary as to pre-testimony documents. Generally documents used in preparation for trial would be protected under attorney client privilege or work product. Therefore, to overcome this protection, it seems the attorney should have to establish opposing witness relied on documents while giving his testimony or that these documents influenced his testimony. o Congress left the task of deciding whether the privilege is waived for documents reviewed before trial on a case-by-case basis. o Courts must strike a balance between the competing interests of full disclosure and the maintenance of confidentiality for caseby-case determination. Note, at the common law you only waived the privilege if you used the document to refresh recollection at trial. FRE 612 suggests you waive the privilege if you use it for wood-shedding (using it to prepare a witness for trial.) Oral communications will receive the most protection under either attorney-client or work-product privileges. But this is unlikely to be a reasonable substitute in complex, document-intensive cases.

When can a witness use a writing in aid of oral testimony? Witnesses are not supposed to read in court. The witness must testify from the witnesses own contemporaneous recollection of the events. Two exceptions: 1. Refreshing Recollection ex: Ms. Gs house was burglarized. The D has been arrested. Ms. G is the key witness. The Prosecutor asks what items have been taken from your house? Ms. G cant remember (she is a forgetful witness). You could ask a leading question, but if there are a lot of articles that were taken, this probably isnt the best course of action. You can refresh the recollection with a writing. The purpose is to jog the independent memory of the witness so the writing itself does not have to be

Evidence Outline

Page 105

reliableyou can use any writing to refresh the recollection of the witness. However, anything you use to refresh the recollection of the witness, opposing counsel is allowed to see it and use it in cross examination of the witness. 2. Recorded Recollection: the witness cannot remember, even though you have tried to refresh her recollection. In this case, the writing itself will come into evidence. You must lay the proper foundation (1) must show that at one time the witness had personal knowledge; (2) must show the writing was made by the witness (3) writing must be timely made by the witness/when the matter was fresh in the witnesses memory; (4) must show the writing is reliable/accurate; (5) witness must be unable to remember the details of the transaction. The writing itself does not go to the juryit is simply read to the jury. (This is an exception to hearsay) Raytheon The attorney compiled and organized various documents in a binder and showed them to his witnesses before the witness testified. Opposing party then tries to claim that because the attorney showed this information to the witnesses, he has waived the attorney-client privilege. Production of a binder of materials prepared by counsel for the witness for purposes of review in preparation for his deposition testimony. The documents are also shown to the witnesses officers. These documents would ordinarily be subject to the work-product privilege, however is the privilege waived once the documents are showed to other witnesses (in this case Julians officers)? o Arguments FOR disclosure: (1) The documents are not work-product; (2) Even if they are work-product, privileges are waived when documents are used to prepare witnesses to testify, according to FRE 612. Holding: As to the first argument, court holds the documents are work product (FRCP 26(b)(3): The binder contains a small percentage of the extensive documents reviewed by plaintiff's counsel. In selecting and ordering a few documents out of thousands counsel could not help but reveal important aspects of his understanding of the case. Does Rule 612 allow for the waiver of privilege? Court in this case said yes. The waiver is triggered by the use of documents prior to testifying or use during testimony. In this case, the court held, under FRE 612, we may authorize the production of privileged documents. Without reviewing those binders defendants' counsel cannot know or inquire into the extent to which the witnesses' testimony has been shaded by counsel's presentation of the factual background. Plaintiff's counsel made a decision to educate their witnesses by supplying them with the binders, and the Raytheon defendants are entitled to know the content of that education. Not all circuits follow the Raython case. There are courts that hold you need more than just the witness relying on the documents prior to their testimony (i.e.: that

Evidence Outline

Page 106

merely reviewing the documents before trial does not suscept the documents to waiver). Other courts say there must be reliance by the witness and some other independent waiver of the privilege. This court also took an extremely broad view as what was waivedi.e.: the court waived the entire binder. Two arguments to this theory: 1. Courts theory: The court ruling is only defensible if you want to put the proponent in a position to cross-examine the witness in the best way possible. If you believe cross examination is extremely important, then you may see why the court ruled this way. 2. Case-book authors: Malavet believes that the only items which should be waived were the documents relied upon by the witness. Although this court did not require it, Malavet believes the opposing side must make a showing that the witness actually relied on those documents before they become discoverable. He says this rule is problematic because it seems there is a double standard--It is your duty to prepare your client and show them documents (under the model rules), but by showing and preparing your client you risk waiver. This could be a problem for your client. The case book thinks the AT&T case better interprets FRE 612while the mere showing of a voluntary disclosure to a third person will generally suffice to show waiver of the attorney-client privilege, it should not suffice itself for waiver of the work product privilege. II. EXCLUDING WITNESSES

Rule 615. Exclusion of Witnesses At the request of a party the court shall order witnesses excluded so that they cannot hear the testimony of other witnesses, and it may make the order of its own motion. This rule does not authorize exclusion of (1) a party who is a natural person, or (2) an officer or employee of a party which is not a natural person designated as its representative by its attorney, or (3) a person whose presence is shown by a party to be essential to the presentation of the party's cause, or (4) a person authorized by statute to be present. "The Rule on Witnesses": Non-party witnesses should be excluded from the courtroom to prevent them from hearing each other's testimony. (FRE 615) This is so they will not tailor their testimony to what the others are saying. Good cross examination will catch details in the witnesses testimony that are in conflict and within those conflicts, it is a better way to determine the truth. A good attorney will use the contradictions in witness testimony to show the jury these inconsistencies are important: the jury is in a position of telling whether inconsistencies matter. Crime victims are not partiestherefore they can be forced to be excluded. (It is the State or the United States that is the party) How far does the rule extend?

Evidence Outline

Page 107

FRE 615 authorizes an order barring the use of daily transcriptsCases have also construed the rule to authorize orders prohibiting one witness, after completing his testimony, from talking to another witness who is sequestered, support this generous construction. FRE 615 authorizes the court to direct witnesses not to confer privately with one another. It also authorizes the court to direct counsel not to convene meetings with several witnesses at once, in preparation for their testimony. However, it is unlikely the rule allows an order directing counsel not to convey to one witness the substance of testimony given by another. On a theoretical level, Malavet thinks that the court does have the authority to instruct counsel to refrain from conveying the testimony. However such orders would be difficult to enforce.

Susanna and the Elders: (Why we have FRE 615!!) Daniel said: "Separate these men and let me cross-examine them." Question during cross examination: "Where were you in the garden, what kind of tree were you standing under, when you saw them together?" The men both said a different tree The story illustrates why we preclude witnesses from watching each other testify, or perhaps even from speaking to each other until after they testify at trial: because it keeps them from coordinating their stories, or otherwise tailoring their testimony to fit with, or directly to contradict, someone else's. ***Note***: When a witness is found to have lied about something (or to have been mistaken about one fact) that does not require the automatic rejection of their entire testimony. But, catching a witness in a lie is a very effective method of totally discrediting them. Problem 7-A Daily Transcripts Exxon sues Mentor for patent infringement. Norvick is a key expert witness for the defense, and he complies with an order to exclude all witnesses. Just before Norvick is to testify, his attorney purchases transcripts of the trial. Exxon presumes the attorney is going to use these to go over them with the witness and therefore they want to exclude Norvick from testifying. How should the court rule? Witnesses are hereby forbidden from: Listening to another witness' testimony at trial, before they themselves testify, or otherwise finding out the substance of another witness' expected or actual testimony, prior to giving their own testimony at trial. The attorneys are hereby enjoined from assisting a witness in obtaining information in violation of this order. If the order is violated: Exclusion of the witness' testimony is NOT automatic. The alternatives are: -- Exclusion of the witness' testimony altogether, at the discretion of the Court -- The testimony may be admitted, subject to jury instruction that it should be

Evidence Outline

Page 108

weighed in light of their ill-gotten knowledge -- The witness may have their testimony limited. For example, the expert in this case may be precluded from making any use of information obtained from the transcripts as the basis of his opinion, and/or, may not in any way comment on the testimony of other experts given at the trial (note that an expert would be allowed to answer questions such as, "if expert X testified as follows, would you disagree?"). A blanket exclusion order bars use of daily transcripts to prepare witnesses: The plain purpose of the order is to keep a witness from learning (before he testifies) the substance of other testimony in the case. It would be a poor argument to try and argue that the court only forbid counsel from listening to live testimony. Trying to get around the order by showing court transcripts to witnesses who have been ordered to sit out of the court room is cause for disbarment. Counsel is assumed to know the principle behind the rule. The lawyers should not be able to get around the order in any way. Witnesses to the extent they might engage in their own conduct, will not be held to the same standard. A blanket order also applies to experts who would or might be exempt under exception (3). An order to exclude a witness includes: Physical exclusion They cannot discuss with other witnesses what has been said in court The lawyer or some other witness may not be used to describe to the witnesses content of what other witnesses have testified about. Forbidden from reading court transcripts Exclusion NOT Automatic If a witness disobeys the order of withdrawal, while he may be proceeded against for contempt and his testimony is open to comment to the jury by reason of his conduct, he is not thereby disqualified, and the weight of authority is that he cannot be excluded on that ground merely, although the right to exclude under particular circumstances may be supported as within the sound discretion of the trial court. Modern authority makes it clear that violating a sequestration order does not automatically call for excluding the testimony of the witness. Prejudice must be found in order to justify both exclusion from trial or some other sanction, and to justify reversal when a sanction is not imposed. How much prejudice is required? Compare one court who says "prejudice is presumed and reversal is required unless it is manifestly clear that the error was harmless or prosecutor proves harmless error by a preponderance with defendant must show "probable prejudice or an abuse of discretion" when witness violates sequestration order and trial court lets him testify.

Evidence Outline

Page 109

"Essential to the presentation" of a cause (FRE 615[B](3)] Also exempt from exclusion by FRE 615(3) is a person shown to be "essential to the presentation" of a cause. (ex: In criminal trials, investigative agents (such as police) generally qualify for this exception. ) Therefore, if you are a prosecutor of a large complex case, you may have a law enforcement official or someone else that would be allowed to stay in and hear the case to help you and assist you. In other words, if the prosecution needs someone to assist them in the coordination of witnesses, so they can pull police, FBI lab technicians, etc. only when it is absolutely necessary, they will be allowed to stay in the court the whole time. It seems like there are a variety of people who can be certified to stay in court under this provision. It is anyone that is essential to the case to help the attorney in the case. An accountant may also be used in a complex white collar crime. This exception includes an expert or an investigating official Problem 7-B : The Sisters In this case, the women are the daughters of the victim and thus they are neither the "victim" nor the parents of a minor who is a victim. Accordingly, they do not fit within existing Arkansas Rules exceptions. Answer: We find appellant has demonstrated prejudice. It illustrates the need for the witness-exclusion rule to prevent the possibility of any of the victim's daughters from shaping her testimony to that of a preceding witness. The court reversed the judgment and remanded for a new trial. Naturally, at the retrial, everyone will know what to say. So you might ask yourselves if the remedy of remanding for a new trial is in fact adequate. However, because they will be in front of a new jury, the possibility of prejudice wont be as strong. Note, under Utah law, a victim is subject to a statutory exemption from the Rule on Witnesses, i.e., they cannot be excluded from the courtroom during testimony, UNLESS the prosecution does not want them in the courtroom. Why would the prosecutor want to waive the privilege of the witness staying in the court room: 1. So the victim isnt accused of corroborating their story 2. They may become intimidated to go on the stand by watching the case 3. They may want the jury to show their emotion. Examples of statutory exemptions: 18 U.S.C. 3510: provides that federal courts shall not exclude "any victim of an offense" from a trial merely because he may testify during sentencing. Florida goes farther: Fla. Evidence Code 90.616 Exclusion of witnesses. (d) In a criminal case, the victim of the crime, the victim's next of kin, the parent or guardian of a minor child victim, or a lawful representative of such

Evidence Outline

Page 110

person, unless, upon motion, the court determines such person's presence to be prejudicial.

COMPETANCY OF A WITNESS
I. PRESUMPTION
OF

COMPETENCY

Common law rule verses Modern Evidence Rules One of the striking contrasts between the early common law and modern rules of evidence is in the area of competency of witnesses. The common law imposed a number of disabilities that rendered many potential witnesses incompetent to testify in court. Often in fact, the most knowledgeable people could not testify. What used to be an automatic disqualification of a witness now becomes inquiry regarding impeachment. These now become areas you can ask on cross for impeachment purposes. Examples: o mental incapacity, o religious beliefs (limited by FRE 610) o criminal convictions, o interested persons, o spouses Therefore, the following are allowed to be witnesses: o Religious Affiliation does not matter o Infants: There is no set age where a child witness is automatically declared incompetent. o Insane People: as long as the insane witness can give testimony on the narrow issue before the court, they are considered competent on the issue o Conviction of crime: even if it is perjury o Interested parties are not automatically disqualified Presumption of Competency Every witness is presumed competent to testify, under FRE 601, unless it can be shown that the witness does not have personal knowledge of the matters about which he is to testify, that he does not have the capacity to recall, or that he does not understand the duty to testify truthfully. The objecting party bears the burden of proof showing the witness is incompetent. The presumption of competency must be overcome by the objecting party (who is seeking to exclude the testimony.) Three elements that must be shown by the objecting party to exclude the testimony 1. Witness does not have personal knowledge of the matters about which he is to testify (witness observed something)

Evidence Outline

Page 111

2. Witness does not have the capacity to recall (witness can remember what he observed and can communicate what he observed) 3. Witness does not understand the duty to testify truthfully (witness must demonstrate an appreciation to tell the truth (the oath or affirmation)) Under the federal rules, these things boil down to two main qualifications for a witness(1) the witness must have personal knowledge and (2) the witness must take the oath or affirmation. The court decides if the witness is competent (i.e.: whether they are qualified to be a witness) under FRE 104(a). However, the jury decides whether the personal knowledge requirement is satisfied under FRE 104(b). If a court finds that a witness is incompetent to testify and therefore does not allow them to testify, in order to get the decision reversed or a new trial, the party must show there was harmful error!

U.S. v. Lightly Is a person who is legally-determined to be criminally insane qualified to testify? Lightly is being charged with assault on McKinley. The defense attempted to have McDuffie testify. McDuffie would have testified that only he and not Lightly had assaulted McKinley. (this would potentially exonerate Lightly) The trial court ruled McDuffie incompetent to testify because he had been found to be criminally insane and incompetent to stand trial, and was subject to hallucinations. The Appeals court found this to be an error and that Lightly is entitled to a new trial. Excluding McDuffie's testimony was not harmless errorthe potential testimony would have substantially corroborated Lightly's testimony. The testimony of McDuffie's treating physician indicated that McDuffie had a sufficient memory, that he understood the oath, and that he could communicate what he saw. The district judge chose not to conduct an in camera examination of McDuffie. On this record, it was clearly improper for the court to disqualify McDuffie from testifying (without first finding he was incompetent at the time of trial). The court should have made a finding as to whether the witness was competent at the time the witness testified. This must occur when the person is being offered as a witness. However, the court did not engage in finding whether he was competent. Notes: In extreme cases of mental impairment, courts may exclude the testimony as irrelevant under FRE 401; as unfairly prejudicial, misleading, or confusing under FRE 403; as lacking in current personal knowledge under FRE 602; or as contrary to the interests protected by FRE 611(a). When the mental capacity of a proposed witness is questioned, the trial judge does have authority to order a psychiatric examination. However he must use due care.

Evidence Outline

Page 112

II. OATH

AND

AFFIRMATION

As a witness, you must be voluntarily willing and able to take an oath! If you fail FRE 603 (not taking an oath), YOU CANNOT TESTIFY! You are required to take the oath. The purpose of the oath or affirmation: (1) To establish that the witness understands the obligation to tell the truth during testimony and (2) To subject them to perjury prosecutions for failing to testify truthfully. Refusing to take an oath in a case might appear quite self-destructive, and it usually is just plain stupid. However, there may be certain reasons for refusing to swear or to affirm that might make sense, at least to the witness. For example: Some religious persons might refuse to take a religious oath in a non religious context. Other people might object to an oath or affirmation before a particular court for political reasons. U.S. v. Fowler Fowler refused to either swear or affirm that he would tell the truth or submit to cross-examination. Rule 603, Federal Rules of Evidence, is clear and simple: "Before testifying, every witness shall be required to declare that he will testify truthfully, by oath or affirmation . . ." No witness has the right to testify but on penalty of perjury and subject to cross-examination. It is important to recognize in this case the D was a pro-se litigant (Did not have an attorney to explain the importance of an oath) and he was distrustful of government and does not respect the sovereign authority of the state. If you refuse to take the oath, you can be precluded from testifying, and you can also be held in contempt for doing so. Difference between oath and affirmation: (FRE 603) Oath: the traditional saying: So help me God! on the bible, etc. Affirmation: Differs from an oath by eliminating reference to swearing and to divine power. For religious, or non-religious, reasons, you are allowed to take an affirmation. Courts are flexible in allowing a witness to have a subjective understanding that you are willing to tell the truththe specific wording is not provided. Ex of affirmation: I understand that I must tell the truth. I agree to testify under penalty of perjury. I understand that if I testify falsely I may be subject to criminal prosecution. Ex of something that will not be affirmation: I am a truthful man. I will not tell a lie. There is a problem here, because he fails to acknowledge a legal obligation to tell the truth. III. COMPETENCY
OF

CHILDREN

Evidence Outline

Page 113

Children have a Qualified presumption of competence: there is a presumption of competency, but this presumption only kicks in after a showing (in voire dire) that the child is competent. The offering party must justify the qualified presumption showing the child understands their obligation to tell the truth. A lesser showing of capacity is required. In other words, a child is not expected to know what perjury is, however they do need to know the difference between the truth and a lie. After the presumption kicks in, it is now the opposing partys burden to show the child is not qualified to be a witness.

Rickets v. Delaware This is an appeal from a conviction of first degree rape of a five year old girl. The sole issue is whether the trial court committed reversible error in allowing the minor victim, then six years old, to testify without an adequate foundation to determine her competency as a witness. The court found that under Rules 601 and 603 of the Delaware Rules of Evidence, the trial court did not err in permitting the child to testify. Before testifying, a voir dire examination was conducted during which the child stated that she went to church, that a lie was a thing that is not true, and that it was a bad thing to tell a lie. She testified further that if you tell a lie you sometimes get a spanking. She also promised to tell the truth about everything that she was asked in court. However, in response to questions by the defense attorney and the court, the witness indicated that she was not sure what heaven was. The court ruled that the child was competent to testify because, although she did not understand the concept of perjury, she knew the difference between truth and falsehood, which was the only test of competency. This was a sufficient affirmation. The child only needs a lesser showing of competency. An adult would be required to understand perjury, where the child is not as long as the child knows the difference between lying and truth telling. This same standard may be used for adults who have mental incapacity. A number of states do not follow the qualified presumption approach and continue to presume incompetency of children below a certain age. (In NY it is 12) Florida: Florida has adopted the modern view, and has created a thorough scheme to allow children to testify and to protect them when they do. In Florida the judge has an express obligation to protect the child witness. ( Fla. Stat. sec. 92.55) Note that Florida does distinguish between minors (under 18), and children under 16, and children under 14. There is a presumption a child needs protection when they are younger than 14. Sadly, the issue of child competency usually arises in the context of allegations that the child has been the victim of a horrible act either in a criminal prosecution or in child-custody or civil tort litigation. In that context, the testimony may be highly relevant and, at least potentially, highly probative. Here the court has to balance the rights of confrontation

Evidence Outline

Page 114

against trauma to the witness, among other things. Ultimately we have to establish if the child is: (1) describing, as best they can, the horrible acts of which they were victims; (2) reciting what adults intentionally or unintentionally programmed them to say; or (3) describing something they watched on Springer without understanding what it means or the real implications of the testimony The lesser standard we apply to children and the Sixth Amendment: Sixth Amendment entitles, in a criminal case, the D to cross examine the witness against him. However, if a witnesses capacity is lessened (such as a child) then the ability to answer cross examination questions is lessoned. The SC accepts the notion of the reduced capacity for children and has found it to be consistent with the Sixth Amendment. Child witnesses (when they were the victims of sexual assault) may be allowed to testify in ways that reduce the trauma, including not requiring the physical presence of the D and closed circuit testimony. You can see the clear conflict with the Ds Sixth Amendment rights. However we are willing to compromise the Ds constitutional rights as to children. Children deserve a certain level of protection that we do not give other witnesses. We allow a higher level of protection for the child. As to adult witnesses, this SC would probably not find adult witnesses should be allowed to testify under such circumstances. This exception has been limited to children in sexual assault cases. NOTE: The Sixth Amendment only applies to criminal cases. However, in a civil case you would refer to the Due Process Clause of the 5th Amendment, which entitles a Ds ability to cross examine a witness in a civil case. This is not as strict as the Sixth Amendment, but it is still in existence and it is still a right! IV. SPECIAL CASES & JURY MISTAKE/MISCONDUCT Dead Mans Statute: This is one exception where an interested party is automatically disqualified. The statute still exists in about half of state jurisdictions and because it can apply in federal court because of the express language of FRE 601[2] (in diversity suits that use state law/Eerie doctrine) An interested witness cannot testify as to a transaction with a party who is now deceased. Rationale for statute is fear of perjurythe interested survivor cannot testify for his interest against the decedent or the decedents representatives about communications or transactions with the decedent in a civil case unless there is a waiver. Ex: The D dies before trial. The survivor (who is bringing the claim) cannot rectify because we are afraid he will lie because he wont be contradicted. This basically means the Trier of fact can render a decision without hearing either side! Lawyers as Witnesses: This is more of a PR issue than an evidence issue.

Evidence Outline

Page 115

Judges as Witnesses: Somewhat surprisingly, the common law did not consider a judge incompetent to testify in a trial over which the judge was presiding. FRE 605 is explicit in making this one of the few federal grounds of incompetency. Jurors as Witnesses & Jury Mistake/Misconduct Today the issue rarely arises because jurors who might be called as witnesses are usually identified in voir dire and excused from serving. Nonetheless, FRE 606(a) prohibits testimony by that juror before the jury panel on which he serves. Rule 606. Competency of Juror as Witness (a) At the trial. A member of the jury may not testify as a witness before that jury in the trial of the case in which the juror is sitting. If the juror is called so to testify, the opposing party shall be afforded an opportunity to object out of the presence of the jury. (b) Inquiry into validity of verdict or indictment. Upon an inquiry into the validity of a verdict or indictment, a juror may not testify as to any matter or statement occurring during the course of the jury's deliberations or to the effect of anything upon that or any other juror's mind or emotions as influencing the juror to assent to or dissent from the verdict or indictment or concerning the juror's mental processes in connection therewith. But a juror may testify about (1) whether extraneous prejudicial information was improperly brought to the jury's attention, (2) whether any outside influence was improperly brought to bear upon any juror, or (3) whether there was a mistake in entering the verdict onto the verdict form. A juror's affidavit or evidence of any statement by the juror may not be received on a matter about which the juror would be precluded from testifying. FRE 606 flatly prohibits the admission of juror testimony to impeach a jury verdict. ExceptionsA juror can testify to these matters: (1) "extraneous influence" which influenced the jury may be testified to; (2) outside influence or (3) mistake. In such cases, a juror can only testify to the fact certain facts were heard, they cannot testify to the effect these facts had on rendering the decision. If a court concludes there is a showing of one of the exceptions that impacted the jurys decision, there must be a showing of gross incompetency by the jury. In other words, reversible error. If you want to catch juror incompetence, you need to find juror incompetence during the voire dire process. After the jury has returned a verdict, you have to meet the above standard. We entrust to the jury the obligation, power and duty to make factual determinations. We trust them so much, that we are not allowed to get into how they make their decisions. o If misconduct is brought to the courts attention before the verdict is rendered, the court can look into this o Voire dire is the entire scheme to evaluate the capability of a jury.

Evidence Outline

Page 116

We have a public trial. It is very difficult for someone in the court room not to notice the jury if it was a general problem.

Problem 6-A Outside Influence In a criminal prosecution, a juror is caught reading the newspaper which had accounts of the trial and discussed those accounts with others at lunch. The judge also hears that an associate of the defendant talked with the jury outside of the court and offered what may have been a bribe. During a recess in trial, after jurors had been excused, the judge questions the juror about both matters. Does FRE 606(a) bar such inquiry? Answer: This examination of the juror regarding matters that occurred prior to jury deliberations is not barred by FRE 606(a). The witness, even though a member of the jury, is not testifying "as a witness before that jury in the trial of the case in which he is sitting as a juror as long as the testimony is taken without the jury being present." But in any case, the juror may be questioned during the trial regarding jury misconduct and that subject-matter of questioning is not barred by 606(a). Note further that the inquiry is also not barred by 606(b), since 606(b) only kicks in after the verdict is rendered. If the sitting juror's substantive testimony is actually needed, one possibility would be to excuse them from the jury, and allow them to testify, provided that enough jurors remain to render a proper verdict. 606(b) does not forbid the "polling" of the jury, i.e., asking them if the verdict that has been read is in fact their unanimous verdict. But questions after the reading of the verdict generally fall within the language of the Rule, which precludes the official questioning of the jury or use of their written statements. After the verdict is rendered, it is not unethical for counsel informally to interview the jury. You may even want to do so simply to learn what works and what does not. However, such questioning is increasingly being prohibited by statute or local court rules. Tanner v. U.S. Petitioners argue that the District Court erred in refusing to admit juror testimony at a post-verdict hearing on juror intoxication during the trial. Several of the jurors consumed alcohol during the lunch breaks at various times throughout the trial, causing them to sleep through the afternoons. The court is taking as skeptical view of the factsthere are important policy reasons for 606(b). We have a perfectly good remedy. If things are as bad as people say, dont you think someone would have said something DURING the trial and not 5 days after the verdict was rendered! If someone would have brought this to the courts attention during trial, it would have been fine! The jurors could have been questioned under oath, and if believed all of these people were doing drugs or were intoxicated, the court would have declared a mistrial. The District Court concluded that juror testimony on intoxication was inadmissible under FRE 606(b) to impeach the jury's verdict.

Evidence Outline

Page 117

The District Court invited petitioners to call any non-juror witnesses, such as courtroom personnel, in support of the motion for new trial.

The near-universal and firmly established common-law rule in the United States flatly prohibits the admission of juror testimony to impeach a jury verdict. Exceptions to the common-law rule are recognized only in situations in which an "extraneous influence" was alleged to have affected the jury. Allegations of the physical or mental incompetence of a juror is "internal" rather than "external" matters and therefore not admissible to impeach juror testimony. Drugs or alcohol voluntarily ingested by a juror is by no means an outside influence. Allegations of juror misconduct, incompetency, or inattentiveness, raised for the first time days, weeks, or months after the verdict, seriously disrupt the finality of the process. Moreover, full and frank discussion in the jury room is encouragedjurors' willingness to return an unpopular verdict, and the community's trust in a system that relies on the decisions of laypeople would all be undermined by a barrage of post verdict scrutiny of internal juror conduct. Even if the court did conclude this was an external matter, the court did not think the petitioners met the standard. Gross or extreme incompetency (certified insanity): There has to be an extreme showing of gross incompetency by the jury. (Malavet could not even think of something that would fit into this) This is the only exception to 606(b). The allegations did not suffice to bring this case under the common-law exception allowing post verdict inquiry when an extremely strong showing of incompetency has been made. Sixth Amendment Implications: The Sixth and Fifth Amendment give you an entitlement to a rational process. This Court has recognized that a defendant has a right to "a tribunal both impartial and mentally competent to afford a hearing. However the court did not err in deciding that an additional post-verdict evidentiary hearing was unnecessary. The lesson to be learned here is that you must catch these things as they occur. The trial court is much more likely to grant a mistrial under circumstances like this, than to grant a new trial when matters come out after the verdict. That is just the way things are. The Situations From Hell: 1. Tossing the Coin: Jury flips of a coin to decide the verdict (manner of reaching a verdict, therefore jury cannot testify to this after the verdict has been rendered) A strong argument can be made that even the toss of the coin is a "manner of reaching the verdict" question and, thus, that inquiry into it by juror testimony or affidavits is barred by FRE 606(b). Commentators have argued that such a method of reaching a verdict constitutes a "denial of a rational process" and is thus subject to inquiry,

Evidence Outline

Page 118

and that flipping a coin is arguably the introduction of "extraneous" information into the jury room. But, scary as this thought is, Malavet thinks that the express language of the Rule may in fact preclude jury testimony or affidavits to prove this, although I certainly think that any rational court would grant a new trial upon hearing of such an occurrence. 2. Racisim: Some courts have ruled that bias cannot be explored through postverdict juror testimony or affidavits, but others hold that bias can be considered an "outside influence" and thus properly inquired into under FRE 606(b). You cannot use jury testimony after the verdict to show someone was racist. However if after trial, someone does acts that shows he is a racist, you can use this as evidence to show a jury member lied during voir dire and this will be grounds for a new trial. 3. Knowledge which is not within the common knowledge of jurors. Extraneous prejudicial information. When one juror knows something other jurors do not. Grounds for reversal of the jury verdict that fall completely outside of FRE 606(b): Failure to follow instructions, although this cannot be proved by jury testimony or affidavits, but may be inferred by the court in granting a new trial motion. (See problem C) Knowledge and opinion which are beyond the lay knowledge and considered expert. This is not acceptable knowledge in a jury room. Problem 6-B Refusal to take the stand Atkins is convicted of unlawful possession of narcotics at a trial in which he did not testify to his own defense. A week after trial, the judge receives a letter from a juror that says the jury violated the judges instructions because it considered Adkins refusal to take the stand as an admission of guilt. May the juror be called to testify on the letter? Answer: Under both prior federal law and FRE 606(b), evidence is generally excluded that one or more jurors ignored or misunderstood the instructions of the court. Courts have also refused to receive evidence that one or more jurors held it against the accused that he failed to take the stand. This is internal to the jury. The letter also may not be received. The last sentence of FRE 606(b) excludes affidavits or other evidence of any statement by a juror about which he would be precluded from testifying. However, if other evidence shows that jurors ignored the instructions, then that may provide the basis for reversal. You cannot admit internal views of the jury after the verdict has been rendered. It is important to realize that if this information would have been determined and brought to the courts attention prior to the rendering of the verdict, the judge could have questioned the jury about it and possibly declared a mistrial. Problem 6-C The $800,000 Jury Error

Evidence Outline

Page 119

The instruction as it should be understood: Damages are the difference between fair market value of the farm and the redemption cost plus the mortgage balance on the property. ($90,000) The instruction as the jury may have understood it: Damages are the difference between fair market value of the farm and the redemption cost, plus the mortgage balance on the property. ($890,000) Upon seeing the jury interpreted the instructions wrong, the attorney obtains affidavits from the jurors saying they misunderstood the instructions. Answers: 1. If the judge agrees that the jury erred, should she grant a new trial? YES. 2. Does FRE 606(b) allow such affidavits? NO! The affidavits of the jurors arguably describe "the effect of anything upon his or any other juror's mind or emotions as influencing him to assent to or dissent from the verdict or indictment or concerning his mental processes in connection therewith," which is prohibited by the rule. Jury miscalculation of damages or misunderstanding of the court's instructions is generally considered to be the type of error that cannot be challenged under FRE 606(b). For jury errors of this type, it is sometimes possible to sidestep the proscription of FRE 606(b) and have the judgment reversed on other grounds, such as insufficiency of the evidence to support the verdict. WAY TO CORRECT THIS INJUSTICE: Note that while you cannot ask the jurors about misunderstanding the instructions, you can nonetheless order a new trial based on a finding that the jury clearly failed to follow its instructions, (this cannot be proved by jury testimony or affidavits, but may be inferred by the court in granting a new trial motion) or order remittitur on the ground that the award is excessive. Do not confuse the language in Rule 608 that says whether there was any mistake in entering the verdict into the verdict form with this. This only applies to where the jury for example found a defendant not liable but wrote liable on the verdict form. In this case, the jury meant $890,000 and wrote $890,000. Misconstruing jury instructions does not fall into this exception. PROBLEM 6-D: The Jury View In a personal injury action arising out of a car accident, a verdict is returned for the P. After trial, counsel of the D finds out the 2 jurors went to the accident scene on a fact finding mission. At a hearing for a motion for a new trial, may the defense attorney make inquiry of the two jurors regarding this report? Answer: Evidence about unauthorized jury views is generally allowed [i.e., it is not barred by FRE 606(b)]. Unauthorized views fit the exception to the rule for "extraneous prejudicial information [that] was improperly brought to the jury's attention." Factual knowledge was not acquired during the receipt of evidence in the court room is extraneous prejudicial information and you may therefore use juror testimony to establish that testimony was received by the jury.

Evidence Outline

Page 120

You can use this information to show the jury received prejudicial information. However you cannot use jury testimony to establish the effect this extraneous prejudicial information had on the jury. Before you can reverse this ruling, you still need to establish if the information rose to reversible error. The court still must make a finding that the case resulted in a different verdict in order for the verdict to be reversed.

PROBLEM 6-E: The Bomber Jones is convicted of detonating an explosive device in a public building. Afterwards, her lawyer is told that one juror is willing to testify that another informed the jury he was a demolitions expert in the Army and that the type of bomb Jones used was powerful enough to kill anyone within 20 feet, even though no injuries were inflicted by the explosion. If Jones moves for a new trial, may she offer such [juror] testimony in support of the motion? Answer: YES If so, may the prosecutor call other jurors to testify that this information had no influence on their votes? NO. Court must Balance "extraneous prejudicial information" and "outside influence," which are bad, against "some sharing of personal knowledge and experience by jurors [which] is expected and proper". The answer depends on how directly the evidence bears on the immediate controversy and how prejudicial the information is in the context of the particular case. Courts are much more likely to admit evidence regarding information which pertains to the immediate parties or controversy. This opinion, which is highly specialized in nature, and is of sufficient prejucide to justify receipt of juror testimony regarding what was saidNOT the effect of what was said. However, although the information is highly prejudicial, it is unlikely this was reversible error. It most likely did not ultimately change the outcome of the trial Problem 6-F The Peacocks Tale Brown, a part-time tax preparer, is charged with preparing and presenting fraudulent income and tax returns on behalf of his clients. At trial, the government offers testimony of an IRS agent named Peacock that she audited 160 returns prepared by Brown and that 95% of them contained overstated deductions. She based this information she received from people for whom the returns were prepared. Is her testimony objectionable? If the testimony is being offered substantively, then there are two types of Personal Knowledge required: (1) Personal knowledge that the statements were made or (2) Personal knowledge of the truth of the contents of the statements. If the testimony is being offered for a non-truth use, the only personal knowledge you need is that the statements were made. Court ended up excluding the testimony saying it was hearsay It can be argued that the court reached the right result for the wrong reason. A better rationale for the opinion might be that the witness lacked the

Evidence Outline

Page 121

personal knowledge necessary to support her testimony. (FRE 602) Here we need the substantive contents of the statements, which is the personal knowledge of the clients, not the IRS agent. The prosecutor should have just brought in the actual clients who had personal knowledge that this happened. If we classify her as an expert (under FRE 703), this would be admissible under her opinion. If she were an expert, she could rely on the facts, such as out-of court statements of tax payers if they were found to be of a type reasonably relied on by experts in that field.

IMPEACHMENT
I. IMPEACHMENT GENERALLY Note: Impeachment is not entirely covered by the FRE. In these questions, first make reference to FRE 404(a) and 405 (character evidence). Then you may move on to other rules! Methods of Impeachment: 1. The prior inconsistent statement: You may use a statement made by the witness on a prior occasion which is different from or inconsistent with a material issue of the witnesses present in court testimony. This statement does not come in for its truth. It only comes in to impeach. We get around the hearsay rule, by saying it is not hearsay because we are only offering it to show the contradiction. This witness should not be believed. The prior inconsistent statement cannot be used to make a prima facie case, or be used to help you meet your burden of productionit serves only to destroy. Except: if it is a prior inconsistent statement that was given under oath and as part as a formal trial hearing, proceeding or deposition, than this statement is admissible to impeach and for its truth. This is an exception to the hearsay rule. You can use extrinsic evidence to show the prior inconsistent statement, however it is likely, you have to lay a foundation and give the target witness the opportunity to explain or deny making the inconsistent statement. A prior inconsistent statement by a PARTY always comes in for its truth because this is considered an admission. P sues D alleging he was speeding. D takes the stand and testifies he was only going 15 mph. The officer says the D told him he was going 70 mph. This could not only come in to impeach but it also comes in for its truth. A prior inconsistent statement by a party is an admission and there need be no foundation whatsoever.

Evidence Outline

Page 122

2. A showing of interest or bias or motive to misrepresent or exaggerate Showing that the witness has some bias, animus, motivation, or corruption that might lead him to fabricate or shade his testimony to help or hurt one of the parties. You can prove this by extrinsic evidence, but you do have to lay a foundation to admit or deny the facts that constitute the interest, bias or motive. Almost always you can use this to show an expert witness is being paid by the party who called him (this doesnt mean he is lying, it is just letting the court know that he is not a neutral expert.) In settlements this rule could be used as well. (ex: there were two people injured in a car crashA and B. B settles with the D but A does not. Now B is testifying for D. The jury is entitled to know that D has paid B in a previous settlement). It is relevant to impeach the credibility of the witness. 3. Prior commission of a crime Under FRE, there is 3 parts Any crime involving dishonesty or false statement is usable to impeach. FRE 403 does not apply. The court does not have the discretion to say I am not going to allow the impeachment because it is substantially outweighed by unfair prejudice. The crime has to have an element of deceitlarceny by trick, embezzlement, perjury, etc. A felony that does not involve deceit of false statement (like robbery or murder) These are useable to impeach but there is discretion under FRE 403. Misdemeanors are not usable at all! (FRE 609) Any crime which is revealed cannot be too remote. (Ex: if 10 years have passed, it is too remote) 4. Specific acts of deceit or lying (NOT convictions) on cross: These acts did not result in convictions. These may be inquired into on cross examination of the target witness. (ex: Did you file a fraudulent income tax return? Did you lie on your application for life insurance application?) However, you have to have a reasonable basis for asking the question it must be asked in good faith with a reasonable belief it was done by the witness. NO extrinsic evidence is allowed. You must take the answer of the witness. (FRE 608(b) 5. Bad reputation or opinion for truth and veracity: (credibility) Extrinsic Evidence is allowed-you can call the community mouth to the stand who can testifies they know the reputation of the witness and it is terrible therefore the witness should not be believed. (FRE 608(a)). 6. Mistake: showing a defect in sensory or mental capacity (perception or memory) that undercuts his testimony.

Evidence Outline

Page 123

7. Contradicting the Witness: showing that he is just plain wrong on a point of testimony (FRE 613(b)) (FRE 609) Types of Impeachment: 1. Definite: there is a particular definitive reason to doubt the witness. (Bias/Corruption, Mistake/Defect in mental capacity, a prior non-conviction, prior crime, bad reputation) These methods are definite in telling the Trier why to doubt the witness, but not specific in not showing what testimony to doubt. 2. Specific: the impeachment brings into question specific parts of testimony (Prior inconsistent statements and Contradiction) These methods are specific in casting doubt particular parts in the testimony of the witness (hence suggesting the possibility of error or falsehood on other points) but indefinite because they do not necessarily reveal an underlying cause. Cross examination and impeachment 1. You have a right to some opportunity to cross examine any witness who testifies live against you. (Sixth Amendment or Due Process Clause of the 5th Amendment) 2. Under the FRE, cross examination is limited to the scope of direct. It cannot exceed the scope of direct. Therefore you can cross on any issues that were raised during direct examination. 3. The collateral matters doctrine (impeachment by contradiction)--The cross examiner is bound by the answers he gets from a witness as to collateral matters. No extrinsic evidence is allowed to contradict the witness as to matters which are only relevant to show contradiction. In these cases, the only relevance of the testimony is to contradict the witness. Ex: a witness testifies against my client claiming he was present on the scene at the time of the accident. However, he lied under oath about where his g-ma lives. There are 23 witnesses that are willing to testify the g-ma does not live there. However, you cannot produce these witnesses, because extrinsic evidence is not allowed to contradict a collateral matter. Where the gma lives has nothing to do with the case. In other words the probative value is substantially outweighed by judicial efficiency (the waste of time of hearing the evidence) REMEMBER, if you label something as counterproof because it not only has an impeaching effect but it has some relevance independent of its contradicting effect that you can use extrinsic evidence. Repairing Credibility: What can you do to increase the credibility of your witness if there has been an attack? Usually the adversary of the attacking party has an interest in repelling the attack or otherwise repairing the credibility of the witness. Under FRE 611, the supporting party may examine the witness in an effort to refute points suggested during the attack. 1. Good Reputation for Truth and Veracity: this is mounting an offensive of your own. Under certain conditions, you may offer

Evidence Outline

Page 124

proof of the good character of the witness. However, there must be a direct character attack on the witness (i.e.: choices 3, 4, and 5) 2. A return to the prior consistent statement: if there has been an appropriate impeachment attack, you can use a prior consistent statement by your witness. You can only show a prior consistent statement to rebut a charge of recent fabrication or improper influence or motive. The rules make no mention of bias or mental or sensory capacity, and no mention of contradiction in repairing credibility.

Credibility of your own witness You cannot bolster the credibility of your witness before there has been an appropriate impeachment attack. In this area, bad must come before good. We assume witnesses are credible until there has been an impeachment attack. You can impeach your own witness! The credibility of a witness may be attacked by any party, including the party calling the witness. II. A SHOWING
OF

INTEREST

OR

BIAS

United States v. Able The SC held Abel guilty for bank robbery claiming that the District Court did not improperly admit testimony which impeached one of respondents witnesses. Abel and two cohorts were indicted for robbing a savings loan. The other 2 cohorts pled guilty, but Abel went to trial. One of the cohorts decided to testify against Abel and identify him as a participant in the robbery. Abel planned to counter Ehles testimony with Mills testimony. Mills had spent time with Ehle and Abel in prison and was prepared to testify that Ehle had told him that he was going to frame Abel. Prosecution tried to counter Ds argument by saying that Mills, Ehle and Abel are all part of the Aryan Brotherhood which promotes illegal activity (such as perjury, theft and murder on each members behalf.) D objects to this testimony saying it was too prejudicial to Abel. The court allowed the Prosecutor to ask Mills about the gang, and if Mills denied it, that he could bring Ehle back on the stand concurring the gang and Mills membership in it. Court said the probative value outweighed its prejudicial effect. Procedural Posture: 1. The District court got an evidentiary proffer from both sides (allowed the sides to present their side of admitting/not admitting the evidence re association with the Arian brotherhood) away from the jury. The court concluded they would allow the organization to be admitted however the parties could not refer to it as the Arian Brotherhood (instead they call it a Secret Prison Organization). Using the name of the organization was unduly prejudicial. D did not request a limiting instruction under FRE 106.

Evidence Outline

Page 125

The only reason the testimony is allowed is to impeach Mills. (To show that he has a bias to lie because it is a major tenant of the organization to which they are all apart) o It is not for the truth of the matter asserted (Abel is a member of the gang which commits robberies and therefore he robbed the savings account). o It is not being offered to show a bad person inference (Abel is a member of a bad organization, is a bad person and therefore robbed the savings account_ 2. The Ninth Circuit said this is too much of a bad person inference and is too prejudicial to the D. This is guilt by association--because D is a member of the Arian Brotherhood, a criminal gang he is a bad man and is guilty. This is clearly improper under FRE 608(b). It is settled law that the government may not convict an individual merely for belonging to an organization that advocates illegal activity. The government must show that the individual knows of and personally accepts the tenants of the organization. It establishes nothing about the individuals own actions, beliefs, or veracity. At the very least, Abel is being connected to Mills and Mills is shown to be a bad person because of membership in this organization without proof that he actually adopted the organizations tenants. 3. S.C agrees with District Court- the common membership in such a bad group is impeaching by itself. It is legitimate to show that Mills has a bias we have a HIGH PROBATIVE VALUE and relatively low unfair prejudice. These members had to lie on behalf of one another or it is suicide. The D is still entitled to a limiting instruction if he wanted one, but in this case, the D did not request one. Notes: Impeachment use of a witness prior statement is deamed to be a non-truth use of the statement and is not hearsay under FRE 801(c). There must be careful process prior to the admission of evidencelistening to the parties offer proof outside the juryediting out the name of the organizationthe fact D did not ask for a special instruction, etc. FRE 608(b) (the rule that allows a cross-examiner to impeach a witness by asking about non-convictions) could not be used here. This rule is limited to cross examination of the witnesses, and prohibits the use of extrinsic evidence. Malavets forest with the case: Step IRelevance: Bias impeachment makes this relevant. Although there is no express rule which discusses bias impeachment, long established common law tradition allows it. A successful showing of bias on the part of a witness would have a tendency to make the facts to which he testified less probably in the eyes of the jury than it would be without that testimonyit is therefore relevant. Proof of bias is almost always relevant for this reason.

Evidence Outline

Page 126

Step 2Specific Rule of Prohibition: There is no rule of prohibition that prevents you in the bias impeachment. Although you do have FRE 404 and FRE 608 concerns (bad person inferences), bias is not explicitly forbidden by these rules and therefore you do not have to prohibit the evidence. o Way you get around 608(b): Membership is not a specific instance of conductby not requiring adoption of the tenants of the organization, the court does not require specific instances of conduct. All we are showing is membership. o Way you get around 404: We are not using this to implicate the witness or D is a bad personwe are using it to show bias. Step 3403 Balancing Test: there is probative value here, because the tenants and views and rules of this organization matter in this casethe rules explicitly say the members should lie, cheat and steal on behalf of one another. Therefore membership in this organization is HIGHLY PROBATIVE: this witness may lead the jury to slant, unconsciously, in one way or another and is therefore very probative. Membership in an organization to show Bias We allow a jury to infer common membership in an organization, even without proof that the witness has adopted it tenets is certainly probative of bias. The type of the organization in which a witness and a party share membership may be relevant to show bias. If the organization is a loosely knit group having nothing to do with the subject matter of the litigation, the inference of bias arising from common membership may be small or nonexistent. Being a member of the Book of the Month club would not have inferred bias in Abel. However, the tenants of the Aryan Brotherhood showed that Mills had a powerful motive to slant his testimony towards the D. The attributes of the Arian Brotherhood bore directly on the facts of bias but also on the source and strength of that bias. The tenants of this group show that Mills had a powerful motive to slant his testimony towards respondent, or even commit perjury outright. Problem 8-A The Hired Gun Riley, was an expert in a trial. The Plaintiff kept trying to show that Riley was being paid by GM and therefore was somehow biased in a way. He keeps asking how much Riley is going to get paid. Ds counsel says: Your Honor, we have nothing to hide here, but plaintiff's counsel clearly wants to browbeat this witness. The professor has said how much he gets paid, and it's a lot of money because he's a highly trained expert. There's no reason to go into great detail here. It just wastes time and distracts us all from what's really at stake here. Ps counsel says: Your Honor, the jury should know how much this man expects to get paid. I want them to know some other things too, including (1) how much he made testifying for GM last year, (2) whether he expects to testify for GM again, (3) how much he made, all told, in court appearances last year testifying for automakers, and (4) approximately what proportion of his total income comes from such appearances.

Evidence Outline

Page 127

Answer: These questions go both to the sources of income and to your party affiliationsources of income and allegiance affiliation. Income related to the case in any way and affiliations to any party are the legitimate subject of bias impeachment cross examination questions and are therefore relevant. On these facts, we would say plaintiff should be able at least to ask whether Riley has a larger economic interest in testifying well for GM, and whether he has regular clients whose positions he tends to support. The Graham Study shows Courts allow questions about: (a) continuing employment by the calling party and (b) prior testimony for the same party or attorney, But not those about: (c) previous compensation from the same party or attorney on other cases, (d) the proportion of the expert's total income which comes from testifying for a party or type of party, or (e) the degree to which the expert limits his testimony to certain causes or certain sides of issues. More recent studies (and Malavets experience) shows the last three things would be allowed as well. The federal courts tend to be more liberal than state courts and state courts are very divided. o FRCP 26(a)(2)(B) reads that that experts shall disclose the compensation to be paid and a listing of any other cases in which the witness has testified as an expert within the four proceeding years. This establishes a strong preference in the federal rules for extensive bias/interest in impeachment of experts. o Under Florida, Syken v. Elkins, gives a list of questions that are admissible. They limit the questions to these: 1. The medical expert may be deposed either orally or by written deposition. 2. The expert may be asked as to the pending case, what he or she has been hired to do and what the compensation is to be. 3. The expert may be asked what expert work he or she generally does. Is the work performed for the plaintiffs, defendants, or some percentage of each? 4. The expert may be asked to give an approximation of the portion of their professional time or work devoted to service as an expert. This can be a fair estimate of some reasonable and truthful component of that work, such as hours expended, or percentage of income earned from that source, or the approximate number of IME's that he or she performs in one year. The expert need not answer how much money he or she earns as an expert or how much the expert's total annual income is. 5. The expert may be required to identify specifically each case in which he or she has actually testified, whether by deposition or at trial, going back a reasonable period of time, which is normally three years. A longer period of time may be inquired into under some circumstances. 6. The production of the expert's business records, files, and 1099's may be ordered produced only upon the most unusual or compelling circumstance. (only if the court is skeptical as to what the expert is saying) 7. The patient's privacy must be observed.

Evidence Outline

Page 128

8. An expert may not be compelled to compile or produce non-existent documents. Florida also allows you to inquire into the relationship between the expert and a party because it goes to the witnesses potential biasness. Malavets Examples from other jurisdictions: Fifth Circuit: (liberal) cross examination of an expert about fees earned in prior cases is not improper Third Circuit: The expert worked for the government in condemnation proceedings for the government. District court did not permit into evidence the experts contract with the government as an appraiser for a project or the amount received for the appraisal work. Illinois: counsel could inquire into an experts financial interest in the case including (1) in the particular case (2) for a particular party or (3) for a particular parties attorney. Maryland: favors a liberal approach to include questions regarding testimony in other cases and patterns of payments received from particular parties. III. MISTAKESENSORY CAPACITY
AND

PRIOR BAD ACTS

Sensory and Mental Capacity: the attacking party may seek to show that a witness had only a brief chance to see or hear what she has described in her testimony, or that she labors under defects in sensory capacity that may affect her observation, or that human perceptive processes work in ways suggesting that her testimony is not so persuasive as it seems. These points may be proved by extrinsic evidence. Ones psychiatric history can only be invaded on cross-examination when required in the interest of justice. IV. ADMITTING NON-CONVICTIONS--TRUTH
AND

VERACITY

Three means of proving untruthfulness through character evidence: 1. Cross examination on non-conviction misconduct (FRE 608) 2. Cross examination on convictions (FRE 609) 3. Use of Character witnesses (FRE 608) Cross Examination on Non-Conviction Misconduct The following questions have all been approved by appellate opinions: 1. Isnt it a fact that you lied on two employment applications 9 years ago when you replied no the question whether you committed a crime? 2. Werent you involved in persuading ineligible voters to fill out false registration forms? Didnt you even steal forms for them to fill out? 3. Didnt you give false information on a bank loan application and on your tax returns 4. Isnt it a fact that you used false names? 5. Have you ever made an oath accepting Satan as your God? 6. Havent you accepted bribes in the context of your official duties?

Evidence Outline

Page 129

Remember, in cross examination, the question does the real damage! (Also remember, courts must have adequate basis to ask these questions.) Regardless of a denial, it seems the odor raised by the question will still resonate with the jury. Note, modern views suggest that even if the cross-examiner has a factual basis for these questions, they can be damaging beyond their power to shed light on veracity. (See FRE 608b) Modern cases disapprove of this type of cross examination about behavior that does not directly involve lies or deception. United States v. Manske Manske wants to cross examine Penske (an alleged co-offender) who testified for the government about threats Penske made to at least 5 different people before they testified. Manske wants to show that these threats show only a propensity for violence, rather than untruthfulness. The government makes a motion in limine to block the proposed cross-examination. The trial court grants the motion saying violence is not probative as to truth telling. Two arguments were being made here (1) the testimony shows bias and (2) Penske is untruthful. The court deals with the first argument. Relevance: By showing the witnesses were afraid for their lives and that is why they testified in a certain way is clearly bias! Clearly, the witnesses were biased against the Ds because they feared for their personal safety. Bias may show fear of the witness for his or her personal safety or the safety of friends and family. Therefore this information is relevant. Generally, however, witnesses specific instances of conduct (threats made) may only be raised on cross-examination if they are probative of truthfulness or untruthfulness. There are three ways to interpret this rule: o Broad: anyone who does anything bad is a liar. Virtually any conduct indicating bad character indicates untruthfulness, including robbery and assault. Any criminal conduct says something about whether you have the capacity to tell the truth. Court throws this out because it must be character for truth telling. o Narrow: a crime bears only on veracity if it involves falsehood or deceptionif it can readily be determined from the elements of the offence that it involved truth telling (ex: fraud, perjury, forgery, etc.) Court says the language of 608(b) is not this narrow. o Middle view: Merely being violent is not enough. You must be violent to elicit pergery or lying. There must be a link between truth telling and acts of violence as long as the violent acts were intended to make people lie. Acts of violence which go to concealing or frightening off witnesses or suborning perjury. This threat clearly implicated Penskes truthfulness. Note: It is questionable if the only argument in this case dealt with Penskes threats and whether this showed his truthful disposition, if this testimony would have been allowed under FRE 608(b). V. PROVING PRIOR BAD CONVICTIONS

Evidence Outline

Page 130

Rule 608. Evidence of Character and Conduct of Witness (b) Specific instances of the conduct of a witness, for the purpose of attacking or supporting the witness' character for truthfulness, other than conviction of crime as provided in rule 609, may not be proved by extrinsic evidence. They may, however, in the discretion of the court, if probative of truthfulness or untruthfulness, be inquired into on crossexamination of the witness (1) concerning the witness' character for truthfulness or untruthfulness, or (2) concerning the character for truthfulness or untruthfulness of another witness as to which character the witness being cross-examined has testified. Extrinsic evidence related to a prior non-conviction offense would not be allowed at trial to be heard by the jury, but it could be used to lay the foundation for the questions in a judicial mini-hearing before the judge. While extrinsic evidence of prior bad acts is inadmissible, it might become admissible for impeachment-by-contradiction evidence if the witness is asked the question during trial, and denies some fact established by the documents. FRE 608(b)(2): Note that this rule constitutes a very narrow exception to the general rule that forbids the use of extrinsic evidence regarding character.

Rule 609. Impeachment by Evidence of Conviction of Crime (a) General rule. For the purpose of attacking the character for truthfulness of a witness, (1) evidence that a witness other than an accused has been convicted of a crime shall be admitted, subject to Rule 403, if the crime was punishable by death or imprisonment in excess of one year under the law under which the witness was convicted, and evidence that an accused has been convicted of such a crime shall be admitted if the court determines that the probative value of admitting this evidence outweighs its prejudicial effect to the accused; and (2) evidence that any witness has been convicted of a crime shall be admitted regardless of the punishment, if it readily can be determined that establishing the elements of the crime required proof or admission of an act of dishonesty or false statement by the witness. Prior conviction of a witness o A prior conviction of a crime (punishable by at least a year) by a witness will be admitted subject to the FRE balancing test This standard favors admissibility, since exclusion is warranted only if the risk of prejudice substantially outweighs probative worth). o A prior conviction of a crime by a witness if the crime is one of falsity or dishonesty will be automatically admitted. The automatic admissibility rule has to be narrowly construedor it would become an exception that swallows the rule. Crimes of theft, violence, etc. do not involve dishonesty or false statement within the meaning of the rule.

Evidence Outline

Page 131

Dishonesty or falsity has to be an element of the crime. Once you say something fits under FRE 609(a)(2), it is admissible. You do not need to go through a 403 balancing test. The rule applies to the D in a criminal case, a Ds witness and a Ps witness. It does not matter. Prior conviction of the accused: o A prior conviction of a crime by the accused shall be admitted subject to the reverse FRE 403 balancing test. This standard favors inadmissibility, since it allows such use of convictions if the probative value substantially outweighs the prejudicial effect. The burden is on the government to show that the probative value of a conviction outweighs its prejudicial effect to the defendant. Therefore, you must find a SUBSTANTIAL PROBATIVE VALUE. The accused opens himself to attack on convictions if he takes the witness stand. This is an exception to FRE 404 that the character of the accused may only be admitted to rebut good character. Remember, the value you give to the prior convictions are relative depending on which witness you are dealing with. 1. If it a felony conviction by a D himself : The D in a criminal case gets the highest benefit of the doubt and therefore unfair prejudice is very important. Unfair prejudice is a specific concern and is more likely to outweigh the probative value because of spill over, etc. 2. A Ds witness: The Ds witnesses in a criminal case are in the middle. This is a more compelling argument to the D witnesses when compared to Ps witnesses, because of things like the spill over effect. 3. Ps witness: Prosecution witnesses in a criminal case are not as unfairly prejudicial: least compelling of the three. Almost all courts agree that the cross-examiner may not go into details of the underlying offense. You are only allowed to ask about the fact of conviction, the name of the crime, the date and the sentence. Courts may restrict the cross-examiner even further. This is to watch for the unfair prejudice that may occur. Side note: A D or witness in Florida may not be impeached on a previous plea agreement, or even a verdict of guilty, when adjudication has been withheld.

Think of FRE 609 as a sliding scale of admissibility with regard to truth telling. The more a prior crime shows truth telling, the more probative value it has. Therefore, perjury will have much more probative value than a violent crime. 0%<--------------------------------------------------------------------- (Violent act) (Perjury)

Evidence Outline

Page 132

Murphy v. Bonanno Brian Murphy was charged and convicted on two counts of unlawful assault and one count of unlawful entry, which convictions were affirmed on appeal. Brian appealed claiming the trial judge erred in refusing on relevance grounds to permit cross-examination of Murphy about several prior instances of asserted conduct reflecting poorly on her veracity as a witness. 1. Murphy falsified financial statements 2. Murphy filed false and fraudulent claims against defendants 3. Murphy filed false claims of sexual harassment A witness may be cross-examined on a prior bad act that has not resulted in a criminal conviction where . . . the bad act bears directly upon the veracity of the witness in respect to the issues involved in the trial." (FRE 608(b)) The second prong of this test asks whether the prior bad acts are probative of truthfulness or untruthfulness. Conduct amounting to "making false statements of a variety of kinds" is "probative of truthfulness or untruthfulness" within the meaning of FRE 608(b). Evidence that Ms. Murphy filed false insurance claims would be admissible under this principle as well. The use of extrinsic evidence and character under Murphy Elizabeth Murphy is the Principal Witness, i.e., she is testifying about ultimate facts that are important to Plaintiff Bonanno's case. Brian Murphy is called by defense counsel to testify regarding the character of Elizabeth Murphy, in order to attack her credibility. In his direct testimony he could testify as to her character, based on opinion or reputation, as allowed by FRE 405(a) and FRE 608(a), but he would not be allowed to testify about the three specific instances of bad acts that are the object of the case, because that would forbidden extrinsic evidence under the initial language of FRE 608(b). However, Plaintiff Bonanno's lawyer, under FRE 608(b)(2) and 405(a)(2) could cross-examine Mr. Murphy and attack his character opinion by using specific instances of (presumably good) conduct that support Elizabeth Murphy's credibility. Additionally, say that after Brian Murphy testifies, Bonanno's lawyer wants to support Elizabeth Murphy's credibility by calling Bonanno to testify about her good character. Opposing counsel, on cross-examination, likewise under FRE 405(a)(2) and 608(b)(2), could bring out prior bad acts by Elizabeth Murphy to attack Bonanno's opinion on the character of the Principal Witness. United States v. Liscomb: Liscomb was convicted for possession of heroin. Liscomb testified in his first trial and had been impeached by cross-examination concerning his conviction for robbery 8 years earlier. Liscomb made a motion in limine to prevent this crossexamination on retrial. Under the rule 609(a)(1), the judge said the P could ask about the Ds convictions. Therefore, Liscomb did not testify. The judge also allowed the prior convictions of three Ds Little (convicted of robbery), Smith

Evidence Outline

Page 133

(convicted of robbery) and Green (convicted of accessory after the fact to manslaughter). How should the balance be performed? Rule 609(a)(1) is broadly phrased to require balancing of probativeness against prejudice, with no specific instructions as to how the balance is to be performed. Rule 609(a)(2) creates a per se rule that probativeness outweighs prejudice for crimes "involving dishonesty or false statement." Often, however, the trial judge will not be able to determine from the name of a crime whether the defendant's conduct involved dishonesty or false statement. All courts have held that the prosecution may adduce specific facts to bring a prior conviction within Rule 609(a)(2). It seems equally appropriate to permit the district court to elicit such facts in balancing probativeness against prejudice under Rule 609(a)(1). Robbery is not one of those crimes that will fall under automatic inadmissibility under 609(a)(2). However, it may be brought in under 609(a) (1). When you are dealing with felony crimes, the court may look into the underlying facts of the conviction used in making its assessment about its probative value. Lipscomb gives courts the authority to look at the underlying facts of the conviction in order to establish how much the probative value the conviction has Robbery does have some unfair prejudice (but not nearly enough as a sexual crime would have). It would most likely not make the jury red. Therefore, it is likely these prior convictions would be admitted. Where the formal title of an offense leaves room for doubt, automatic admissibility under Rule 609(a)(2) will normally not be permitted, unless the prosecution first demonstrates to the court, outside the jury's hearing, that a particular prior conviction rested on facts warranting the dishonesty or false statement description. How much Background Facts Are Allowed? Rule 609 allows the court to inquire into the background facts and circumstances, but need not always do so. Because the Rule does not indicate when the district court should seek this additional information, the court has discretion to decide when it should do so. The court has discretion to determine when to inquire into the facts and circumstances underlying a prior conviction and how extensive an inquiry to conduct. The simple or basic inquiry is into the name and date of the conviction. Beyond that, we get into discretion. The use of Limiting Instructions: In these matters, The jury is told to consider the defendants prior conviction only on the issue of credibility and not on the overall issue of guilt The court is weary of limiting instructionsthe jury cannot be expected to perform a mental gymnastic which is beyond, not only their powers but anybody elses. Convicts/Felons are automatically prejudicial and this is difficult to manage even with a jury instruction.

Evidence Outline

Page 134

The nave assumption that prejudicial effects can be overcome by instructions to the jury, all practicing lawyers know to be unmitigated fiction. Therefore the balancing test that the court does is extremely important in these cases

Problem 8-B Hit the Deck There was a bank robbery. Two weeks later, Dennant is arrested and charged with robbery. Elmo is and the Ps witness and Farr is the Ds witnesses. Assume that Dennat, Elmo and Farr each have a prior conviction for bank robbery occurring within the last 8 years. As we have said, robbery does not fit into the automatic admissibility under FRE 609(a)(2). Therefore, as to each one of the convictions how do you apply FRE 609(a)(1)? Deference counsel would object to the convictions of Dennant and Farr, while Ps counsel would object to Elmos conviction. There is no doubt each party has standing to object to the admission of these prior convictions, since the purpose in each is to harm the case of the party who called the witness. The evidentiary hypothesis for this would be: (i.e.: the probative value of the prior conviction): A person who would commit robbery would also lie under oath. Someone who commits a felony crime through the use of deceit is more likely than other witnesses to lie under oath. As to the Prosecution witness, Elmo: The P would likely lose as to the admission of this evidence. He can prevail only by invoking the FRE 403, which means the conviction comes in unless probative worth is substantially outweighed by the risk of unfair prejudice. It seems unlikely that the jury would misuse the conviction. Anticipating this outcome, the P would be well-advises to bring out on direct examination that Elmo has the prior robbery conviction (See FRE 607) As to the Defendant, Ds conviction: The defense has a strong objection to Ds own conviction. Coupled with the fact that robbery is not high on the scale of veracity-related crimes, the big argument for exclusion is that the prior charged offenses are the same. Similarity of the offense is a strong danger of unfair prejudice! And therefore the potential for jury misuse is high. The jury is not supposed to use the conviction of D as proof of guilt, but it is hard to believe that wont happen. And Ds burden is eased by the fact that the Rule favors exclusion. Unless probative worth exceeds risk of prejudice, the conviction stays out. The argument to exclude is strong. As to the Defense Witness, Farrs conviction: the risk of prejudice here is more real than it is with Elmo. Farr is a friend of the defendant, and Farrs conviction may likely have a spill over effect, especially if the jury learns that D has also committed a prior robbery. All other things being equal, the risk of prejudice to D seems less with the Farr conviction than with Ds own, but there is a risk nonetheless. Gordon FRE 609(a) Factors: (1) the nature of the conviction, (2) its recency or remoteness, (3) whether it is similar to the charged offense, (4) whether defendant's record is otherwise clean (convictions are presumably more

Evidence Outline

Page 135

probative of credibility if they show a continuing pattern rather than isolated instances), (5) the importance of credibility issues, and (6) the importance of getting the defendant's own testimony Problem 8-C A History of Lying Ryan is charged with embezzlement of union funds. Wendy is the principle government witness, and she testifies that she saw couriers from the law firms deliver envelopes apparently containing cash to Ryan. Ryan testifies that the envelopes just contained legal documents. After Wendys testimony, D wants the jury to hear the Ps witness has a long history of lying. She has a rap sheet of convictions! Witnesses Convictions: 1. Displayed false handicap sign in car: this is an act of dishonesty or false statement 609(a)(2)automatic admissibility. This crime is completely dishonest. It is lying saying she was handicap. This dishonesty was an element of the crimefalsely displaying a handicap sign. 2. Felony conviction for obstructing justice: this also seems like it is a deceitful nature which is tampering with witness testimony. By its very elements (which require proof of a false statement and misleading conduct) it is an element of the offense which goes to truth telling which fits in the automatic admissibility language. Even if it is not automatically admissible, it is a felony and will pass muster under FRE 403. 3. Misdemeanor for peti-theft: not admissible because truth telling is not an element and this is a misdemeanor! (Does not fit under 609(a)(1). How could you try and argue this could fall under 609(a)(2)? The ACN tells you, you may use the charging documents and/or the instructions given to the jury if those define the crime as being dishonest or lying. Therefore, even if a generic statute does not define truth or falsity, if a charging document does, than it would be admissible. It readily can be determined that establishing the elements of the crime required proof or admission ad an act of dishonesty or false statement by the witness. Defendants Convictions: 4. Felony conviction for forgery: this squarely fits under 609(a)(2) because the crime requires proof of dishonesty or false statement in the form of the deceit that accompanies any act of claiming that something was created or signed by someone other than the defendant, when in fact he made it or signed itself. 5. Felony conviction for obstructing justice through physical force (as opposed to obstruction of justice by deceit) to influence or delay. This is not an automatic admissibility. However, because it is a felony it may fit under FRE 609(a)(1). Judge will use the reverse balancing test to determine if the probative worth outweighs the unfair prejudice. (remember, this standard favors exclusion) 6. Misdemeanor for peti theft altering the electrical meter attached to his house. The elements of the crime do not include dishonesty or false

Evidence Outline

Page 136

statement and it is a misdemeanor so it likely will not fit under 609(a) (1). On the other hand, the charging document and jury instruction will refer to rigging an electric meter to create a false reading, which does require proof of deceit which might bring the conviction within FRE 609(a)(2). What constitutes a criminal falsy? (i.e.: automatically admissible under FRE 609(a)(2)) (FN 6) 1. If on the exam Malavet says a crime is not a criminal falsy (automatically inadmissible), then we cannot apply 609(a)(2) and we have to go back to 609(a)(1). Remember, 609(a)(1) only applies to a FELONY! FRE 609(a)(1) does not apply to misdemeanors. 2. If Malavet gives us one of these crimes on the test, we must assume they are a criminal falsy. Forgery Bribery Falsifing a police report Mail Fraud 3. If Malavet gives us one of these on the test, it is likely not a criminal falsy: Prostitution Rape Narcotics Public intoxication Assault Looking at Charging Documents or Jury Instructions from previous convictions Looking behind a theft conviction to decide whether the particular facts involve dishonesty or false statement under FRE 609(a) (2) is consistent with the mandate of Lipscomb to consider background information in applying FRE 609(a)(1). The notion of allowing this to occur in front of the jury however, is absurd, unless the defendant opens the door by testifying and attempting to justify himself. Problem 8-DFaker, Thug? (Malavet thinks this problem is not important) Allen is charged with burglary. During cross examination, the P asks him if he falsified federal documents which led to a tax fraud prosecution and a felony charge. The P wants to bring out that Allen: 1. Claimed an exemption on his tax return for a child even though he didnt have one 2. Claimed a deduction of $4,000 of mortgage interests though he live in a rented house 3. Claimed a deduction of $800 for charitable gifts. Allen objects arguing that the cross examination under FRE 609 is limited to the fact of conviction, the date and place and the sentence imposed.

Evidence Outline

Page 137

If events of the past have led to conviction, FRE 609 applies, (not FRE 608) and the cross-examiner should confine his questions accordinglymeaning the questioning is restricted. o Malavet says you are allowed to ask date of conviction, name of the crime (and sometimes not even that) and sentence. Underlying facts are almost never admissible. But they will be admissible if the D opens the door. (i.e.: to contradict D) o FRE 609 precludes defense from cross-examining government witness on facts of the underlying offense. But, in rare situations, courts may hold the trial court has the discretion to allow questioning of the witness about the underlying facts of convictions under FRE 608(b). o Note, this is more likely to be allowed in the examination of crucial verses non-crucial witnesses. The defense has a sixthamendment confrontation right to inquire into the credibility of the state's witnesses most important or "crucial" witnesses, and that includes asking about misconduct under 608(b). Non-crucial witnesses provide a less-compelling need for 608(b)/confrontation questioning. VI. BAD
REPUTATION OR OPINION FOR TRUTH AND VERACITY/CREDIBILITY

Parties can introduce testimony by a character witness that the witness in question (the principle witness) is untruthful. FRE 608(a) authorizes testimony of this sort. However, this testimony is limited to opinion" as well as "reputation" testimony, meaning that the character witness may say what he personally thinks of the veracity of the principal witness. In order to testify to opinion testimony, the foundation is neededthe party must establish there is a period of personal acquaintance. This type of testimony is generally limited to lay witnesses. But there are exceptions: o One case admitted defense psychiatric testimony on the mental condition of the main prosecution witness. (very rare) o Some courts admit expert testimony concerning the trauma suffered by victims of rape or child abuse, and such testimony sometimes has the effect of bolstering credibility. Prior Inconsistent Statement Rule 613. Prior Statements of Witnesses (a) Examining witness concerning prior statement. In examining a witness concerning a prior statement made by the witness, whether written or not, the statement need not be shown nor its contents disclosed to the witness at that time, but on request the same shall be shown or disclosed to opposing counsel. (b) Extrinsic evidence of prior inconsistent statement of witness. Extrinsic evidence of a prior inconsistent statement by a witness is not admissible unless the witness is afforded an opportunity to explain or deny the same and the

Evidence Outline

Page 138

opposite party is afforded an opportunity to interrogate the witness thereon, or the interests of justice otherwise require. This provision does not apply to admissions of a party-opponent as defined in rule 801(d)(2). If a witnesses testimony differs on some point from her prior statements, the attacking party may cross-examine on these statements and (subject to some conditions) prove them by "extrinsic evidence" (i.e.: testimony by other witnesses) If a prior inconsistency is proved by "extrinsic evidence," generally the witness must have an opportunity "to explain or deny" it (unless "the interests of justice otherwise require"), and the adverse party (usually the one who called the witness) must have a chance to interrogate her. Note that sequence is not specified:

Generally, prior inconsistent statements are found in the following: depositions Prior testimony before a grand jury or in preliminary hearings; insurance investigators who have seeked out witnesses and obtained formal written or recorded statements investigators sent out by lawyers who obtain statements Problem 8-E: He is trying to Sandbag Us In a civil suit a man named Clemton (P) claims Dirk hit him with a shovel (D). Ps witness (Welch) testifies there was no provocation. On cross, D does not bring up the provocation issue. Then during the Ds case in chief Ds witness Murphey (a police officer who investigated the incident) testifies that Welch told him that he did not actually see the blow being struck, and thinks Clemton may have thrown a rock at D. Murphey is testifying as to the oral assertions of Ps witness Welch. This is extrinsic evidence of a prior inconsistent statement. However, this is the non-truth use of impeachment by a prior inconsistent statement. This is being offered for the impeachment value, not the truth use. Problem: The P claims Welch is gone and has no opportunity to respond to the suggestion that he changed his story. D had a chance to put the same question to Welch, and deliberately did not. He should have laid the foundation back then. Rule 613(b) requires this. Rule 613(b) is not admissible unless the witness is afforded an opportunity to explain or deny the same and the opposite party is afforded an opportunity to interrogate the witness thereon. Therefore Welch should have a chance to explain/respond to this statement. Note, under FRE 611 (which grants plenary authority in the judge to control the mode and order of interrogating witnesses and presenting evidence) the court could have chosen to demand disclosure of impeachment or the possibility of extrinsic evidence directly following the admission of the extrinsic evidence, but it did not. However, using their discretionary powers, they chose not to. The way FRE 613 will apply is if the witness is still available to testify. FRE 613 will not be admissible if the witness dies or becomes unavailable to testify. Basically, if the witness does not have an opportunity to

Evidence Outline

Page 139

rebut/respond to the evidence, then the evidence cannot be admitted. If Welch is not available in this case, the testimony by Murphy is not admissible. Because Murphys evidence has already been said there are a few options 1. P should move for a mistrial 2. P should move for the evidence to be stricken from the record, and you should tell the jury to erase it from their memory 3. In the interest of justice, the court can leave it in the record 613(b). Therefore, the D here is making a tactical decision which is at risk of being stricken/mistrial buy not cross-examining the witness immediately after Welchs testimony. VII. CONTRADICTION

Contradiction is not explicitly discussed under the rules. But it is simple logic that a witness cannot be allowed to go unchallenged when there is evidence that directly contradicts what he or she has said on the stand. it needs to be determined based on the weight of the evidence (using the principles of FRE 403) Impeaching a witness by contradiction entails a showing that something he said in his testimony is not so. This can be accomplished on crossexamination, as questions force the witness to admit that he erred (even lied) on some point, but often it is accomplished by extrinsic evidence (testimony or something else, like a writing or recording), which for convenience we may call counterproof. Note that if we label extrinsic evidence as counterproof, we are allowing its use for the form of impeachment known as contradiction. If we label it "collateral" we are excluding it. If you can find some relevance the testimony will have in the case, this is counterproof and not merely collateral. Difference Between Counterproof and Collateral: All contradicting counterproof has some impeaching effect but you can let it in if it has additional relevance in the case-some relevance independent of its contradicting effect. Three kinds of counterproof may be discerned in the cases: 1. Counterproof that not only contradicts but also tends to prove a substantive point 2. Counterproof that not only contradicts but tends to prove some other impeaching point, (for example bias) 3. Counterproof that only contradicts. This=collateral and the evidence is usually excluded, for it has no relevancy apart from contradicting the witness. To avoid wasting time on trivial matters, we bar impeachment on collateral matters, admitting only counterproof that has dual relevancy in the case. FRE 403 and FRE 611 are read to limit this form of impeachment. Contradiction and FRE 608(b)

Evidence Outline

Page 140

Note that FRE 608(b) provides that an attacking party may cross-examine a witness on non-conviction misconduct "if probative of . . . untruthfulness" but that such misconduct "may not be proved by extrinsic evidence." Doesn't that mean that if the witness denies the misconduct, the attacking party is simply stuck with the answer? Doesn't FRE 608(b) in effect tell us that we cannot engage in contradiction in this situation? FRE 608(b) regulates only one mechanism of impeachment, not others: It speaks to impeachment by showing untruthful disposition, not impeachment by contradiction. Malavet considers there to be three types of contradiction: 1. Substantive: it not only contradicts, but it also tends to prove a substantive point; goes to relevanceit has high importance to your case; it goes directly to the issue/crime 2. Something else: it not only contradicts, but tends to prove some other impeaching point (such as bias) 3. Collateral: it has no relevance apart from contradicting the witness. (no extrinsic evidence allowed) Problem 8-F Thats Just Collateral Oswald is charged with a robbery that happened in Seattle on July 14 at 7 p.m. He is claiming an alibiArdiss testifies to this point as principal witness during the defense case-in-chief. Ardiss testifies that he operates the restaurant in Portland, that Oswald is a regular customer, and that he was there for the entire evening on July 14. Alibi= At the time of the crime, I was in Portland, not Seattle. This seems like a good alibi. Prosecution asks if Oswald was in the restaurant during the weeks prior to July 14, or if he was gone for occasional periods of 3 and 4 days. Ardiss answered that he was in there every day during that time. During the states case in-rebuttal, the prosecutor calls police detective Kinney. Kinney testifies that he saw Oswald on July 27 and that Mr. Oswald told him he had been in Seattle for the last couple of days. Testimony by officer Kinney seems excludable under the collateral matter bar. Kinnys testimony does not seem to prove a substantive point, so it is collateral as opposed to contradicting counterproof. Samules, a waiter in the bar, testifies that Oswald was not there on the 14th. Samuels should be allowed to testify. This testimony not only contradicts, but supports the testimony that Ardiss was not there and therefore his alibi collapses. Contradicting Ardiss should be allowed because he is the main witness for the defense. Your ability to contradict a main critical witness is admissible. The timing rules: 1. Courts disallow any attempt to repair credibility before the attack has come. In the specific context of character witnesses, FRE 608(a) states this principle directly.

Evidence Outline

Page 141

2. Rule 609 contains no limitation that precludes a party from offering character evidence under circumstances where it anticipates an impeachment however the event that triggers the rule is an attack on the witnesses veracity. 3. However, it is permissible on direct (1) for the prosecution or the defense to bring out that its witness has been convicted of crimes, (2) for the prosecutor to bring out that its witness has entered into a plea bargain, and (3) for the calling party to bring out any connect on or affinity that she has with the witness (obvious grounds of bias that the other side would raise). 4. This does not deprive the cross-examiner of the right to ask about impeachment information because the direct asked about it first. Some questioning on cross would still be appropriate. Ex: "Remind me once again now, you said you have been twice convicted of forgery, is that right?"

OPINION AND EXPERT TESTIMONY


With opinion/expert testimony you are establishing the distinctions between FRE 602 and FRE 701 or FRE 701 and FRE 702. I. LAY OPINION TESTIMONY Rule 701. Opinion Testimony by Lay Witnesses If the witness is not testifying as an expert, the witness' testimony in the form of opinions or inferences is limited to those opinions or inferences which are (a) rationally based on the perception of the witness, and (b) helpful to a clear understanding of the witness' testimony or the determination of a fact in issue, and (c) not based on scientific, technical, or other specialized knowledge within the scope of Rule 702. Lay Witness opinion is admissible providing: o That the witness has personal knowledgethe witness observed something that is being described and can formulate a common sense, rational opinion or inference. o That the opinion is helpful to the trial of fact o The witness cannot have a legal opinion (i.e.: they cannot say I believe the defendant was grossly negligent)

Witnesses Testifying to their subjective opinions Lay witnesses are not allowed to testify to their subjective opinions and inferences about what someone said. The lay expressions of opinion or inference may be permitted but only if they are objectionably, rationally based on perception of a witness and are helpful either to the understanding of the testimony of the witness on the stand or helpful to the determination of a fact in issue. That is witnesses are

Evidence Outline

Page 142

allowed to interpret what their friends told them, but it has to be objectionably, rationally based. The rationally-based conclusion by the witness has to be reasonable. The reasonableness depends on the situation. It will depend a lot on the particular relationship between the partiesa particular relationship may in fact put a witness in a better position to understand what the person meant and the conclusion will be rational. (See problem 9-A)

Problem 9-A It was My Impression Cox is on trial for unlawful detonation of explosives. The P calls Coxs ex-girlfriend who testifies that Cox told her twice that he would blow up cars for $50 and that he showed her a newspaper account of one of the bombings giving rise to the present charges. The P asked the ex-girlfriend if Cox admitted to being involved in the bombing. She answered that he never actually said that but it was her impression that his actions inferred he had blown it up. The defense objects saying it is opinion and calls for speculation. Answer: Probably the testimony should not be allowed as not rationally based on her perceptions, although the question is close. Most likely, the witnesses understanding will not aid the jury in their understanding of what Cox said and did. The problem is that her rather broad but definite conclusion goes to the heart of the case and rests on a thin factual basisintuition more than logic. On a point further from the heart of the case probably she could express such a conclusion. She might well testify, for example, that she was upset about the news clipping (a less definite conclusion that must still point toward some involvement by the defendant.) Witnesses are allowed interpret what friends told them, but the inference should be stronger than the one suggested by these facts. Opposing Argument: It is rationally based on her perceptions based on what she knows about the defendant. This is a legitimate rational conclusion. These are words that, although were not an outright confession, were rational, in the context of their relationship meant a confession. If you can establish there was a long term relationship, and that they communicated in a particular way, such that there was a code between the couple that only they can understand, it is possible the ex girlfriend could come to this conclusion rationally. In that case, it would be considered a confession that was objectively rational. Problem 9-B- The Watchful Neighbor There was a car accident between Pinkston and David. Hanson, an eyewitness, is seeking to testify to things that he has inferred or concluded from what he observed. Examples: He testifies Pinkston was backing out of her car to take her daughter to ballet lessons. This is a conclusion, not an observation, because how could he know they were going to ballet lessons? However, it is not important to the case. He testifies the pickup (Daviss car) plowed right into the side of the car. This is a point of substantive importancesuggesting Davis was at fault: plowed seems to get you closer to a conclusion. However, this is a

Evidence Outline

Page 143

rationally based conclusion as long as an attorney lays the foundation that the witness had position to determine how the accident occurred, that he drives all the time and has a normal level of experience with driving, etc. He testifies the pickup was going well over 35 mph. This could also be a conclusion, however it is rational--a lay person who drives all the time would be able to answer that; Driving at such speed, and seeing others doing so, are matters of common experience and well within the capacity of grown lay persons to estimate (FRE 701, FRE 702) He testifies you can only go 20 mph at a school zone which is where the car was. This is a question of law, which the witness cannot properly testify to. This is closer to expert legal testimony, but you can show this is within the normal level of knowledge a reasonable adult would have, if you can prove there are lots of posted signs that say this, or that everyone who takes the driving test must answer this question then you may establish this is within the normal knowledge of a lay person. He testifies he would guess the driver of the car is Davis-- he is not sure if the person he saw driving the pickup is in the court room. If the witness can do no more than guess, he may be limited to describing the driver the best he canhe cannot guess. He needs to stick to what he knows or thinks to be true. A lay person has the capability of recognizing people even though there has been some changes in their appearance. Therefore, if the witness can describe certain characteristics which describe Davis, this is ok. He testifies there was a strong smell of pot and was sure the driver was smoking pot. This could go either way. This could be in the personal knowledge of a lay person (but think of how this may go over with a jury). However, the way pot smells is not considered a part of common human experiencei.e.: the smell of marijuana requires expertise. He testifies Pinkerson had a guilty look The witness can testify to the apparent emotional state of Davis, however be careful of speculation (ex: Davis feared license suspension and a lawsuit) He testifies Pinkerson was upset about the little girl Appraisal of emotional condition is proper. He testifies as to what injuries Pinkerton had claiming Pinkerton looked like she had a broken back. Unless the witness is a doctor, he cannot testify to a medical diagnosis. But he can testify that the little girl was lying prone, and that any movement of her torso produced a response of pain, or that she seemed unable to move her arm. He testifies the car was totaled with damages that looked like they were $5,000. He can describe physical damage, though the conclusion that the car was totaled may need an expert opinion. Clearly, the witness cannot testify to a dollar figure, unless he has expertise on this He testifies he looked like she was in a hurry He can express this conclusion if he saw things which would reasonably lead him to conclude she was in a hurryi.e.: that he saw her enter the car in a rush, backup quickly, she didnt turn around to look, etc.)

Evidence Outline

Page 144

He testifies David did all he could do to avoid collision and could not have stopped in that short space. These conclusory appraisals are correct if he saw the whole incident, however it could be objected to if opposing counsel can show that it is not rationally based. II. EXPERT TESTIMONY

In evaluating the admissibility and the content of expert testimony you must consider FRE 702, 703, 704 and 705. Also keep in mind FRE 615[B](3) (allowing certain necessary witnesses to remain in the courtroom during other witnesses' testimony). Rule 702. Testimony by Experts If scientific, technical, or other specialized knowledge will assist the trier of fact to understand the evidence or to determine a fact in issue, a witness qualified as an expert by knowledge, skill, experience, training, or education, may testify thereto in the form of an opinion or otherwise, if (1) the testimony is based upon sufficient facts or data, (2) the testimony is the product of reliable principles and methods, and (3) the witness has applied the principles and methods reliably to the facts of the case. Rule 703. Bases of Opinion Testimony by Experts The facts or data in the particular case upon which an expert bases an opinion or inference may be those perceived by or made known to the expert at or before the hearing. If of a type reasonably relied upon by experts in the particular field in forming opinions or inferences upon the subject, the facts or data need not be admissible in evidence in order for the opinion or inference to be admitted. Facts or data that are otherwise inadmissible shall not be disclosed to the jury by the proponent of the opinion or inference unless the court determines that their probative value in assisting the jury to evaluate the expert's opinion substantially outweighs their prejudicial effect. Four basic requirements for expert testimony: 1. The subject matter must be appropriate for expert testimony Helpfulness/Assistancethe expert opinion must be helpful to the trier of fact and "assist the trier of fact to understand the evidence or to determine a fact in issue." Reliability: methodology underlying the expert opinion must be reliable. Relevant: it must fit the facts of the case Where expertise is only marginally helpful because the subject is simple or familiar, special education or experience may add little or nothing to common jury understanding, but in such cases a decision excluding such testimony might better rest on FRE 403. As a condition to admissibility, the proponent of expert testimony must convince the judge by a preponderance of the evidence that the methodology is reliable and that it is relevant in the sense it fits the facts of the case. 2. The witness must be qualified as an expert

Evidence Outline

Page 145

The qualifications need not be formal or academic. It can be based on skills. The standard is intended to be lenient. A person with suitable training or education may qualify even if he is not a specialist or not renowned, and even if he lacks a certification or experience. Rule 702 embraces people with practical experience but no formal training. 3. The expert should possess reasonable certainty or probability regarding the opinion The opinion must be more than mere guess work. It must be something more than mere speculation (ex: a Drs opinion must be based on reasonable medical certainty.) 4. The opinion must be supported by a proper factual basis provided they are reasonably relied upon by experts in the particular field. Three acceptable basis for expert testimony: Firsthand-knowledge: Facts within the personal knowledge of the expert that he knows of before the hearing. This is basically what is required of lay witnesses. (ex: the treating physician who examined the patient provides an opinion as to what was wrong with the patient) Facts learned at trial: Facts not within personal knowledge, but facts which are supplied to the expert in court by the evidence. (This could be testimony heard by an expert while sitting in the courtroom or information conveyed in a hypothetical summing up what has been previously admitted.) Outside Information/Data: Facts that are of a type that experts in that field would reasonably rely upon in making out of court professional decisions. The expert could base his opinion on hearsay, as long as he would reasonably rely upon it out of court in making professional decisions. This is generally information the expert gleans before trial by consulting other sources. FRE 403 is in play here: we face the difficulty of admitting evidence susceptible of misuse of the information. Problem 9-CThey saw it the same way I did The expert (Dr. Weaver) wants to testify as to what other urologists (11 colleagues) have said. What the Expert is allowed to testify to: She is allowed to testify to the standard of care in Tampa, because she has personal knowledge of that. (the prevailing standard of care in this area) She practices there as a urologist, and surely has insight and knowledge based on training and experience that help her address this matter. She can use knowledge from other experts to generate her own opinion as long as it is her own opinion. We allow her opinion to be based upon inadmissible hearsay (opinion of other people). She can read treatises, books, and consult with other experts, but she cant tell the jury what these outside sources said (i.e.: you cannot say the other urologists share my viewYou cannot say other experts agree with you or are vouching for you)

Evidence Outline

Page 146

What the Expert is NOT allowed to testify to: The court says that unlike the other experts, Dr. Weaver became a conduit for inadmissible hearsay (the opinions of other experts). You cannot turn other expert opinions into your evidence. You can merely use them for your own opinion. You cannot acquire knowledge specifically for a case and then use that knowledge of others to relay that to the jury, Cannot testify to the ultimate issue (i.e.: Cannot say that a Dr. breached the standard of care in Tampa.) (ex: an expert could not testify that decedent had "capacity to make a will" but could testify that he knew "the nature and extent of his property and the natural objects of his bounty") The reasoning behind this is the same policy reasons for hearsayyou cant cross examine the other witnesses as to the basis of that opinion. This creates too much of an inference that the experts agree with her without them being available for cross examination. You cannot substitute a treatise opinion or another Dr.s position as your own. Other opinions can only be used to supplement the experts opinion Bottom Line: An expert must give her own opinion even if she relies on others. An expert cannot bolster his or her testimony by testifying that a particular treatise or another Dr. supports an opinion. This is considered hearsay opinions. An expert may testify if she relies in part on consultations with other experts, but still the expert may not be used as a conduit. Using a Text Treatise or Article: Can you get into evidence the content of a treatise or text of whatever discipline is at issue in the case? The major problem here is the hearsay rule. (This is a form of expertise that is in writing made out of court). You CAN use it to impeach an expert (but you have to first establish it is reputablecan show in 4 main ways:) 1. Opponents expert actually relied on the treatise in his direct examination 2. Elicit an admission on cross examination (ex: Dr. are you familiar with this treaty? Is it a standard work in the field?/Is it reliable?) 3. You may call your own expert witness who says the treatise is reliable 4. Judicial noticethe judge can take judicial notice that this treatise/book is a standard book in the field. Ex: I represent the P who is suing the D alleging the Ds negligence caused serious injury to the client. The P has holes in the head. In Greys Anatomy it says having a hole in your head adversely affects life expectancy. The expert is claiming holes in your head do not have this effect. You can use this to impeach or rebut your opponents expert. You are not offering the content for its truth, you are offering it to show the authorities arent all the same way. Therefore it comes in to impeach the credibility or impeach your opponents expert. The federal rules have liberalized this standard: You can use a treatise in support of your own expert. Your own expert can say the text is reliable and can read from the text the part that supports the opinion. The content is admissible for its truth (the learned treatise exception to the rule against hearsay). If you

Evidence Outline

Page 147

establish the document is a learned text treatise or article will be admissible for its truth as an exception to the rule against hearsay. Two limitations: 1. There must be an expert on the stand. This information must be filtered through an expert witness (either your own expert in which the text will be supportive, or your opponents expert in which it will be contradictory) 2. The treatise does not go to the jury. It only comes into evidence by being read to the jury. Qualifying a witness 1. When a party calls an expert witness, usually the first questions establish that the matter at hand could benefit from expertise. Then comes the foundation. In the case of a professional person (such as a physician or engineer), usually the calling party brings out: educational background, including degree and perhaps certificate or license to practice, experience, such as employment or practice in the area to be covered by the questioning, and familiarity with the subject in suit. The pattern is similar (usually less elaborate) with skilled people having informal expertise resting on experience. 2. Qualifications are important, but you can have the most qualified expert in the world and her field still has to be relevant to the case and her opinion must still have a reasonable basis. 3. Deciding whether a person qualifies as an expert, the sufficiency and acceptability of the data and the acceptability of the science is a question for the judge under 104(a) 4. Note the difference between voir dire (presumptively to take place without the jury) and cross-examination (which takes place before the jury) in determining whether someone is qualified as an expert. 5. The court will rarely appoint expert witnesses III. SCIENTIFIC EVIDENCE Admitting Expert Testimony After Daubert: (Frye is dead: The question is how much of a lower standard have we allowed after Daubert) Preliminarily: The Judge decides whether the expert is qualified (FRE 104(a)). The judge must also determine if what the expert is testifying to is scientific knowledge that will assist the trier of fact. The reasoning or methodology underlying the testimony must be scientifically valid and the reasoning or methodology must be able to be properly applied to the facts in issue. (look at factors below that judge will rely upon in making his decision) 1. Relevance of Expert Testimony (FRE 401, 402) "Relevant evidence" is defined as that which has "any tendency to make the existence of any fact that is of consequence to the determination of the action more probable or less probable than it would be without the evidence." The Rule's basic standard of relevance thus is a liberal one. 2. Reliability of Expert Testimony [Daubert, FRE 702]

Evidence Outline

Page 148

Under the Rules the trial judge must ensure that any and all scientific testimony or evidence admitted is not only [1] relevant, (assist the trier of fact) but [2] reliable (an expert has scientific knowledge). (a) Qualified Expert (b) Valid Science 3. Balancing probative value and the six FRE 403 dangers. (FRE 704) This is always the final hurdle, but it should be kept in mind at every stage). Expert evidence can be both powerful and quite misleading because of the difficulty in evaluating it. Because of this risk, the judge in weighing possible prejudice against probative force under FRE 403 of the present rules exercises more control over experts than over lay witnesses. 4. Scientific Evidence: Assuming the experts are qualified THE JUDGE could: First, admit the evidence because the experts are qualified and let the jury resolve the dispute. (this is highly disfavored method, because you are basically putting a lot in the hands of the jury) Second, decide for yourself whether the plaintiffs proof is valid science and admit or exclude accordingly. (Daubert) o Malavet thinks this gives too much differential to the judicial community and too much difference on the jury to sort it all out. (However, FRE 701, 702, and 703 were all amended in 2000 to require the court to exercise more authority over expert testimony, at the admissibility stage. Third, defer to the broader scientific community for its judgment on the validity of the science, asking the proponent to show not only what the proof means and what it is, but also to show that scientists generally agree with it. (Frye). o Gives trial judge a way out o Frye allows the judge to say, "I'm not going to let everything in and I'm not going to resolve scientific disputes myself; if I exclude anything, I'll do it because other scientists don't accept the proof and I can tell the expert it's not my opinion that counts, but the verdict of other scientists." These cases liberalize the standard of what constitutes good science or good expertise under FRE 702In other words, the FRE superseded the Frye test of deferring to the scientific community. Daubert v. Merrell Dow Pharmaceuticals (overruled Frye v. United States) Children are born with birth defects because the mothers took a medicine which was told to cause. Here we have experts on both sides that were qualified under FRE 702. The validity of the science becomes the major questionpetitioners file a motion for SJ arguing the scientific knowledge put forth by the plaintiffs is not generally accepted practice (in other words, if the court applies Frye, the evidence should not be admitted because it is not generally accepted practice in the scientific community) District Court, using the Frye standard, concluded opinions that diverge significantly from the procedures accepted by recognized

Evidence Outline

Page 149

authorities in the field cannot be shown to be generally accepted as a reliable technique. SC held the Frye test is too restrictive. FRE 702 gives us the scientific reliability standard. For purposes of litigation, it has to be a legal evidentiary standard not a scientific one: meaning science does not have to be absolutely sure of the fact. We cannot afford, as a matter of public policy, to wait as long as scientists do to declare something science. (In this case, we arent going to wait until science says this pill definitely caused the birth defects) Daubert makes it a legal rule of reliability: the science must be reliable enough to assist the jury (it is a lower standard than the reliability called for in Frye.) It must meet the requirements of Rule 702, not the requirements of peer reviews. See the factors below. The trial judge acts as a gatekeeper, determining whether the proffered evidence is scientifically valid and relevant to the case at hand. It is up to the court to make this finding before the jury hears it. Cross examination and the presentation of contrary evidence will allow the jury to determine whether the proffered scientific evidence is ultimately credible. However, Florida is not a Daubert state. Frye is still the law in Florida. Florida has expressly declined to adopt more lenient Daubert standard."

Implementation, Scientifically Valid, Factors There is a distinction between scientific validity and scientific reliability. Scientific validity may be enough to make the testimony legally reliable. The Factors in Determining Scientific Knowledge: as noted above, an inference or assertion must be derived from scientific methods and be supported by appropriate validation. Factors (non exclusive) the court can analyze to determine whether a theory or technique is scientific knowledge: 1. Tested: Has the theory been tested? 2. Peer Review and Publication: Another pertinent consideration is whether the theory or technique has been subjected to peer review and publication. 3. Scientific reliability: does the application of the principle produce consistent results? 4. Control Standards: should consider the known or potential rate of error, and the existence and maintenance of standards controlling the technique's operation. They have to apply the right procedures and that those procedures were followed during the testing. 5. General Acceptance: Finally, "general acceptance" can yet have a bearing on the inquiry"general acceptance" is not a necessary a precondition to the admissibility of scientific evidence under the FRE-especially FRE 702, but it definitely helps! **These factors should be weighed/balanced at the FRE 403 stage Criticisms of Frye and Daubert: Frye excluded too much evidence that was probably good science. With Daubert, Courts are being flooded with "junk science," and are taking more responsibility for risk management than they could handle.

Evidence Outline

Page 150

Malavet says: o It is hard to deal with scientific evidence in courts of law o Judges have shown a reluctance to become, "amateur scientists." The Authors argue that while Daubert is not so bad, abuse of discretion review is. They would prefer de novo, or at least mixed law/fact review.

Kumho Tire Kumho Tire generally expands Daubert to apply to all expert testimony, not just to scientific evidence. The ruling: Extends the Daubert standard beyond the parameters of "scientific evidence" to all expert testimony Lays such stress on flexibility and "discretion" that it seems almost to dilute Daubert. In the case there was an automobile accident, the Ps sued Kuhmo Tires saying the tires were faulty and therefore caused the crash. Plaintiffs relied on deposition testimony from an expert in tire failure analysis. He claimed the tires failed and caused the crash. This testimony is clearly relevant because it goes to the improperly designed tire It is being given by a qualified expert The issue the court needs to determine is if this is accepted scientific methodology is considered scientific knowledge. The trial court ruled Carlson's testimony inadmissible because his methodology failed the reliability requirement of FRE 702 and Daubert, and granted summary judgment for the defense. The defense tried to argue the scientific knowledge requirement should not be used on non-scientific evidence. Court said no! The Rules grant that latitude to all experts, not just to "scientific" ones. FRE 702 Rule applies its reliability standard to all "scientific," "technical," or "other specialized" matters within its scope. The Rules grant that latitude to all experts, not just to "scientific" ones. It would be difficult, if not impossible, for judges to administer evidentiary rules under which a gatekeeping obligation dependeds upon a distinction between "scientific" knowledge and "technical" or "other specialized" knowledge. There is no clear line that divides the one from the others. The Rule, in respect to all such matters, "establishes a standard of evidentiary reliability." The trial judge must determine whether the testimony has "a reliable basis in the knowledge and experience of [the relevant] discipline." Flexibility in Judging Reliability A trial judge, in determining the "admissibility of an engineering expert's testimony" may consider several more specific factors that Daubert said might "bear on" a judge's gate-keeping. The factors which are considered depend on the specific type of issue you have before you determination. The gatekeeping inquiry must be "tied

Evidence Outline

Page 151

to the facts" of a particular "case, depending on the nature of the issue, the expert's particular expertise, and the subject of his testimony." Generally, most fields are acceptable technical areas and acceptable science. There are exceptions: Stuff that has been put into question Junk sciencesuch as speaking to the dead, ESP, and Science we are still skeptical of: Lie Detecting Tests; truth telling Abuse of Discretion Standard: A court of appeals is to apply an abuse-of-discretion standard when it "reviews a trial court's decision to admit or exclude expert testimony." This means a trial judge has the discretionary authority, reviewable for its abuse, to determine reliability in light of the particular facts and circumstances of the particular case. (FRE 702) Basically, the trial court is fine as long as their holding is not clearly erroneous. In Kuhmo, the SC held the District Court did not abuse its discretionary authority. Scalia, OConner and Thomas concurred in the opinion, agreeing that trial judges have discretion to "choose among reasonable means" of appraising science; they do not have discretion to perform the gatekeeping responsibility "inadequately." This seems to suggest that abuse of discretion review may not be appropriate in this area. They think de novo review would be more appropriate. (i.e.: determining the case as if no prior trial had been held) They claim you should defer to the trial court on what the relevant factors should be used, but review whether the trial judge came to the right conclusion based on those factors. The trial court is in a better position to determine what factors to consider. But in the decision regarding whether the factors match the conclusions the trial court came to, the appellate court should be allowed to make its own conclusion on de novo review. Notes from Review: Abuse of Discretion verses De Novo Err on the side of using the abuse of discretion review unless the judge has made a decision as a matter of law. The more subjective the decision is, the more likely an abuse of discretion review will be used. Malavet says we have read enough to know the court sort of willy nilly decides what standard they are going to use depending on if they want to affirm or reverse. To AFFIRM: This was a factual finding and we are therefore deferring to the factual finding of the court TO REVERSE: This was a finding as a matter of law On the test, Malavet will say this is the result you SHALL reach. So if you need to Affirm you would want to use the abuse of discretion. If you want to reverse, you would want to use De Novo. 104b Credibility-whether something was said

Evidence Outline

Page 152

Prior Bad Acts-whether the act was actually done. Bourgaly and Huddleston only deal with situations where you have predicate facts (independent facts that need to be established before a piece of evidence is admitted.)

Whether you use 104b or 104a will determine who decides who whether the predicate facts occured.
Preponderance standard under 104(a) Its more likely than not this predicate fact happened. Scintilla standard under 104b A reasonable jury could find this (predicate fact) happened.

Evidence Outline

Page 153

You might also like